Multiple Choice: 12-1: D. This Is Recorded When The Working Fund Is Replenished. 12-2: C

You might also like

Download as pdf or txt
Download as pdf or txt
You are on page 1of 170

CHAPTER 12

MULTIPLE CHOICE

12-1: d. This is recorded when the working fund is replenished.

12-2: c.

Sales P 700,000
Cost of goods sold:
Purchases P800,000
Merchandise inventory, end 180,000 620,000
Gross profit P 80,000
Expenses 198,000
Net income (loss) P (118,000)

12-3: b

Sales P 70,000
Cost of goods sold (P70,000 / 140%) 50,000
Gross profit P 20,000
Less: Samples (P8,000 – P6,000) P 2,000
Expenses 2,800 4,800
Net income P 15,200

12-4: a

Sales P 100,000
Cost of goods sold 72,000
Gross profit P 28,000
Expenses (P9,000 + P4,500) 13,500
Net income P 14,500

12-5: a

12-6: a

12-7: c

12-8 a

Shipment of merchandise to home office P 80,000


Equipment sent to home office 50,000
Expenses assigned to branch by the home office 8,000
Cash remittance to home office (40,000)
Home office account balance P 98,000

1
12-9: d

12-10: a

Home Office account balance before closing, Dec. 32, 1008 P 35,000
Net income (loss)
Sales P147,000
Cost of cost goods sold
Shipment to branch P135,000
Inventory, 12/31 18,500 116,500
Gross profit P 30,500
Expenses 13,500 17,000
Home Office account balance (Investment in Branch account balance) P 52,000

Shipment to Branch account has no beginning balance, because this was closed at the end
of 2008.

12-11: b
Jan. 1, 2008 Jan. 1, 2009
Petty cash fund P 6,000 P 6,000
Accounts receivable 86,000 98,000
Inventory 74,000 82,000
Home Office account balance P166,000 P186,000

12-12: d

(Branch Books) (Home Office Books)


Home Office Investment in Branch
Unadjusted balances, Dec. 31 P 21,320 P 38,600
Remittance in transit (10,400)
Shipment in transit 7,280
Cash collections of home office ( 400)
Adjusted balances, Dec. 31 P 28,200 P 28,200

12-13: a

Unadjusted balance – Investment in Branch account, 12/31 P430,000


Charge for advances by president (5,500)
Erroneous entry for merchandise allowance ( 600)
Share in advertising expense (9,000)
Unadjusted balance – Home Office account, 12/31 P414,900

2
12-14: a
(Branch Books) (Home Office Books)
Home Office Investment in Branch
Unadjusted balances, 12/31 P 97,350 P 84,000
Shipment in transit 6,150
Collection of HO A/R by branch 25,000
Error in recording of branch profit 900
Returns of merchandise in transit ( 6,400)
Adjusted balances, 12/31 P103,500 P103,500

12-15: a
(Branch Books) (Home Office Books)
Home Office Investment in Branch
Unadjusted balances P25,550 P27,350
Error in recording shipment to Cavity branch (12,000)
Error in recording shipment to Tagaytay branch 15,000
Branch AR collected by home office (3,000)
Merchandise returns in transit ( 1,200)
Error in recording branch profit ( 3,600)
Adjusted balances P23,750 P23,750

12-16: c
Unadjusted balance- Investment in Branch account P 85,000
Remittance in transit (10,000)
Shipment in transit (20,000)
Expenses allocated ( 5,000)
Error in recording remittance 3,000
Error in recording shipments ( 9,000)
Unadjusted balance – Home Office account P 44,000

( Branch Books) (HomeOffice Books)


Home Office Investment in Branch
Unadjusted balances, P 44,000 P 85,000
Remittance in transit (10,000)
Shipment in transit 20,000
Expenses allocated 5,000
Unrecorded HO collection of AR (3,000)
Error in recording shipments 9,000
Adjusted balances P 75,000 P 75,000

12-17 a
(Branch Books) (Home Office Books)
Home Office Investment in Branch
Unadjusted balances P 440,000 P 496,000
Branch AR collected by Home Office ( 8,000)
Shipments in transit 32,000
Acquisition of furniture (12,000)
Merchandise returns (15,000)
Cash remittance in transit ( 5,000)
Adjusted balances P 464,000 P 464,000

3
PROBLEMS

Problem 12-1

Home Office Books Branch Books

1. Investment in branch 30,000 Cash 30,000


Cash 30,000 Home office 30,000

2. Investment in branch 75,000 Shipment from home office 75,000


Shipment to branch 75,000 Home office 75,000

3. No entry Purchases 10,000


Accounts payable 10,000

4. No entry Accounts receivable 125,000


Sales 125,000

5. Shipment to branch 2,000 Home office 2,000


Investment in branch 2,000 Shipment from home office 2,000

6. No entry Cash 105,000


Accounts receivable 105,000

7. No entry Accounts payable 7,000


Cash 7,000

8. No entry Salaries 10,000


Rent 5,000
Utilities 2,000
Other operating expenses 12,000
Cash 29,500

9. Investment in branch 7,500 Depreciation 1,500


Accumulated dep’n 7,500 Rent 5,000
Insurance 1,000
Home office 7,500

10. Cash 65,000 Home office 65,000


Investment in branch 65,000 Cash 65,000

11. Cash 3,000 Home office 3,000


Investment in branch 3,000 Accounts receivable 3,000

12. Investment in branch 10,000 Sales 125,000


Branch income 10,000 Inventory, end 5,000
Shipment from HO 73,000
Purchases 10,000
Salaries 10,000
Rent 10,000
Utilities 2,000
Other operating expenses 12,500
Income 10,000

4
Problem 12-2

a. Books of the Branch

1. Cash 200,000
Merchandise inventory 350,000
Home office 550,000

2. Merchandise inventory 400,000


Accounts payable 400,000

3. Accounts receivable 650,000


Sales 650,000

Cost of goods sold 425,000


Merchandise inventory 425,000

Cash 600,000
Accounts receivable 600,000

4. Advertising expense 40,000


Sales commission 65,000
Other expense 45,000
Cash 150,000

5. Accounts payable 370,000


Home office 120,000
Cash 490,000

b. Manila Sales – Naga Branch


Income Statement
Year Ended December 31, 2008

Sales P650,000
Cost of goods sold 425,000
Gross profit 225,000
Expenses:
Advertising expense P40,000
Sales commissions 65,000
Other expenses 45,000 150,000
Net income P 75,000

c. Manila Sales – Naga Branch


Balance Sheet
December 31, 2008

Cash P160,000 Accounts payable P 30,000


Accounts receivable 50,000 Home office 505,000
Merchandise inventory 325,000
Total assets P535,000 Total liabilities and capital P535,000

5
Problem 12-3

Home Office Books Branch Books


(1) Adjusting Entries

a. Investment in branch 63,750 Cash 63,750


Cash 63,750 Home office 63,750

b. Investment in branch 75,300 Shipment from HO 75,300


Shipment to branch 73,300 Home office 75,300

c. Accounts receivable 157,500 Accounts receivable 99,000


Sales 157,500 Sales 99,000

d. Purchases 183,750 Purchases 33,750


Accounts payable 183,750 Accounts payable 33,750

e. Cash 170,400 Cash 80,100


Accounts receivable 170,400 Accounts receivable 80,100

Home office 80,100


Cash 80,100

f. Accounts payable 186,000 Accounts payable 18,375


Cash 186,000 Cash 18,375

g. Expenses 39,900 -
Cash 39,900

Furniture & fixtures – branch 12,000 Home office 12,000


Investment in branch 12,000 Cash 12,000

h. Cash 80,100 -
Investment in branch 80,100

Expenses 27,000
Cash 27,000

i. Retained earnings 15,000


Cash 15.000

(2) Adjusting Entries

j. Expenses 1,750
Acc. Depreciation 1,750

k. Investment in branch 975 Expenses 975


Acc. Dep’n – Br. F & F 975 Home office 975

l. Prepaid expenses 375 Prepaid expenses 1,125


Expenses 375 Expenses 1,125

m. Expenses 150 Expenses 450


Accrued expenses 150 Accrued expenses 450

6
Closing Entries
Home Office Books Branch Books

n. Sales 157,500 Sales 99,000


Shipments to branch 75,300 Merchandise inv., 12/31 35,250
Merchandise inv., 12/31 72,750 Income summary 2,100
Merchandise inv. 1/1 60,180 Purchases 33,750
Purchases 183,750 Shipment from HO 75,300
Expenses 41,445 Expenses 27,300
Income summary 20,175

o. Branch loss 2,100 Home office 2,100


Investment in branch 2,100 Income summary 2,100

p. Income summary 2,100


Branch loss 2,100

q. Income summary 18,075


Retained earnings 18,075

3. Individual Financial Statements

Cebu Company – Home Office


Income Statement
Year Ended December 31, 2008

Sales P157,500
Cost of sales
Merchandise inventory, 1/1 P 60,180
Purchases 183,750
Goods available for sale P243,930
Shipment to branch ( 75,300)
Goods available for own sale P168,630
Merchandise inventory, 12/31 ( 72,750) 95,880
Gross profit P 61,620
Expenses 41,445
Net operating income P 20,175
Branch income (loss) ( 2,100)
Net income P 18,075

Cebu Company – Branch


Income Statement
Year Ended December 31, 2008

Sales P 99,000
Cost of sales
Purchases P 33,750
Shipments from home office 75,300
Goods available for sale P109,050
Merchandise inventory, 12/31 35,250 73,800
Gross profit P 25,200
Expenses 27,300
Net income (loss) P( 2,100)

7
Cebu Company – Home Office
Balance Sheet
December 31, 2008

Assets
Cash P 34,800
Accounts receivable 28,575
Merchandise inventory, 12/31 72,750
Prepaid expenses 3,075
Furniture and fixtures P30,000
Less: Accumulated depreciation 8,370 21,630
Branch furniture and fixtures P12,000
Less: Accumulated depreciation 975 11,025
Investment in branch 45,825
Total assets P217,680

Liabilities and Stockholders’ Equity


Liabilities
Accrued expenses P 2,025
Accounts payable 31,950
Total liabilities P 33,975
Stockholders’ Equity
Capital stock P 75,000
Retained earnings 108,705 183,705
Total liabilities and stockholders’ equity P217,680

Cebu Company – Branch


Balance Sheet
December 31, 2008

Assets
Cash P 6,375
Accounts receivable 18,000
Merchandise inventory, 12/31 35,250
Prepaid expenses 1,125
Total assets P61,650

Liabilities and Capital


Accounts payable P 450
Home office 15,375
Total liabilities and capital P61,650

8
4. Combined Financial Statements
Cebu Company
Combined Income Statement
Year Ended December 31, 2008

Sales P256,500
Cost of sales
Merchandise inventory, 1/1 P 60,180
Purchases 217,500
Goods available for sale P277,680
Merchandise inventory, 12/31 108,000 169,680
Gross profit P 86,820
Expenses 68,745
Combined net income P 18,075

Cebu Company
Balance Sheet
December 31, 2008

Assets
Cash P 41,175
Accounts receivable 47,475
Merchandise inventory 108,000
Prepaid expenses 4,200
Furniture and fixtures P42,000
Less: accumulated depreciation 9,345 32,655
Total assets P233,505

Liabilities and Stockholders’ Equity


Accrued expenses P 2,475
Accounts payable 47,325
Capital stock 75,000
Retained earnings 108,705
Total liabilities and stockholders’ equity P233,505

Problem 12-4

Branch Books Home Office Books


(a) and (b) Closing Entries

Sales 145,000 Sales 560,000


Inventory, 12/31 60,000 Inventory, 12/31 90,000
Inventory, 1/1 18,000 Shipments to branch 145,000
Shipments from HO 145,000 Inventory, 1/1 45,000
Expenses 20,000 Purchases 540,000
Income summary 23,000 Expenses 90,000
Income summary 120,000

Income summary 22,000 Investment in branch 22,000


Home office 22,000 Branch income 22,000

Branch income 22,000


Income summary 22,000

Income summary 142,000


Retained earnings 142,000

9
© CG Corporation
Combined Statement Working Paper
Year Ended December 31, 2008

Eliminations
Income
Home Statement Balance
Office Branch Debit Credit Dr (Cr) Sheet
Debits
Cash 36,000 7,000 43,000
Accounts receivable 54,000 29,000 83,000
Inventory, 1/1 45,000 18,000 63,000
Investment in branch 70,000 (2) 70,000
Equipment (net) 95,000 95,000
Purchases 540,000 540,000
Shipments from HO 145,000 (1)145,000
Expenses 90,000 20,000 110,000
Total debits 930,000 219,000

Inventory 12/31 (BS) 150,000


Total assets 371,000

Credits
Accounts payable 27,000 4,000 31,000
Home Office 70,000 (2) 70,000
Capital stock 54,000 54,000
Retained earnings, 1/1 144,000 144,000
Sales 560,000 145,000 (705,000)
Shipments to branch 145,000 (1)145,000
Total credits 930,000 219,000

Inventory, 12/31 (IS) 90,000 60,000 (150,000)


215,000 215,000
Net income 142,000 142,000

Total liabilities & equity 371,000

1. To eliminate shipments to branch and shipments from HO


2. To eliminate reciprocal accounts.

10
Problem 12-5

(1) Oro Company


Working Paper for Combined Statements
Year Ended December 31, 2008

Income
Home Eliminations Statements Balance
Office Branch Debit Credit Dr (CR) Sheet
Debits
Cash 63,000 21,900 84,900
Notes receivable 10,500 10,500
Accounts receivable (net) 120,600 55,950 176,550
Inventories 143,700 36,300 (2)135,000 45,000
Furniture & fixtures (net) 72,150 72,150
Investment in Branch 124,050 (1)124,050
Cost of goods sold 300,750 128,700 (2)135,000 564,050
Operating expenses 104,250 32,850 137,100

Totals 939,000 275,700 389,100

Credits
Accounts payable 61,500 61,500
Common stock 300,000 300,000
Retained earnings 37,500 37,500
Home Office 124,050 (1)124,050
Sales 540,000 151,650 (691,650)

Totals 939,000 275,700 289,050 289,050

Net Income 9,900 (9,900)


389,100
(1) To eliminate shipments
(2) To eliminate reciprocal accounts.

Closing Entries

2. Branch Books 3. Home Office Books

Sales 151,650
Income Summary 9,900
Cost of goods sold 128,700
Operating expenses 32,850

Home Office 9,900 Branch loss 9,900


Income summary 9,900 Investment in Branch 9,900

Income summary 9,900


Branch loss 9,900

11
Problem 12-6

a. Investment in Branch account (Home Office Books)


Unadjusted balance P138,200
Error in recording cash transfer, April 8 ( 45,000)
Cash transfer recorded in subsequent year, Dec. 31 ( 15,000)
Error in recording allocated depreciation, Dec. 31 6,000
Adjusted balance P 84,200

Home Office account (Branch Books)


Unadjusted balance P(93,000)
Error in recording salary allocation, April 5 ( 200)
Error in recording inventory transfer, July 6 12,000
Unrecorded allocated depreciation, Dec. 31 ( 3,000)
Adjusted balance P(84,200)

b. Adjusting Entries

Home Office Books Branch Books


Other income 45,000 Salary expense 200
Investment in branch – Home office 200
Rizal 45,000

Cash 15,000 Home office 12,000


Investment in branch- Shipments from HO 12,000
Rizal 15,000

Investment in branch 6,000 Depreciation expense 3,000


Accumulated dep’n 6,000 Home office 3,000

Problem 12-7

a. Investment in Branch account (Home Office Books)


Unadjusted balance, Dec. 31 P166,400
Cash remittance in transit (30,000)
Merchandise returns in transit (12,000)
Adjusted balance, Dec. 31 P124,400

Home Office account (Branch Books)


Unadjusted balance, Dec. 31 P103,200
Error in recording expense 7,200
Shipment in transit 24,000
Supplies charged to branch 8,000
Collection of branch receivable ( 18,000)
Adjusted balance, Dec. 31 P124,400

b. Adjusting Entries
Home Office Books Branch Books
Cash 30,000 Shipment from HO 24,000
Shipment to branch 12,000 Supplies 8,000
Investment in branch 42,000 Expenses 7,200
Accounts receivable 18,000
Home office 21,200

12
Problem 12-8

(1) Reconciliation Statement


(Home Office Books) (Branch Books)
Investment in Branch Home Office

Unadjusted balances, 1/31 P59,720 P 43,268


Advertising charged to branch 480
Home office AR collected by branch 600
Shipment in transit ( 180)
Error in recording receipt of merchandise ( 432)
Understatement of depreciation (12,800)
Remittance in transit, 1/31 P47,088 P 47,088

(2) Adjusting Entries

Home Office Books Branch Books


Retained earnings 432 Advertising 480
Cash 12,800 Shipments from HO 3,520
Accounts receivable 600 Shipment from HO 180
Investment in branch 12,632 Home office 3,820

Problem 12-9

(1) Branch Books

Adjusting Entries

Shipment from home office 57,600


Operating expenses (P4,200 + P3,900) 8,100
Home office 65,700

Closing Entries

Sales 778,200
Inventory, 12/31 (P64,580 + P57,600) 122,180
Inventory, 1/1 47,800
Shipment from HO (P623,200 + P57,600) 680,800
Operating expenses 54,790
Income summary 116,990

Income summary 116,990


Home office 116,900

13
(2) Home Office Books

Accounts receivable 470


Investment in branch 330
Cash (P20,000 + P19,200) 800

Investment in branch 116,990


Branch income 116,900

(3) Reconciliation Statement

Home Office Books Branch Books


(Investment in Branch) (Home Office)
Unadjusted balances, 12/31 P 206,344 P 140,974
Error in recording remittance to branch 20,000
Shipment in transit 57,600
Expenses charged to branch 8,100
Branch net income 116,990 116,990
Freight erroneously charged to branch ( 470)
Cash remittance in transit to HO ( 19,200)
Adjusted balances, 12/31 P 323,664 P 323,664

Problem 12-1111

a. P 2,000

Sales (P 27,000 + P 33,000 + P 26,000) …………………. P 86,000


Cost of Goods Sold (P 36,000 + P 18,000) ………………. (54,000)
Gross Profit ……………………………………………… P 32,000
Rent Expense …………………………………………….. P 4,000
Property Tax Expense …………………………………… 5,000
Depreciation Expense …………………………………… 4,000
Miscellaneous Expense …………………………………. 11,000
General Corporate Expense ……………………………… 6,000 (30,000)
Net Income ……………………………………………… P 2,000

b. P 180,000

Initial Transfers …………………………………………. P 188,000


June Inventory Shipment ……………………………….. 18,000
Property Tax Payment ………………………………….. 5,000
September Inventory Shipment ………………………… 26,000
Expense Allocation …………………………………….. 6,000
Cash Transfer …………………………………………... (63,000)
Balance in Home Office/Branch Accounts (correct) ….. P 180,000

14
c. Journal Entries – Tarlac Branch

1/10/08 Cash …………………………………. 30,000


Inventory ……………………………. 36,000
Equipment …………………………… 122,000
Home Office …………………… 188,000
1/20/08 Rent Expense ………………………… 4,000
Cash ……………………………. 4,000
2/1/08 Cash ………………………………….. 27,000
Sales …………………………… 27,000
Cost of Goods Sold ………………….. 18,000
Inventory ………………………. 18,000
4/1/08 Cash …………………………………. 33,000
Sales …………………………... 33,000
Cost of Goods Sold …………………. 18,000
Inventory ……………………… 18,000
5/1/08 Miscellaneous Expenses ……………. 7,000
Cash …………………………... 7,000
6/5/08 Inventory ……………………………. 18,000
Home office …………………... 18,000
7/6/08 Property Tax Expense ………………. 5,000
Home Office ………………….. 5,000
9/9/08 Inventory …………………………… 26,000
Home Office …………………. 26,000

10/1/08 Cash ………………………………… 26,000


Sales …………………………. 26,000
Cost of Goods Sold ……………….. 18,000
Inventory …………………….. 18,000
11/1/08 Miscellaneous Expenses …………... 4,000
Cash …………………………. 4,000
12/22/08 Home Office ……………………… 63,000
Cash …………………………. 63,000
12/31/08 Depreciation Expense ……………. 4,000
Accumulated depreciation ….. 4,000
12/31/08 General Corporate Expenses ……… 6,000
Home Office ………………….. 6,000

15
d. TARLAC BRANCH
Balance Sheet
December 31, 2008

Assets
Cash ……………………………………………. P 38,000
Inventory ………………………………………. 26,000
Equipment ……………………………………... P 122,000
Accumulated Depreciation ……………………. (4,000) 118,000
Total Assets …………………………… P 182,000

Equity
Home Office* ………………………………….. P 182,000

*Home office balance is P 180,000 as computed in Part b plus the P 2,000 net
income for the period.

16
CHAPTER 13

MULTIPLE CHOICE

13-1: c

13-2: a

Goods available for sale:


At billed price (P30,000 + P180,000) P210,000
At cost (P210,000 / 120%) 175,000
Balance of Allowance for Overvaluation account before adjustment P 35,000

13-3: c

Inter-company inventory profit (IIP) before closing P 66,000


Less: IIP from shipment from home office
Billed price P300,000
Cost (P300,000 / 120%) 250,000 50,000
IIP from beginning inventory at billed price P 16,000
Divided by ÷ 20%
Cost of branch’s beginning inventory P 80,000

13-4: a

Billed Price % Cost Overvaluation


Beginning inventory from HO P15,000 150% P10,000 P 5,000
Shipments 110,000 150% 73,333 36,667
Balance before adjustment P41.667
Ending inventory from HO 5,000 150% 3,333 1,667
Required adjustments P40,000

13-5: b

Shipment to branch, at billed price P375,000


Shipping cost 2,000
Total cost P377,000
Sold (50%) 188,500
Inventory P188,500

13-6: a

Shipment to branch, at cost P312,500


Shipping cost 2,000
Billed price P314,500
Sold (50%) 157,250
Inventory, at billed price P157,250

17
13-7: c

Home office account balance after closing branch profit P765,000


Less: branch profit 130,000
Investment in branch account balance before closing branch profit P635,000

13-8: d

Branch ending inventory, at billed price P 50,000


Acquired from home office, at billed price:
Cost (P6,000 / 20%) P30,000
Mark-up 6,000 36,000
Purchased from outsiders P 14,000

13-9: b

Cost of goods sold – Home office P590,000


Cost of goods sold – Branch:
Billed price P300,000
Less: overvaluation (P110,000 – P90,000) 20,000 280,000
Combined cost of goods sold P870,000

13-10: c

13-11: d

Overvaluation of branch ending inventory acquired from HO:


Billed price P 28,600
Cost (P28,600 / 130%) 22,000
Adjusted balance of allowance for overvaluation account P 6,600

13-12: b

Shipment from home office P 90,000


Expenses 17,000
Cash remittance to home office (70,000)
Home Office account balance before closing P 37.000

13-13: b

Shipment to branch, at cost P 72,000


Ending inventory, at cost (P70,000 / 30%) ( 21,600)
Cost of goods sold P 50,400
Freight (P6,000 x P50,400/P72,000) 4,200
Total P 54,600

13-14: b (20% of P30,000)

13-15: b (P151,200 / 140%)

18
13-16: c

Sales P270,000
Cost of goods sold
Shipments from home office (P151,200/140%) P108,000
Inventory, 1/1 (P28,350 / 140%) 20,250
Inventory, 12/31 (P25,200 / 140%) ( 18,000) 110,250
Gross profit P159,750
Expenses 90,000
Branch profit as far as the home office is concerned P 69,750

13-17: c

Unsold merchandise P 60,000


Less: Merchandise acquired from home office, at billed price 45,000
Merchandise acquired from outsiders P 15,000
Merchandise acquired from home, at cost (P7,500 / 20%) 37,500
Branch inventory at cost, 12/31 P 52,500

13-18: a

Branch inventory, 1/1 P 54,600


Acquired from home office – at billed price:
Overvaluation [P99,900 – (P390,000 – P300,000)] P 9,900
Cost (P9,900 / 30%) 33,000 42,900
Purchases from outsiders P 11,700

13-19: c

Acquired from home office [(P60,000 x 80%) ÷ 120%] P 40,000


Acquired from outsiders (P60,000 x 20%) 12,000
Branch inventory, 12/31 – at cost P 52,000

13-20: b

Sales (P148,000 + P144,000) P192,000


Cost of sales – at cost to home office:
Shipment from home office (P108,000 / 120%) P90,000
Purchases 52,000
Inventory, 12/31 (no. 19 above) (52,000) 90,000
Gross profit P102,000
Expenses (P76,000 + P24,000) 100,000
Branch net income (actual) P 2,000

13-21: b

Allowance for overvaluation account balance P 57,500


Overvaluation on the shipment (P200,000 x 25%) 50,000
Overvaluation on the branch beginning inventory P 7,500
Cost of branch beginning inventory (P7,500 / 25%) 30,000
Branch beginning inventory – at billed price P 37,500

19
13-22: b

Sales P400,000
Cost of goods sold – cost to home office
Beginning inventory P 30,000
Shipment from home office 200,000
Ending inventory (P40,000 / 125%) ( 32,000) 198,000
Gross profit P202,000
Expenses 100,000
Branch net income as far as the home office is concerned P102,000

13-23: b

Branch inventory, 1/1 P 20,000


Acquired from home- at billed price
Overvaluation [P24,000 – (P80,000 – P60,000)] P 4,000
At cost [(P4,000 ÷ (P20,000 / P60,000)] 12,000 16,000
Acquired from outsiders P 4,000

13-24: a

Sales P200,000
Cost of sales (at cost to home office)
Inventory, 1/1 (P12,000 + P4,000) P16,000
Shipments from home office 60,000
Purchases 30,000
Inventory, 12/31 [(P20,000÷133 1/3%) +P6,000] (21,000) 85,000
Gross profit P115,000
Expenses 60,000
Branch net income (actual) P 55,000

13-25: a

Inventory, 1/1 P 75,000


Shipments from home office 360,000
Overvaluation ( 72,500)
Cost of goods available for sale P362,500

Percentage of mark-up (P72,500 / P362,500) 20%

13-26: b

20
13-27: a

Billing percentage above cost (P20,000 / P80,000) 25%

Branch inventory, 6/1 – at cost (P12,000 / 125%) P 9,600


Home office inventory, 6/1 40,000
Purchases 160,000
Goods available for sale P209,600
Inventory, 6/30 – at cost:
Branch (P10,000 / 125%) P 8,000
Home office 60,000 68,000
Combined cost of goods sold P141.600

13-28: d

Sales P450,000
Cost of goods sold 141,600
Gross profit P308,400
Expenses 150,000
Combined net income P158,400

13-29: d

Sales P687,500
Cost of goods sold:
Inventory, 1/1: Home office P57,500
Branch (P22,250 / 125%) 17,800 P 75,300
Purchases 410,000
Goods available for sale P 485,300
Inventory, 12/31: Home office P71,250
Branch (P29,250/120%) 24,375 95,625 389,675
Gross profit P297,825
Expenses 241,750
Combined net income P 56,075

13-30: a

Sales P669,000
Cost of goods sold:
Inventory, 1/1:
Home office P160,000
Branch [P15,000 + (P49,000 / 122.5%)] 55,000 P215,000
Purchases 460,000
Goods available for sale P675,000
Inventory, 12/31:
Home office P110,000
Branch [P11,000 + (P52,000 / 133 1/3%)] 50,000 160,000 515,000
Gross profit P154,000
Expenses 145,000
Combined net income P 9,000

21
13-31: a
The entries made by the branch to record the interbranch transfer of merchandise are:
Books of Branch 1:
Home office 19,500
Freight in 3,500
Shipment from home office 16,000
Books of Branch 3:
Shipment from home office 16,000
Freight in 4,000
Cash 2,500
Home office 17,500

Therefore the home office would make the following entry:


Investment in Branch 3 17,500
Excess freight 2,000
Investment in Branch 1 19,500

13-32: a
(Home office books) (Branch books)
Investment in branch Home office
Unadjusted balances 77,000 61,000
Error in recording shipment (10,000)
Error in recording expense 5,000
Unrecorded cash remittance (31,000) -
Adjusted balances 46,000 46,000

13-33: c

13-34: a

Home office books Cebu branch books Bacolod branch books


Inv in Bacolod 25,000 Home office 25,000 Cash 25,000
Inv in Cebu 25,000 Cash 25,000 Home office 25,000

Inv in Bacolod 34,300 Home office 34,300 Cash 34,300


Inv in Cebu 34,300 SD 700 Home office 34,300
AR 35,000

Inv in Bacolod 62,500 Home office 212,500 Expenses 62,500


Expenses 150,000 Expenses 37,500 Home office 62,500
Inv in Cebu 212,500 Cash 250,000

Inv in Cebu 253,000 Freight in 3,000


S to branch 200,000 S from HO 250,000
Allowance 50,000 Home office 253,000
Cash 3,000

Inv in Bacolod 252,700 Home office 253,000


Excess freight 300 S from H 253,000
Inv in Cebu 253,000

22
(Home office books) (Bacolod branch books)
Investment in Cebu Branch Home Office
25,000 25,000
34,300 34,300
212,500 62,500
253,000 252,700
253,000 524,800 374,500
271,800

23
PROBLEMS

Problem 13-1

(a) Journal Entries

Home Office Books Branch Books

(1) Investment in branch 18,000 Equipment 18,000


Cash 18,000 Home office 18,000

(2) Investment in branch 3,000 Rent expense 3,000


Cash 3,000 Home office 3,000

(3) Investment in branch 100,000 Shipment from HO 100,000


Shipment to branch 80,000 Home office 100,000
Allowance for over-
Valuation 20,000

(4) No entry Operating expenses 11,000


Cash 11,000

Cash 105,000
Sales 105,000

(5) Cash 60,000 Home office 60,000


Investment in branch 60,000 Cash 60,000

(b) Working Paper Elimination Entries

(1) Home office 61,000


Investment in branch 61,000
To eliminate reciprocal accounts computed
as follows:
Equipment purchased P 18,000
Rent paid 3,000
Inventory shipped 100,000
Cash transfer ( 60,000)
Balance P 61,000

(2) Shipment to branch 80,000


Allowance for overvaluation of branch inventory 20,000
Shipment from home office 100,000
To eliminate inter-company shipments

(3) Inventory, 12/31 (Income statement) 5,000


Inventory, 12/31 (Balance Sheet) 5,000
To reduce inventory, 12/31 to cost.

24
(c) Closing Entries – Branch Books

Sales 105,000
Inventory, 12/31 25,000
Rent expense 3,000
Shipment from home office 100,000
Operating expenses 11,000
Income summary 16,000

Income summary 16,000


Home office 16,000

Problem 13-2

a. Branch Books

- Equipment 50,000
Shipment from home office 60,000
Cash 10,000
Home office 120,000

- Purchases 30,000
Cash or accounts payable 30,000

- Prepaid rent 10,000


Home office 10,000

- Cash 40,000
Accounts receivable 50,000
Sales 90,000

- Advertising expense 8,000


Salary expense 5,000
Cash 13,000

- Home office 10,000


Cash 10,000

- Home office 3,000


Accounts receivable 3,000

- Rent expense 5,000


Prepaid rent 5,000

25
Home Office Books

- Investment in branch 120,000


Equipment 50,000
Shipment to branch 40,000
Allowance for overvaluation of branch inventory 20,000
Cash 10,000
To record assets sent to branch

- Investment in branch 10,000


Cash 10,000
To record rent expense of the branch

- Cash 10,000
Investment in branch 10,000
To record cash remittance from branch

- Cash 3,000
Investment in branch 3,000
To record collection of branch receivable.

b. Income Statement

Sales P90,000
Cost of goods sold
Shipment from home office – at cost P40,000
Purchases 30,000
Goods available for sale 70,000
Ending inventory:
From home office (1/3) P13,333
From outsiders (1/4) 7,500 (20,833) 49,167
Gross profit P40,833
Expenses:
Advertising expense P 8,000
Salary expense 5,000
Rent expense 5,000 18,000
Net income P22,833

Problem 13-3

a. Investment in Branch account – beginning balance P 86,000


Cash transfer ( 32,000)
Inventory transfer 34,500
Rent allocated 1,000
Expenses allocated 3,000
Inventory transfer 46,000
Transportation allocated 3,000
Unadjusted balance – Investment in Branch account P141,500

26
b. Home Office account – beginning balance P 54,000
Inventory transfer 34,500
Rent allocated 1,000
Expenses allocated 3,000
Inventory transfer (error made) 64,000
Cash transfer ( 74,000)
Home Office account – unadjusted balance P 82,500

c. Reconciliation Statement
Investment in Branch Home Office
Unadjusted balances, 1/31 P141,500 P 82,500
Unrecorded cash transfer ( 74,000)
Error in recording transfer (overstated) 18,000
Expense allocation not recorded ( 3,000)
Adjusted balances, 1/31 P 67,500 P 67,500

Problem 13-4

a. Books of Branch X

Shipment from home office 5,000


Freight-in 300
Home office 5,300

Home office 5,800


Shipment from office 5,800

b. Books of Branch Y

Shipment from home office 5,000


Freight-in 600
Home office 5,600

c. Books of the Home Office

Investment in branch – X 5,300


Shipment to branch – X 5,000
Cash 300

Investment in branch – Y 5,000


Inter-branch freight expense 600
Investment in branch – X 5,600

Shipment to branch – X 5,000


Shipment to branch – Y 5,000

27
Malakas Company
Combination Worksheet
Year Ended December 31, 2008

Adjustments and Income Retained


Eliminations Statement Earnings Balance
Malakas Davao Debit Credit Dr (Cr) Dr (Cr) Sheet
Debits
Cash 25,000 18,000 43,000
Accounts receivable 108,000 25,000 133,000
Inventory, 12/31 209,000 42,000 (4) 14,000 (5) 16,000 249,000
Investment in branch 207,000 - (7)207,000
Land, bldg, and equipment 340,000 112,000 452,000
Shipment from office - 96,000 (3) 14,000 (6)110,000
Purchases 348,000 - 348,000
Depreciation expense 25,000 8,000 33,000
Advertising expense 36,000 15,000 (1) 9,000 60,000
Rent expense 12,000 5,000 (1) 6,000 23,000
Miscellaneous expense 40,000 20,000 (1) 2,000 62,000
Inventory, 1/1 175,000 35,000 (2) 10,000 200,000
Total debits 1,525,000 376,000 877,000

Credits
Accumulated depreciation 80,000 16,000 96,000
Accounts payable 37,000 15,000 52,000
Notes payable 220,000 - 220,000
Home office - 176,000 (7)207,000 (1) 17,000 -
(3) 14,000
Common stock 100,000 - 100,000
Retained earnings, 1/1 240,000 - (2) 10,000 (230,000)
Sales 529,000 127,000 (655,000)
Shipment to branch 110,000 - (6)110,000
Inventory, 12/31 209,000 42,000 (5) 16,000 (4) 14,000 (249,000)

Combined net income (179,000) (179,000)

Combined retained earnings (409,000) (409,000)

Totals 1,525,000 376,000 388,000 388,000 877,000

Adjustments and Elimination Entries

(1) Advertising expense 9,000


Rent expense 6,000
Miscellaneous expenses 2,000
Home office 17,000
Unrecorded expenses allocated to the branch

(2) Retained earnings, 1/1 10,000


Inventory, 1-1 10,000
To eliminate unrealized inventory profit of preceding year

(3) Shipment from home office 14,000


Home office 14,000
Unrecorded shipments

28
(4) Inventory, 12/31 (debits) 14,000
Inventory (credits) 14,000
Shipment not yet received by the branch

(5) Inventory, 12/31 (debits) 16,000


Inventory (credits) 16,000
To reduce ending inventory to cost

(6) Shipment to branch 110,000


Shipment from home office 110,000
To eliminate inter-company shipments

(7) Home office 207,000


Investment in branch 207,000
To eliminate reciprocal accounts

Problem 13-6

a. Eliminating Entries

(1) Home office 395,000


Investment in branch – Silver 395,000

(2) Home office 260,000


Investment in branch – Opal 260,000

(3) Unrealized intra-company profit – Silver 20,000


Unrealized intra-company profit – Opal 16,000
Inventory – from home office 36,000

(4) Inventory 90,000


Inventory – from home office 90,000

(5) Unrealized intra-company profit – Silver 40,000


Equipment 40,000

29
Ginto Company
Balance Sheet Working Paper
December 31, 2008

Home Silver Opal Eliminations


Office Branch Branch Debit Credit Combined
Cash 81,000 20,000 15,000 116,000
Accounts receivable 100,000 40,000 25,000 165,000
Inventory 260,000 50,000 44,000 (4) 90,000 444,000
Inventory – from home office 70,000 56,000 ( 3) 36,000
(4) 90,000
Land 70,000 30,000 20,000 120,000
Buildings and equipment 700,000 350,000 200,000 (5) 40,000 1,210,000
Investment in branch – Silver 395,000 (1)395,000
Investment in branch – Opal 260,000 (2)260,000
Total debits 1,866,000 560,000 360,000 2,055,000

Accumulated depreciation 280,000 120,000 80,000 480,000


Accounts payable 110,000 45,000 20,000 175,000
Bonds payable 400,000 400,000
Common stock 300,000 300,000
Retained earnings 700,000 700,000
Home office - 395,000 260,000 (1)395,000
(2)260,000
Unrealized intra-company profit
Silver 60,000 (3) 20,000
(5) 40,000
Opal 16,000 (3) 16,000
Total credits 1,866,000 560,000 360,000 821,000 821,000 2,055,000

b. Ginto Company
Combined Balance Sheet
December 31, 2008

Assets
Cash P 116,000
Accounts receivable 165,000
Inventory 444,000
Land 120,000
Buildings and equipment P1,210,000
Less: Accumulated depreciation 480,000 730,000
Total assets P1,575,000

Liabilities and Stockholders’ Equity


Liabilities
Accounts payable P 175,000
Bonds payable 400,000
Total liabilities P 575,000
Stockholders’ Equity
Common stock P 300,000
Retained earnings 700,000 1,000,000
Total liabilities and stockholders’ equity P1,575,000

30
Problem 13-7

a. Books of Branch P

Shipment from home office 8,000


Freight-in 50
Home office 8,050

Home office 8,120


Shipment from home office 8,000
Freight-in 50
Cash 70

b. Books of Branch Q

Shipment from home office 8,000


Freight-in 80
Home office 8,080

c. Books of Home Office

Investment in branch – P 8,050


Shipment to branch – P 8,000
Cash 50

Investment in branch – Q 8,080


Inter-branch freight expense 40
Investment in branch – P 8,120

Shipment to branch - P 8,000


Shipment to branch – Q 8,000

Problem 13-8

Debits:
Cash = P36,000 (add the book values and include the P9,000 transfer in transit)
Accounts receivable = P118,000
Inventory, 12/31 = P151,000 (branch balance would be P81,000 when the shipment in transit is
included. This balance must be adjusted to cost of P54,000
(P81,000 ÷ 150%) and then add to home office balance of P97,000.
Investment in branch = 0 (eliminated)
Land, buildings and equipment = P460,000
Shipment from home office = 0 (eliminated)
Purchases = P429,000
Depreciation expense = P28,000 (add the two book values and the year-end allocation)
Advertising expense = P58,000 (add the two book values and the year-end allocation)
Rent expense = P30,000 (add the two book values and the year-end allocation)
Miscellaneous expense = P100,000 (add the two book values and the year-end allocation)
Inventory, 1/1 = P145,000 (branch balance is adjusted to cost of P24,000 (P36,000 / 150%),
and then added to home office balance.
Total debits = P1,555,000 (add the above totals)

31
Credits
Accumulated depreciation = P108,000
Accounts payable = P104,000
Notes payable = P180,000
Home office = 0 (eliminated)
Common stock = P60,000 (home office balance)
Retained earnings, 1/1 = P248,000 (home office balance after reduction of P12,000 unrealized
profit in beginning inventory of branch. Cost is P24,000
(P36,000 / 150%) which indicates the P12,000 unrealized.
Sales = P704,000
Shipment to branch = 0 (eliminated)
Inventory, 12/31 = P151,000
Total credits = P1,555,000 (add the above totals)

Reconciliation Statement
Investment in Branch account balance (Home office books) P177,000
Unrecorded cash transfer ( 9,000)
Adjusted balance P168,000

Home Office account balance (Branch books) P123,000


Inventory transfer in transit 21,000
Expense allocated not yet recorded 24,000
Adjusted balance P168,000

Problem 13-9

Home Office Books


Case A Case B Case C
(1) Investment in branch 60,000 75,000 90,000
Shipment to branch 60,000 60,000 60,000
Unrealized inventory profit - 15,000 30,000

(2) Cash 61,200 61,200 61,200


Investment in branch 61,200 61,200 61,200
Closing entries:
(3) Sales 130,000 130,000 130,000
Inventory, 12/31 8,000 8,000 8,000
Shipment to branch 60,000 60,000 60,000
Purchases 150,000 150,000 150,000
Expenses 17,200 17,200 17,200
Income summary 30,800 30,800 30,800
(4) Investment in branch 13,000
Branch income summary 13,000
Branch income summary 500 14,000
Investment in branch 500 14,000

Unrealized inventory profit 13,500 27,000


Branch income summary 500 14,000
Income summary 13,000 13,000

Income summary 43,800 43,800 43,800


Retained earnings 43,800 43,800 43,800

32
Ilocos Branch Books

Case A Case B Case C

(1) Shipment from home office 60,000 75,000 90,000


Home office 60,000 75,000 90,000

(2) Accounts receivable 81,000 81,000 81,000


Sales 81,000 81,000 81,000

(3) Cash 64,000 64,000 64,000


Accounts receivable 64,000 64,000 64,000

(4) Expenses 14,000 14,000 14,000


Cash 14,000 14,000 14,000

(5) Home office 61,200 61,200 61,200


Cash 61,200 61,200 61,200

Closing entries

(6) Sales 81,000 81,000 81,000


Inventory 12/31 6,000 7,500 9,000
Shipment from HO 60,000 75,000 90,000
Expenses 14,000 14,000 14,000
Income summary 13,000 500 14,000

(7) Income summary 13,000


Home office 13,000

Home office 500 14,000


Income summary 500 14,000

33
Working Paper for Combined Financial Statements
December 31, 2008

Eliminations
Home Office Branch Debit Credit Combined
Income Statement
Sales 130,000 81,000 211,000
Merchandise inventory, 12/31 8,000 9,000 (3) 3,000 14,000
Shipment to branch 60,000 (2) 60,000 -
Total credits 198,000 90,000 225,000

Shipment from home office 90,000 (2) 90,000 -


Purchases 150,000 150,000
Expenses 17,200 14,000 31,200
Total debits 167,200 104,000 181,200
Net income(loss) carried forward 30,800 (14,000) 43,800

Retained Earnings Statement


Net income (loss) from above 30,800 (14,000) 43,800
Retained earnings, 12/31 -
Carried forward 30,800 (14,000) 43,800

Balance Sheet
Cash (overdraft) 39,000 (11,200) 27,800
Accounts receivable 45,000 17,000 62,000
Merchandise inventory, 12/31 8,000 9,000 (3) 3,000 14,000
Investment in branch 28,800 (1) 28,800 -
Total debits 120,800 14,800 103,800

Accounts payable 20,000 20,000


Unrealized inventory profit 30,000 (2) 30,000 -
Capital stock 40,000 40,000
Retained earnings, from above 30,800 (14,000) 43,800
Home office 28,800 (1) 28,800 -
Total credits 120,800 14,800 121,800 121,800 103,800

34
Problem 13-10

(1) Consolidated Working Paper

Home Adj. & Elim. Income Balance


Office Branch A Branch B (dr) Cr Statement Sheet
Debits
Cash 33,000 22,000 13,000 68,000
Inventories 70,000 21,000 15,000 A (12,000)
B 8,000 110,000
Other current assets 50,000 25,000 23,000 98,000
Investment in Branch A 45,000 D 45,000
Investment in Branch B 42,000 D 42,000
Cost of sales * 80,000 57,000 45,000 B (8,000) (165,000)
C 25,000
Expenses 90,000 25,000 20,000 (135,000)
410,000 150,000 116,000 276,000

Credits
Current liabilities 40,000 15,000 11,000 66,000
Capital stock 100,000 100,000
Retained earnings, Jan. 1 50,000 50,000
Home Office 45,000 30,000 A 12,000
D (87,000)
Allow. for overvaluation of
Branch inv. – Branch A 13,000 C (13,000)
Allow. for overvaluation of
Branch inv. – Branch B 12,000 C (12,000)
Sales 195,000 90,000 75,000 360,000
410,000 150,000 116,000
Net income 60,000 60,000
276,000

 Book value of cost of sales from home office and branches

Investment in Investment in
Home Office Branch A Branch B

Inventory, January 1, P 80,000 P 18,000 P24,000


Purchases 160,000
Shipment to branch ( 90,000)
Shipment from home office 60,000 36,000
Goods available for sale P150,000 P 78,000 P 60,000
Inventory, Dec. 31 ( 70,000) ( 21,000) (15,000)
Cost of sales P 80,000 P 57,000 P 45,000

35
(2) Reconciliation of Home Office and Investment in Branch accounts.

Books of Home Office Books of Books of


Investment Investment Branch A Branch B
In Branch A In Branch B Home Office Home Office
Unadjusted balances, Dec.31 P 45,000 P 42,000 P 45,000 P 30,000

Shipments in transit to Branch B 12,000

Branch Profit (Schedule 1) 8,000 10,000 8,000 10,000

Adjusted balances, December 31 P 53,000 P 52,000 P 53,000 P 52,000

Schedule 1:

Branch A Branch B
Sales P90,000 P75,000
Cost of sales:
Beginning inventory P18,000 P24,000
Shipment from home office 60,000 48,000
Goods available for sale 78,000 72,000
Ending inventory 21,000 27,000
Cost of sales 57,000 45,000
Gross profit 33,000 30,000
Expenses 25,000 20,000
Net profit P 8,000 P10,000

36
37

CHAPTER 14

MULTIPLE CHOICES - COMPUTATIONAL

14-1: d

Price paid (8,000 shares x P30) P240,000


Contingent consideration 5,000
Acquisition cost P245,000

14-2: b

Purchase price P250,000


Less: Fair value of net assets acquired 180,000
Goodwill P70,000

14-3: c

Purchase price (100,000 shares x P36) P3,600,000


Contingent consideration 120,000
Total costs P3,720,000

14-4: d

Price paid (600,000 shares x P50) P30,000,000


Less: goodwill recorded 6,120,000
Fair value of net assets acquired P23,880,000

Capital stock issued (600,000 shares x P10) P 6,000,000


APIC (600,000 shares xP40) – P30,000 23,970,000
Increase in CJ’s equity P29,970,000

14-5: a

Price paid P2,550,000


Less: Fair value of net assets acquired
Current assets P1,100,000
Plant assets 2,200,000
Liabilities ( 300,000) 3,000,000
Income from acquisition P( 450,000)

APIC: [(P2,550,000 – P1,200,000) - P15,000] P1,335,000

14-6: a (at fair value at date of acquisition)

37
38

14-7: d

Abel net income, January to December (P80,000 + P1,320,000) P1,400,000


Cain net income, April to December 400,000
Total net income P1,800,000

14-8: a

Price paid P 800,000


Less: Fair value of net assets acquired
Cash P 160,000
Inventory 380,000
Property, plant and equipment 1,120,000
Liabilities ( 360,000) 1,300,000
Income from acquisition P (500,000)

14-9 a

Price paid P 700,000


Less: Fair value of net assets acquired (P600,000 – P188,000) 412,000
Goodwill P 288,000
Avon’s assets 2,000,000
Bell’s assets at fair value 600,000
Total assets P2,888,000

14-10: c

Debit to expenses:
Broker’s fee P 50,000
Pre-acquisition audit fee 40,000
General administrative costs 15,000
Legal fees for business combination 32,000
Other acquisition costs 6,000
Total P 143,000

Debit to APIC
Audit fee for SEC registration of stock issue P 46,000
SEC registration fee for stock issue 5,000
Total P 51,000

38
39

14-11: d

Consideration given:
Cash P270,000
Stocks issued at fair value 330,000
Total P600,000
Less: fair value of net assets acquired:
Cash P40,000
Inventories 100,000
Other current assets 20,000
Plant assets (net) 180,000
Current liabilities (30,000)
Other liabilities (40,000) 270,000
Goodwill P330,000

Total assets after combination:


Total assets before combination P 760,000
Cash paid (P270,000 + P70,000) (340,000)
Registration and issuance costs of shares issued ( 30,000)
Polo’s assets after combination P 390,000
Assets acquired at fair values 340,000
Goodwill 330,000
Total assets after combination P1,060,000

14-12: d

Price paid P1,400,000


Less: Fair value net assets acquired 1,350,000
Goodwill P 50,000

14-13: a

Price paid P160,000


Less: Fair value of net identifiable assets acquired:
Current assets P 80,000
Non-current assets 120,000
Liabilities ( 20,000) 180,000
Income from acquisition P(20,000)

Non- current assets P120,000

14-14: c

Price paid P600,000


Less: Fair value of identifiable assets acquired:
Cash P 60,000
Merchandise inventory 142,500
Plant assets (net) 420,000
Liabilities (135,000) 487,500
Goodwill P112,500

39
40

14-15: b

Price paid P1,000,000


Less: Fair value of identifiable assets acquired 800,000
Goodwill P 200,000
MM’s net assets at book value 1,200,000
PP’s net assets at fair value 800,000
Total assets after combination P2,200,000

14-16: c, Under the acquisition method assets are recorded at their fair values (P225.000)

14-17: d

Capital stock issued at par (10,000 shares x P10) P100,000


APIC (10,000 shares x P40) 400,000
Total P500,000

14-18: d, net assets are recorded at their fair values.

14-19: a

Income from acquisition P 100,000


Fair value of net assets acquired P2,000,000 – P400,000) 1,600,000
Price paid 1,500,000

Shares to be issued (P1,500,000 ÷ P40) 37,500 shares

14-20: d

Goodwill P 200,000
Fair value of net assets acquired 1,600,000
Price paid P1,800,000

Shares to be issued (P1,800,000 ÷ P40) 45,000 shares

14-21: c

Total assets of Pablo before acquisition at book value P 700,000


Total assets acquired from Siso at fair value (100,000 +440,000) 540,000
Total assets 1,240,000
Less: cash paid (15,000 + 25,000) 40,000
Total assets after cash payment 1,200,000
Goodwill to be recognized (Sched 1) 195,000
Total assets after combination 1,395,000

Sched 1: Consideration given:


Purchase price (30,000 shares x P20) 600,000
Contingent consideration 75,000 675,000
Fair value of net assets acquired (540,000 – 60,000) 480,000
Goodwill 195,000

40
41

14-22: a

Capital stock issued at par (P500,000 + P300,000) P 800,000


APIC (50,000 + 300,000) – 15,000 335,000
Retained earnings (P100,000 – 25,000) 75,000
Stockholders equity after acquisition 1,210,000

14-23: a
B Company C Company
Consideration given P4,400,000 P638,000
Less: fair value of net assets acquired 4,150,000 370,000
Goodwill P 250,000 P268,000

Total goodwill recorded (250,000 + 268,000) 518,000

14-24: a
A Company 5,250,000
B Company 6,800,000
C Company 900,000
Cash paid for acquisition costs (P20,000 + P10,000) (30,000)
Goodwill (see 14-23) 518,000
Total assets after combination 13,438,000

14-25: a
Stockholders equity before acquisition P1,300,000
Capital stock issued at par (229,000 shares x P10) 2,290,000
Additional paid-in-capital [(229,000 x 12) – 10,000] 2,738,000
Other acquisition cost (reduction from retained earnings) (20,000)
Stockholders equity after acquisition 6,308,000

14-26: 1. a
Equipment: P180,000/5 yrs. = P36,000
Building: P550,000/20 yrs. = 27,500
Total depreciation P63,500

2. b
Price paid P900,000
Less fair value of net assets acquired:
Current assets P100,000
Land 50,000
Equipment 180,000
Building 550,000
Current liabilities (150,000) 730,000
Goodwill P170,000

14-27: b
Price paid P32 M
Final fair value of net assets 28 M
Goodwill P 4 M

41
42

PROBLEMS

Problem 14-1

1. Books of Big Corporation


(a) To record acquisition of net assets of Small:
Accounts receivable 120,000
Inventories 140,000
Property, plant and equipment 300,000
Current liabilities 50,000
Income from acquisition 10,000
Cash 500,000

(b) To record acquisition-related costs:


Acquisition expense 5,000
Cash 5,000

Computation of Income from Acquisition:


Price paid P500,000
Less: Fair value of net identifiable assets acquired:
Accounts receivable P120,000
Inventories 140,000
Property, plant and equipment 300,000
Current liabilities ( 50,000) 510,000
Income from acquisition P( 10,000)

2. Books of Small Corporation


(a) To record the sale of net assets to Big:
Cash 500,000
Current liabilities 50,000
Accounts receivable 120,000
Inventories 100,000
Property, plant and equipment 280,000
Retained earnings 50,000

(b) To record liquidation of the corporation:


Common stock 200,000
Retained earnings 300,000
Cash 500,000

42
43

Problem 14-2

(1) To record the acquisition of net assets:


Cash 50,000
Inventory 150,000
Building and equipment – net 300,000
Patent 200,000
Accounts payable 30,000
Cash 565,000
Income from acquisition 105,000

Computation of Income from Acquisition


Price paid P565,000
Less: Fair value of net identifiable assets acquired
Total assets P700,000
Accounts payable ( 30,000) 670,000
Income from acquisition P(105,000)

(2) To record acquisition-related costs:


Acquisition expenses 5,000
Cash 5,000

Problem 14-3

(1) To record acquisition of net assets:


Cash and receivables 50,000
Inventory 200,000
Building and equipment 300,000
Goodwill 40,000
Accounts payable 50,000
Common stock, P10 par value 60,000
Additional paid-in capital 480,000

Computation of Goodwill
Price paid (6,000 shares x P90) P540,000
Less: fair value of net identifiable assets acquired
Total assets P550,000
Accounts payable ( 50,000) 500,000
Goodwill P 40,000

(2) To record acquisition-related costs:


Additional paid-in capital 25,000
Acquisition expenses 15,000
Cash 40,000

43
44

Problem 14-4

(1) To record acquisition of net assets:

Cash 60,000
Accounts receivable 100,000
Inventory 115,000
Land 70,000
Building and equipment 350,000
Bond discount 20,000
Goodwill 95,000
Accounts payable 10,000
Bonds payable 200,000
Common stock, P10 par value 120,000
Additional paid-in capital 480,000

Computation of Goodwill
Purchase price (12,000 shares x P50) P600,000
Less: Fair value of net identifiable assets acquired
Total assets P695,000
Total liabilities ( 190,000) 505,000
Goodwill P 95,000

(2) To record acquisition-related costs:

Additional paid in capital 18,000


Acquisition expense 10,000
Cash 28,000

Problem 14-5

1. Common stock:: P200,000 + (8,000 shares x P10) P280,000


2. Cash and receivables: P150,000 + P40,000 190,000
3. Land: P100,000 + P85,000 185,000
4. Building and equipment – net: P300,000 + P230,000 530,000
5. Goodwill: (8,000 shares x P50) - P355,000 45,000
6. APIC: P20,000 + (8,000 shares x P40) 340,000
7. Retained earnings 330,000

44
45

Problem 14-6

Combined Statement of Financial Position


After acquisition

Based on P40/share Based on P20/share


Cash and receivables P 350,000 P 350,000
Inventory 645,000 645,000
Building and equipment 1,050,000 1,050,000
Accumulated depreciation (200,000) (200,000)
Goodwill 180,000 -
Total assets P2,025,000 P1,845,000

Accounts payable P 140,000 P 140,000


Bonds payable 485,000 485,000
Common stock P10 Par value 450,000 450,000
Additional paid-in capital 550,000 250,000
Retained earnings(including income from acquisition) 400,000 520,000
Total liabilities and stockholders’ equity P2,025,000 P1,845,000

Computation of Goodwill – Based on P40 per share:


Price paid (15,000 shares x P40) P600,000
Less: Fair value of net identifiable assets (P545,000 – P125,000) 420,000
Goodwill P180,000

Computation of Income from Acquisition – Based on P20 per share:


Price paid (15,000 shares x P20) P300,000
Less: Fair value of net identifiable assets 420,000
Income from acquisition (added to retained earnings of Red) P(120,000)

Problem 14-7

(a) Combined Statement of Financial Position


January 1, 2011

ASSETS
Cash and receivables P 110,000
Inventory 142,000
Land 115,000
Plant and equipment P540,000
Less: Accumulated depreciation 150,000 390,000
Goodwill 13,000
Total assets P 770,000

LIABILITIES AND STOCKHOLDERS’ EQUITY


Current liabilities P 100,000
Capital stock, P20 par value 214,000
Capital in excess of par 216,000
Retained earnings 240,000
Total liabilities and stockholders’ equity P 770,000

45
46

Problem 14-7, continued:


Computation of Goodwill
Price paid (700 shares x P300) P210,000
Less: Fair value of net identifiable assets acquired
(P217,000 – P20,000) 197,000
Goodwill P 13,000

(b) Stockholders’ Equity section

(1) With 1,100 shares issued

Capital stock: P200,000 + (1,100 shares x P20) P222,000


Capital in excess of par: P20,000 + (1,100 x P280) 328,000
Retained earnings 240,000
Total P790,000

(2) With 1,800 shares issued

Capital stock: P200,000 + (1,800 shares x P20) P 236,000


Capital in excess of par: P20,000 + (1,800 x P280) 524,000
Retained earnings 240,000
Total P1,000,000

(3) With 3,000 shares issued

Capital stock: P200,000 + (3,000 shares x P20) P260,000


Capital in excess of par: P20,000 + (3,000 x P280) 860,000
Retained earnings 240,000
Total P1,360,000

Problem 14-8

2010 (a) 2011 2012


Revenue P1,400,000 P1,800,000 (b) P2,100,000
Net income 500,000 545,000 © 700,000
Earnings per share P 5.00 P 4.84 (d) P 5.60 (e)

(a) Separate figures for Dollar Transport only.


(b) P2,000,000 – P200,000
(c) P620,000 - P55,000
(d) P545,000 / 112,000 shares (100,000 + 125,000) ÷ 2
(e) P700,000 / 125 shares

46
47

Problem 14-9

a. Books of Peter Industries:

(1) To record acquisition of net assets:

Cash 28,000
Accounts receivable 258,000
Inventory 395,000
Long-term investments 175,000
Land 100,000
Rolling stock 63,000
Plant and equipment 2,500,000
Patents 500,000
Special licenses 100,000
Discount on equipment trust notes 5,000
Discount on debentures 50,000
Goodwill 109,700
Allowance for bad debts 6,500
Current payables 137,200
Mortgage payables 500,000
Premium on mortgage payable 20,000
Equipment trust notes 100,000
Debenture payable 1,000,000
Common stock 180,000
APIC – common 2,340,000

Computation of Goodwill
Price paid (180,000 shares x P14) P2,520,000
Less: fair value of net identifiable assets acquired
Total assets P4,112,500
Total liabilities (1,702,200) 2,410,300
Goodwill P 109,700

(2) To record acquisition-related costs:


Additional paid in capital 42,000
Acquisition expenses 135,000
Cash 42,000

47
48

Problem 14-9, continued:


b. Books of HCC:

Common stock 7,500


APIC – Common 4,500
Treasury stock 12,000
To record retirement of treasury stock.
P7,500 = P5 x 1,500 shares
P4,500 = P12,000 – P7,500

Investment in stock - Peter 2,520,000


Allowance for bad debts 6,500
Accumulated depreciation 614,000
Current payable 137,200
Mortgage payable 500,000
Equipment trust notes 100,000
Debentures payable 1,000,000
Discount on bonds payable 40,000
Cash 28,000
Accounts receivable 258,000
Inventory 381,000
Long-term investments 150,000
Land 55,000
Rolling stock 130,000
Plant and equipment 2,425,000
Patents 125,000
Special licenses 95,800
Gain on sale of assets and liabilities 1,189,900
To record sale of assets and liabilities to Peter.

Common stock 592,500


APIC – Common 495,500
APIC – Retirement of preferred 22,000
Retained earnings 1,410,000
Investment in stock – Peter 2,520,000
To record retirement of HCC stock and distribution of
Peter Industries stock:
P592,500 = P600,000 - P7,500
P495,500 = P500,000 – P4,500
P1,410,000 = P220,000 + P1,189,900

48
49

Problem 14-10

a. Increase in capital stock (P240,00 – P200,000) P 40,000


Increase in APIC (P420,000 – P60,000) 360,000
Value of shares issued P 400,000

b. Total assets after combination P1,130,000


Total assets of Subic before combination 650,000
Total fair value of assets of Clark before combination P 480,000

Total liabilities after combination P220,000


Total liabilities of Subic before combination (140,000) ( 80,000)
Fair value of Clark’s net assets (including goodwill) P 400,000
Less: Goodwill 55,000
Fair value of Clark’s net assets before combination P 345,000

c. Par value of common stock after combination P 240,000


Par value of common stock before combination 200,000
Increase in par value P 40,000
Divided by par value per share ÷ P5
Number of shares issued 8,000 shares

d. Value of shares computed in (a) P 400,000


Number of shares issued computed in © ÷ 8,000
Market price per share P 50

Problem 14-11

a. Inventory reported by Son at date of combination was P70,000


(325,000 – P20,000 – P55,000 – P140,000 – P40,000)

b. Fair value of total assets reported by Son:

Fair value of cash P 20,000


Fair value of accounts receivable 55,000
Fair value of inventory 110,000
Buildings and equipment reported following purchase P570,000
Buildings and equipment reported by Papa (350,000) 220,000
Fair value of Son’s total assets P405,000

c. Market value of Son’s bond:

Book value reported by Son P100,000


Bond premium reported following purchase 5,000
Market value of bond P105,000

49
50

Problem 14-11, continued:

d. Shares issued by Papa Corporation:

Par value of stock following acquisition P190,000


Par value of stock before acquisition (120,000)
Increase in par value of shares outstanding P 70,000
Divide by par value per share ÷ P5
Number of shares issued 14,000

e. Market price per share of stock issued by Papa Corporation

Par value of stock following acquisition P190,000


Additional paid-in capital following acquisition 262,000 P452,000

Par value of stock before acquisition P120,000


Additional paid-in capital before acquisition 10,000 (130,000)
Market value of shares issued in acquisition P322,000
Divide by number of shares issued ÷ 14,000
Market price per share P 23.00

f. Goodwill reported following the business combination:

Market value of shares issued by Papa P322,000


Fair value of Son’s assets P405,000
Fair value of Son’s liabilities:
Accounts payable P 30,000
Bond payable 105,000
Fair value of liabilities (135,000)
Fair value of Son’s net assets (270,000)
Goodwill recorded in business combination P 52,000
Goodwill previously on the books of Papa 30,000
Goodwill reported P 82,000

g. Retained earnings reported by Son at date of combination was P90,000


(P325,000 – P30,000 – P100,000 – P50,000 – P55,000)

h. Papa’s retained earnings of P120,000 will be reported.

i. 1. Acquisition expense 8,500


Additional paid-in capital 6,300
Cash 14,800

2. Goodwill previously computed (no changes) P82,000

3. Additional paid-in capital reported following combination P262,000


Stock issue costs (6,300)
Total additional paid-in capital reported P255,700

50
51

Problem 14-12

(1) Liability from contingent consideration 80,000


Loss on contingent payment 40,000
Cash 120,000
2 x (average income of P110,000 – P50,000) = P120,000

(2) Additional paid in capital 12,000


Common stock, P1 par 12,000
2 x (average income of P110,000 – P50,000) ÷ P10

(3) Additional paid in capital 100,000


Common stock, P1 par 100,000
Deficiency (P12 – P8) x 200,000 shares P800,000
Divided by fair value per share ÷ 8
Additional shares to be issued 100,000 shares

Problem 14-13

(1) To record the acquisition of net assets of Baby Company:

Current assets 256,000


Non-current assets 660,000
Goodwill 761,000
Current liabilities 162,000
Non-current liabilities 440,000
Estimated liability for contingent consideration 75,000
Cash 400,000
Common stock, (15,000 shares x P4) 60,000
Additional paid in capital (15,000 shares x P36) 540,000

Goodwill computation:
Price paid:
Cash P 400,000
Common stock (15,000 shares x P40) 600,000
Contingent consideration (P100,000 x 75%) 75,000
Total price paid 1,075,000
Less: Fair value of net assets acquired
Current assets P 256,000
Non-current assets 660,000
Current liabilities ( 162,000)
Non-current liabilities ( 440,000) 314,000
Goodwill P 716,000

(2) Goodwill 15,000


Estimated liability for contingent consideration 15,000
(P100,000 x 90%) - P75,000

51
52

Problem 14-14

(1) Price paid P500,000


Less: Fair value of net assets acquired 400,000
Goodwill recorded P100,000

(2 – a) No, because the carrying amount of the net assets of the business is less
than the recoverable of the unit.

(2 – b) Yes.

Estimated recoverable amount of the unit P400,000


Carrying value of the unit, excluding goodwill 340,000
Implied fair value of the goodwill 60,000
Existing recorded goodwill (No. 1) 100,000
Estimated impairment loss P(40,000)

Entry:
Impairment loss 40,000
Goodwill 40,000

52
CHAPTER 15

MULTIPLE CHOICES - COMPUTATIONAL

15-1: d

Price paid P4,000,000


Less fair value of net assets acquired (P6,100 – P2,800) 3,300,000
Goodwill P 700,000

15-2: a, should be P180,000

Price paid P 450,000


Non-controlling interest (P450,000/90%) x 10% 50,000
Total 500,000
Less fair value of net assets acquired (P360,000 – P40,000) 320,000
Goodwill P 180,000

15-3: c

Plant assets – Pall Company (at book value) P 220,000


Plant assets – Mall Company (at fair value) 180,000
Consolidated P 400,000

15-4: a

Price paid P 495,000


Less fair value of net assets acquired:
Cash P 60,000
Inventory 125,000
Property and equipment 385,000
Liabilities ( 70,000) 500,000
Gain on acquisition P ( 5,000)

15-5: a

Price paid P350,000


Non-controlling interest (P350,000/80%) x 20% 87,500
Total 437,500
Less fair value of net assets excluding goodwill 330,000
Goodwill P107,500

51
15-6: a

Inventory (P360,000 + P130,000) P490,000

Plant and equipment (P500,000 + P420,000) P920,000

15-7: a

Building P180,000

Land P 90,000

15-8: a

Price paid P480,000


NCI [(P480,000/80%) x 20%] 120,000
Total 600,000
Less fair value of net assets acquired 450,000
Goodwill P150,000

15-9: d

Price paid P160,000


Non-controlling interest (P160,000/80%) x 20% 40,000
Total 200,000
Less fair value of net assets acquired (P300,000 – P160,000) 140,000
Goodwill P 60,000

Therefore:
Total assets (P800,000 + P300,000 + P60,000) P1,160,000
Total liabilities (P250,000 + P155,000 + P160,000 + P5,000) 570,000

15-10: b (P900,000 x 1%)

15-11: d

Number of shares acquired (P120,000/P120) 1,000


Divided by outstanding shares of Soda (P125,000/P100) 1,250
Controlling interest 80%

Non-controlling interest [(P120,000/80%) x 20%} P30,000

52
15-12: a, should be P700,000

Goodwill P250,000
FV of net assets acquired excluding goodwill (P700,000 – P150,000) 550,000
NCI (100,000)
Price paid by the Pepsi Company P700,000

15-13: b

Price paid (P247,095 + P69,955) P317,050


NCI [(P317,050/85%) x 15%*) 55,950
Total 373,000
Less net assets at fair value excluding goodwill:
Net assets at book value P290,700
Inventories 6,630
Plant and equipment 48,450
Patent 7,650 353,430
Goodwill P 19,570

* P43,605/P290,700 = 15%

15-14: d (P500,000 + P300,000)

15-15: c

Price paid P260,000


NCI [(P260,000/80%) x 20%] 65,000
Total 325,000
Less fair value of net acquired (P450,000 – P150,000) 300,000
Goodwill P 25,000

15-16: a (The retained earnings of the parent only).

15-17: b, should be P615,000

Controlling interest (Stockholders’ equity of the parent) P550,000


Non-controlling interest (per no. 15-15) 65,000
Stockholders equity P615,000

15-18: a (refer to 15-15)

15-19: d (P380,000 + P150,000)

53
15-20: c
Cash and cash equivalent (P70,000 + P90,000) P 160,000
Inventory (P100,000 + P60,000) 160,000
Property and equipment (P500,000 + P300,000) 800,000
Goodwill 25,000
Total assets P1,145,000

15-21: a:
Fair value per share:
New acquisition (P630,000/7,000 shares) P90

Fair value of previously owned shares (1,000* shares x P90) P 90,000 (10%)
Acquisition of new shares 630,000 (70%)
Total price paid for 80% interest P 720,000
Non-controlling interest (P720,000/80%) x 20% P 180,000
* P200,000 / P20 x 10% = 1,000 shares

15-22: c
Fair value of previously owned interest (10%) P 90,000
Price paid for new additional interest (70%) 630,000
Non-controlling interest 180,000
Total 900,000
Less fair value of net assets acquired (P910,000 – P130,000) 780,000
Goodwill P120,000

15-23: a The amount reported is equal to Primo’s retained earnings of P567,000

15-24: d 100% – (P163,000/460,000) = 65%

15-25: a (340,000- 200,000)

15-26: b
Cash P 40,000
Accounts receivable 20,000
Inventories (see 15-25) 140,000
Equipment (800,000 - 500,000) 300,000
Accounts payable (40,000)
Fair value of net assets P460,000

15-27: c

Goodwill P 10,000
Fair value of net assets acquired (15-26) 460,000
Total 470,000
NCI (163,000)
Price paid by Primo P 307,000

54
15-28: b

Parent NCI
Total 65% 35%
Total implied value P470,000 P307,000 P163,000
Less fair value of net assets 460,000 299,000 161,000
Goodwill P 10,000 P 8,000 P 2,000

15-29: b

Non-controlling interest should be valued at the higher amount between the following:

At estimated fair value (P512,000/80%) x 20% P128,000


At proportionate share of acquiree’s net identifiable assets (P670,000 x 20%) 134,000

Therefore, NCI is measured at P134,000.

15-30: c

Price paid (8,000 shares x P64) P512,000


NCI 134,000
Total 646,000
Less fair value of net assets acquired excluding goodwill:
Cash P 20,000
Inventory 400,000
Equipment 500,000
Current liabilities ( 250,000) 670,000
Gain on acquisition P(24,000)

Proof:

Total Parent (80%) NCI (20%)

Fair value of the company P646,000 P512,000 P134,000


Fair of net assets excluding goodwill 670,000 536,000 134,000
Gain on acquisition P(24,000) P(24,000) P -

NCI does not share a gain on the acquisition. IFRS 3 (2008) provides that the gain is
attributed to the acquirer only.

55
PROBLEMS
Problem 15-1

a. Investment in Solo Company stock 1,080,000


Cash 1,080,000
To record acquisition of 90%
of the outstanding shares of Solo.

Retained earnings – Polo Company 50,000


Cash 50,000
To record acquisition-related costs direct to
Retained earnings of Polo Company.

b. Working paper elimination entries:

(1) Common stock – Solo 400,000


Retained earnings – Solo 500,000
Investment in Solo company stock 810,000
Non-controlling interest 90,000
To eliminate Solo’s equity accounts at date of acquisition.

(2) Inventories 30,000


Plant assets 60,000
Goodwill 210,000
Investment in Solo company stock 270,000
Non-controlling interest 30,000
To allocate excess

Determination and Allocation of Excess Schedule:

Total Parent (80%) NCI (10%)


Company fair value P1,200,000 P1,080,000 P120,000*
Less BV of interest acquired:
Common stock 400,000
Retained earnings 500,000
Total equity 900,000 P 900,000 P900,000
Interest aquired 90% 10%
Book value P 810,000 P 90,000
Excess P 300,000 P 270,000 P 30,000
Adjustments:
Inventory (30,000)
Plant assets (60,000
Goodwill P 210,000

* (P1,080,000/90%) x 10% = P120,000

56
Problem 15-2

a. Investment in Straw Company 600,000


Cash 600,000
To record acquisition of 100% of Straw stock.

b. Price paid P600,000


Less: Book value of interest acquired (100%) 420,000
Difference 180,000
Allocation (100%:
Inventories P( 40,000)
Land ( 80,000)
Building 150,000
Equipment ( 20,000)
Patents ( 20,000) ( 10,000)
Goodwill P170,000

c. Working paper elimination entries:

(1) Common stock – Straw 100,000


Retained earnings – Straw 320,000
Investment in Straw Company 420,000
To eliminate equity accounts of Straw at
date of acquisition.

(2) Inventories 40,000


Land 80,000
Equipment 20,000
Patents 20,000
Goodwill 170,000
Buildings 150,000
Investment in Straw Company 180,000
To allocate excess.

57
Problem 15-3

a. Investment in Soto Company 950,000


Cash 950,000
To record acquisition of 80% stock of Sotto.

Retained earnings – Pedro Company 80,000


Cash 80,000
To record acquisition costs.

b. Price paid by the Parent Company P950,000


Non-controlling interest (NCI) 230,000
Total 1,180,000
Less: Book value of net assets 900,000
Excess 280,000
Allocation:
Current assets P 50,000
Property and equipment (100,000)
Long-term debt ( 40,000) ( 90,000)
Goodwill P190,000

c. Working paper elimination entries:

(1) Common stock – Sotto 100,000


APIC – Sotto 200,000
Retained earnings – Sotto 600,000
Investment in Sotto stock 720,000
Non-controlling interest 180,000
To eliminate equity accounts of Sotto at date of
acquisition.

(2) Property, plant and equipment 100,000


Goodwill 190,000
Long-term debt 40,000
Current assets 50,000
Investment in Sotto stock 230,000
Non-controlling interest 50,000
To allocate excess

58
Problem 15-4

Paco Company and Subsidiary


Consolidated Statement of Financial Position
January 2, 2011

Current assets P475,000


Property, plant and equipment 285,000
Other assets 70,000
Total assets P830,000

Current liabilities P280,000


Mortgage payable 85,000
Common stock 200,000
Additional paid-in capital 65,000
Retained earnings (including gain on acquisition of P20,000) 200,000
Total liabilities and stockholders’ equity P830,000

Computation of income from acquisition:


Consideration given (20,000 shares x P6) P120,000
Less fair value of net assets:
Current assets P100,000
Property and equipment 85,000
Other assets 40,000
Current liabilities (60,000)
Mortgage payable (25,000) 140,000
Gain on acquisition P(20,000)

Problem 15-5

The entry to record the acquisition of stock is as follows:

(a) Investment in Solo stock 250,000


Common stock, at par 100,000
Additional paid-in capital 150,000
To record acquisition of stock.

(b) Retained earnings – Polo 10,000


Additional paid-in capital 20,000
Cash 30,000
To record acquisition-related costs.

59
Problem 15-5, continued
Palo Company and Subsidiary
Consolidated Statement of Financial Position
December 31, 2011
Cash P 70,000
Receivables 120,000
Inventory 170,000
Property and equipment – net 340,000
Goodwill 20,000
Total assets P720,000

Current liabilities P 30,000


Long-term liabilities 120,000
Common stock 210,000
Additional paid-in capital (P20,000 + P150,000 – P20,000) 150,000
Retained earnings, 12/31 (P220,000 – P10,000) 210,000
Total liabilities and stockholders’ equity P720,000

Computation of goodwill:
Consideration given P250,000
Less fair value of net assets (P290,000 – 60,000) 230,000
Goodwill P 20,000

Problem 15-6
a. Investment in Seed Company 350,000
Cash 350,000
To record acquisition of 100% of Seed company stock.

Determination and Allocation of Excess schedule:


Price paid P350,000
Less: Book value of interest acquired 320,000
Excess 30,000
Allocation:
Inventory P(20,000)
Plant assets (80,000)
Long-term liabilities 40,000 (60,000)
Income from acquisition P(30,000)

b. Working paper elimination entries


(1) Common stock – Seed 100,000
Additional paid-in capital – Seed 40,000
Retained earnings – Seed 180,000
Investment in Seed stock 320,000
To eliminate equity accounts of Seed Company

(2) Inventory 20,000


Plant assets 80,000
Long-term debt 40,000
Investment in Seed stock 30,000
Retained earnings – Pill (income from acquisition) 30,000
To allocate excess

60
Problem 15-6, continued:
Pill Corporation and Subsidiary
Consolidated Working Paper
May 31, 2011 – Date of Acquisition

Pill Seed Eliminations & adjustment Conso-


Corporation Company Debit Credit lidated
Assets
Cash 200,000 10,000 210,000
Accounts receivable 700,000 60,000 760,000
Inventories 1,400,000 120,000 (2) 20,000 1,540,000
Investment in Seed company 350,000 (1)320,000 -
(2) 30,000
Plant assets 2,850,000 610,000 (2) 80,000 3,540,000
Total 5,500,000 800,000 6,050,000

Liabilities & Stockholders’


Equity
Current liabilities 500,000 80,000 580,000
Long-term debt 1,000,000 400,000 (2) 40,000 1,440,000
Common stock:
Pill 1,500,000 1,500,000
Seed 100,000 (1)100,000
Additional paid-in capital
Pill 1,200,000 1,200,000
Seed 40,000 (1) 40,000
Retained earnings
Pill 1,300,000 (2) 30,000 1,330,000
Seed 180,000 (1)180,000
Total 5,500,000 800,000 420,000 420,000 6,050,000

Problem 15-7

a. Accounts Receivable 70,000


Cash 70,000

b. Investment in Sea Company stock 600,000


Common stock ((30,000 shares x P20) 600,000

Retained earnings – Pop Corporation 40,000


Common stock 30,000
Current liabilities 70,000

61
Problem 15-7, continued:
Pop Corporation and Subsidiary
Working Paper for Consolidated Balance Sheet
April 30, 2011 – Date of acquisition

Pop Sea Adjustments & Eliminatio Consoli-


Corporation Company Debit Credit dated
Assets
Cash 50,000 80,000 130,000
Accounts receivable – net 230,000 270,000 (3) 70,000 430,000
Inventories 400,000 350,000 (2) 90,000 840,000
Investment in Sea Company 600,000 (1)328,000 -
(2)272,000
Plant assets 1,300,000 560,000 (2)220,000 2,080,000
Goodwill (2) 50,000 50,000
Total 2,580,000 1,260,000 3,530,000

Liabilities & Stockholders’


Equity
Current liabilities 380,000 250,000 (3) 70,000 560,000
Long-term debt 800,000 600,000 (2) 20,000 1,420,000
Common stock
Pop 1,070,000 1,070,000
Sea 100,000 (1)100,000
Additional paid-in capital 360,000 (1)360,000
Retained earnings
Pop 330,000 330,000
Sea (50,000) (1) 50,000

NCI (1) 82,000 150,000


(2) 68,000
Total 2,580,000 1,260,000 890,000 890,000 3,530,000

(1) To eliminate equity accounts of Sea Company on the date of acquisition .


(2) To allocate difference, computed as follows:
Price paid P600,000
NCI (P600,000/80%) x 20% 150,000
Total 750,000
Less: Book value of net assets of Sea 410,000
Excess 340,000
Allocation:
Inventories P( 90,000)
Plant assets (220,000)
Long-term debt 20,000 (290,000)
Goodwill P 50,000
(3) To eliminate intercompany receivables and payables.

62
Problem 15-8

1. Price paid P500,000


Less book value of interest acquired
Common stock P100,000
APIC 200,000
Retained earnings 230,000 530,000
Excess ( 30,000)
Allocation:
Inventory P( 20,000)
Land ( 10,000)
Building 50,000
Equipment 60,000
Bonds payable ( 50,000) 30,000

2. P Company and Subsidiary


Consolidated Working Paper
January 2, 2011 – Date of acquisition

P S Adjustments & Eliminations Consoli-


Company Company Debit Credit dated
Debits
Cash 300,000 50,000 350,000
Accounts receivable 200,000 100,000 300,000
Inventory 200,000 80,000 (2) 20,000 300,000
Land 100,000 50,000 (2) 10,000 160,000
Building 600,000 400,000 (2) 50,000 950,000
Equipment 800,000 200,000 (2) 60,000 940,000
Investment in S Company 500,000 (2) 30,000 (1)530,000 -
Total 2,700,000 880,000 3,000,000

Credits
Accounts payable 150,000 60,000 210,000
Bonds payable 290,000 (2) 50,000 240,000
Common stock – P Company 1,500,000 1,500,000
Common stock – S Company 100,000 (1)100,000
APIC – S Company 200,000 (1)200,000
Retained earnings – P Co. 1,050,000
Retained earnings – S Co. 230,000 (1)230,000 1,050,000
Total 2,700,000 880,000 640,000 640,000 3,000,000

(1) To eliminate equity accounts of S Company.


(2) To allocate excess

63
Problem 15-9

1. Price paid P500,000


NCI (20% of FV of S Co’s net assets excluding GW (P500,000 x 20%) 100,000*
Total 600,000
Less book of net assets 530,000
Excess 70,000
Allocation
Inventory P (20,000)
Land (10,000)
Building 50,000
Equipment 60,000
Bonds payable (50,000) 30,000
Goodwill P100,000

* NCI is measured at its proportionate interest in S Company’s net assets because the assessed
fair value of P80,000 is smaller.

2. P Company and Subsidiary


Consolidated Working Paper
January 2, 2011 – Date of acquisition

P S Adjustments & Eliminations Consoli-


Company Company Debit Credit dated
Debits
Cash 300,000 50,000 350,000
Accounts receivable 200,000 100,000 300,000
Inventory 200,000 80,000 (2) 20,000 300,000
Land 100,000 50,000 (2) 10,000 160,000
Building 600,000 400,000 (2) 50,000 950,000
Equipment 800,000 200,000 (2) 60,000 940,000
Investment in S Company 500,000 (1)424,000 -
(2) 76,000
Goodwill (2)100,000 100,000
Total 2,700,000 880,000 3,100,000

Credits
Accounts payable 150,000 60,000 210,000
Bonds payable 290,000 (2) 50,000 240,000
Common stock – P Co. 1,500,000 1,500,000
Common stock – S Co. 100,000 (1)100,000
APIC – S Co. 200,000 (1)200,000
Retained earnings – P Co. 1,050,000 1,050,000
Retained earnings – S Co. 230,000 (1)230,000

NCI (2) 6,000 (1)106,000 100,000


Total 2,700,000 880,000 716,000 716,000 3,100,000
(1) To eliminate equity accounts of S Company
(2) To allocate excess

64
Problem 15-10

1. Price paid P542,000


Less book value of interest acquired (100%): 670,000
Excess (128,000)
Allocation
Inventory P (10,000)
Land (40,000)
Equipment 20,000
Long-term investment in MS (15,000) ( 45,000)
Gain on acquisition P(173,000)

2. P Company and Subsidiary


Consolidated Working Paper
January 2, 2011 – Date of acquisition

P S Adjustments & Eliminations Consoli-


Company Company Debit Credit dated
Assets
Cash 100,000 100,000 200,000
Accounts receivable 200,000 150,000 350,000
Inventory 150,000 130,000 (2) 10,000 290,000
Land 50,000 80,000 (2) 40,000 170,000
Equipment 300,000 200,000 (2) 20,000 480,000
Investment in S Company 542,000 (2)128,000 (1)670,000 -
Long-term investment in MS 100,000 125,000 (2) 15,000 240,000
Total 1,442,000 785,000 1,730,000

Liabilities & Stockholders’


Equity
Accounts payable 175,000 115,000 290,000
Common Stock – P Co. 400,000 400,000
Common Stock – S Co. 200,000 (1)200,000
APIC – P Co. 200,000 200,000
Retained earnings – P Co. 667,000 (2)173,000 840,000
Retained earnings – S Co. 470,000 (1)470,000
Total 1,442,000 785,000 863,000 863,000 1,730,000

(1) To eliminate equity accounts of S Company.


(2) To allocate excess

65
Problem 15-11

1. Investment in Sun Company 1,900,000


Cash 1,900,000

2. Price paid P1,900,000


Less book value of interest acquired:
Common stock P 600,000
Retained earnings 840,000 1,440,000
Excess 460,000
Allocation:
Land (100,000)
Building (200,000)
Bond payable (bond discount) ( 40,000)
Deferred taxes ( 20,000) (360,000)
Goodwill P 100,000

3. Land 100,000
Building 200,000
Bond discount 40,000
Goodwill 100,000
Deferred taxes 20,000
Retained earnings 840,000
Additional paid in capital 1,300,000

4. Common stock 600,000


Additional paid in capital 1,300,000
Investment in Sun Company 1,900,000

Problem 15-12

Supporting computations:

Fair value of existing X Company equity (200 shares P50) P10,000


P Company interest in X Company [300/(300 + 200)] 60%
Acquisition price P 6,000

Entry to record the issuance of 300 shares – Books of X Company (legal parent)

Investment in P Company 6,000


Common stock (300 shares x P2) 600
APIC 5,400

66
Problem 15-12, continued:
Fair value analysis: Implied FV Parent (60%) NCI (40%)

Company fair value P10,000 P6,000 P4,000


Fair value of net assets excluding goodwill 6,000 3,600 2,400
Goodwill P 4,000 P2,400 P1,600

1. Distribution and allocation of excess schedule:

Implied FV Parent (60%) NCI (40%)

Fair value of subsidiary P10,000 P6,000 P4,000


Less book value of interest acquired:
Common stock P2 par 4,000
APIC 1,600
Retained earnings 2,000
Total 4,000 P4,000 P4,000
Interest acquired 60% 40%
Book value P2,500 P1,600
Excess 6,000 P3,600 P2,400
Allocated to Non-current assets ( 2,000)
Goodwill P 4,000

2. X Company and Subsidiary P Company


Consolidated Statement of Financial Position
December 31, 2011

Assets Liabilities and Equity

Current assets P 4,000 Non-current liabilities P 6,000


Non-current assets 16,000 Common stock (300 shares x P2) 600
Goodwill 4,000 APIC 1,400*
Retained earnings 6,000**
NCI 10,000***
Total assets P24,000 Total liabilities and equity P24,000

* Total paid in capital of P Company (P200 + P1800) P2,000


New shares issued (300 shares x P2) 600
APIC P1,400

** Retained earnings of the legal subsidiary – P Company

*** The remaining shares of the original C Company equity.

67
CHAPTER 16

MULTIPLE CHOICES - COMPUTATIONAL

16-1: c, [(P260,000/80%) x 20%]

16-2: d, consolidated net income will decrease by P6,000 due to amortization of the allocated
excess (P60,000 / 10 years).

16-3: a, because there is no NCI in a wholly owned subsidiary.

16-4: c

Investment cost (price paid) P500,000


Less: Book value of interest acquired 480,000
Excess P 60,000

Cost Method Equity Method


Investment cost P500,000 P500,000
Parent’s share of subsidiary’s net income - 120,000
Dividends received from subsidiary - ( 48,000)
Amortization of allocated excess (P60,000/20) - ( 3,000)
Investment account balance, Dec. 31, 2011 P500,000 P569,000

16-5: a, should be P114,000

NCI, January 2, 2011 [(P270,000/75%) x 25%] P 90,000


NCI in S Company dividends [(P60,000/75%) x 25%] (16,000)
NCI in S Company net income (P160,000 x 25%) 40,000
NCI balance, December 31, 2011 P114,000

16-6: a

Puno’s net income P 145,000


Dividend income (P40,000 x 90%) ( 36,000)
Puno’s net income from own operations 109,000
Salas’ net income from own operations 120,000
Consolidated net income P 229,000

16-7: b

Peter’s net income from own operations P1,000,000


Seller’s net income from own operations 200,000
Consolidated net income 1,200,000
Attributable to NCI (P200,000 x 20%) 40,000
Attributable to parent P1,160,000

68
16-8: a

2009 2010 2011


Investment in Son, Jan. 1 P310,000 P396,200 P512,400
Pop’s share of Son’s net income (100%) 150,000 180,000 200,000
Dividends received (100%) ( 60,000) (60,000) ( 60,000)
Amortization of allocated excess to
Equipment (P38,000 / 10) ( 3,800) ( 3,800) ( 3,800)
Investment in Son, Dec. 31 P396,200 P512,400 P648,600

16-9: a

Sy’s net income P300,000


Amortization of allocated excess ( 60,000)
Adjusted net income of Sy P240,000

NCI in net income of subsidiary (P240,000 x 10%) P 24,000

16-10: a. Under the equity method consolidated retained earnings is equal to the retained
earnings of the parent company.

16-11: c

Retained earnings, Jan. 2, 2011 – Puzon P500,000


Consolidated net income attributable to parent:
Net income – Puzon P200,000
Net income – Suarez 40,000
Dividend income (P20,000 x 80%) (16,000)
NCI in Suarez net income (P40,000 x 20%) ( 8,000) 216,000
Dividends paid – Puzon ( 50,000)
Consolidated retained earnings, Dec. 31, 2011 P666,000

16-12: c

Price price P1,700,000


Less book value of interest acquired: 1,260,000
Excess P 440,000
Allocation due to undervaluation of net assets ( 40,000)
Goodwill P 400,000

16-13: d, should be P182,750

NCI, January 2, 2011 [(P975,000/80%) x 20%] P243,750


NCI in subsidiary dividends (P125,000 x20%) (25,000)
NCI in adjusted net of subisidiary (P190,000 – P10,000) x 20% 36,000
NCI, December 32, 2011 P182,750

69
16-14: b

Presto’s net income from own operations P140,000


Stork’s net income – March to December (P80,000 – P23,000) 57,000
NCI share in Stork’s net income (P57,000 x 10%) ( 5,700)
Consolidated net income attributable to parent P191,300

16-15: b

Investment in Siso Company (at date of acquisition) P600,000

Dividend income (P30,000 x 5%) P 1,500

16-16 d

Consolidated net income:


Pepe’s net income from own operations P210,000
Sison’s adjusted net income:
Net income -2011 P67,000
Amortization of allocated excess
to equipment (P20,000 / 5) 4,000 63,000
Consolidated net income P273,000

Consolidated retained earnings:


Pepe’s retained earnings, Jan.2, 2010 P701,000
Consolidated net income attributable to parent– 2010
Pepe’s NI from own operations P185,000
Sison’s adjusted NI;
Net income – 2010 P40,000
Amortization -2010 4,000 36,000
NCI in Sison’s net income (P36,000 x 30%) (10,800) 210,200
Dividends paid ,2010 - Pepe ( 50,000)
Pepe’s retained earnings, Jan. 2, 2011 P861,200
Consolidated net income attributable to parent– 2009:
Consolidated net income (see above) P273,000
NCI in Sison’s net income (P63,000 x 30%) ( 18,900) 254,100
Dividends paid, 2011 – Pepe ( 60,000)
Consolidated retained earnings, Dec. 31, 2011 P1,055,300

16-17: a, should be P821,500 and P328,500 respectively.

Price paid P 700,000


NCI, June 30, 2011 [(P700,000/70%) x 30%} 300,000
Total 1,000,000
Less book value of Susy’s net assets (P650,000 + P250,000) 900,000
Excess, allocated to building P 100,000

Amortization (P100,000 / 10) x 2/12 5,000

70
16-17, continued:
Consolidated retained earnings
Retained earnings, Jan. 1, 2011 – Pepe P550,000
Consolidated net income attributable to parent:
Net income – Precy P275,000
Adjusted net income of Susy:
Net income of Susy P100,000
Amortization (P100,000 / 10) ÷ 2 ( 5,000) 95,000
NCI in Susy’s net income (P95,000 x 30%) (28,500) 341,500
Dividends paid – Precy ( 70,000)
Consolidated retained earnings, Dec. 31, 2011 P821,500

Non-controlling interest
NCI, June 30, 2011 P300,000
NCI in Susy’s dividends, July 1 to December 31 -0-
NCI in Susy’s net income (P100,000 – P5,000) x 30% 28,500
NCI, December 31, 2011 P328,500

16-18: a

Goodwill
Price paid P1,200,000
Less: Book value of interest acquired (P1,320,000 – P320,000) 1,000,000
Goodwill (not impaired) P 200,000

Consolidated retained earnings under the equity method is equal to the retained
earnings of the parent company, P1,240,000.

16-19: b

Net income – Pablo P130,000


Dividend income (P40,000 x 70%) (28,000)
Sito’s net income 70,000
NCI in Sito’s net income (P70,000 x 30%) (21,000)
Consolidated net income attributable to parent P151,000

16-20: c

Consolidated net income – 2011


Net income – Ponce P 90,000
Dividend income (P15,000 x 60%) (9,000)
Solis’ net income 40,000
NCIin Solis’ net income (P40,000 x 40%) (16,000)
Consolidated net income attributable to parent – 2011 P105,000

71
16-20, continued:
Consolidated retained earnings – 2011
Retained earnings, Jan. 2, 2010- Ponce P 400,000
Consolidated net income attributable to parent– 2010
Net income – Ponce P70,000
Dividend income (P30,000 x 60%) (18,000)
Solis’ net income 35,000
NCI in Solis’s net income (P35,000 x 40%) ( 14,000) 75,000
Dividends paid, 2010– Ponce (25,000)
Consolidated retained earnings, Dec. 31, 2010 P450,000
Consolidated net income attributable to parent– 2011 105,000
Dividends paid. 2011 – Ponce (30,000)
Consolidated retained earnings, Dec. 31, 2011 P525,000

16-21 b

Price paid, January 2, 2011 P216,000


NCI, January 2, 2011 {(P216,000/80%) x 20%] 54.000
Total 270,000
Less book value of Seed’s net assets (P80,000 + P140,000) 220,000
Excess 50,000
Allocated to:
Depreciable assets (40,000)
Goodwill 10,000

Consolidated net income, December 31, 2011:


Polo net income from own corporation P 95,000
Seed net income from own operation:
Net income 35,000
Amortization (40,000 ÷ 10%) (4,000)
GW impairment lost (8,000) 23,000
Total P118.000

16-22: c

Retained earnings 1/1/011 – Polo P520,000


Consolidated net income attributed to parent:
Consolidated net income 118,000
NCI in Seed’s adjusted NI (23,000x 20%) (4,600) 113,400
Total 633,400
Dividends paid- Polo (46,000)
Consolidated retained earnings 12/31/11 P587,400

16-23: b, P4,600 (see 16-22)

72
16-24: c

NCI, January 2, 2011 P 54,000


NCI ins Seed’s dividends (P15,000 x 20%) (3,000)
NCI in Seed’s net income 4,600
NCI, December 31, 2011 P 55,600

16-25: c, should be P113,400 (see no. 16-22)

16-26: a

Price paid, January 1, 2010 P231,000


NCI, January 1, 2010 [(P231,000/70%) x 30%] 99,000
Total 330,000
Less book value of Sisa’s net assets 280,000
Excess 50,000
Allocated to depreciable assets (10 years remaining life) (50,000)

Retained earnings, 1/1/11-Sisa company P230,000


Retained earnings, 1/1/10-Sisa company (squeeze) 155,000
Increase 75,000
Amortization- prior years (50,000 ÷ 10 years) (5,000)
Adjusted increase in earnings of Sisa (21,000/30% ) P70,000

16-27: a
Retained earnings 1/1/11- Pepe P520,000
Retained earnings 1/1/11- Sisa 230,000
Adjustment and elimination:
Date of acquisition (155,000)
Undistributed earnings to NCI (21,000)
Amortization- prior year (5,000) 49,000
Consolidated retained earnings 1/1/11 P569,000

16-28: a
Pepe company net income, 2011 P120,000
Sisa company net income, 2011 25,000
Dividend income (10,000 x 70%), 2011 (7,000)
Amortization- 2011 (5,000)
Consolidated net income P133,000

16-29: a
Consolidated retained earnings 1/1/11(see 16 – 27) P569,000
Consolidated net income attributable to parent:
Consolidated net income (see 16-28) 133,000
NCI in Sisa NI (25,000 – 5,000) 30% (6,000) 127,000
Dividend paid- Pepe company ( 50,000)
Consolidated retained earnings 12/31/11 P646,000

73
PROBLEMS
Problem 16-1

1. Determination and Allocation of Excess Schedule:

Implied Parent Price NCI Value


Fair Value (80%) (20%)

Fair value of subsidiary P 312,500 P 250,000 P 62,500


Less book value of interest acquired
Capital stock P 100,000
Retained earnings 150,000
Total equity P 250,000 P 250,000 P 250,000
Interest acquired 80% 20%
Book value P200,000 P 50,000
Excess P 62,500 P 50,000 P 12,500
Allocation to:
Fixed assets 62,500

2. Working Paper Elimination Entries:

a. Eliminate dividends declared by the subsidiary against dividend income and NCI:

Dividend income 4,000


NCI 1,000
Dividends declared – Sulu 5,000

b. Eliminate equity accounts of the subsidiary against the investment account and
the NCI account.

Common stock – Sulu 100,000


Retained earnings – Sulu 150,000
Investment in Sulu Company 200,000
NCI 50,000

c Allocate excess to fixed assets:

Fixed assets 62,500


Investment in Sulu Company 50,000
NCI 12,500

d. Amortized fixed assets (P62,500 / 10)


Expenses 6,250
Fixed assets 6,250

e. Recognize NCI in subsidiary net income:


NCI in subsidiary net income 3,750
NCI 3,750

74
Probem 16-1 concluded

3. Pedro Company
Consolidated Income Statement
Year Ended December 31, 2011

Sales P250,000
Expenses 191,250
Consolidated net income P 58,750
Attributable to NCI 3,750
Attributable to controlling interest P 55,000

4. Pedro Company
Statement of Retained Earnings
Year Ended December 31, 2011

Retained earnings, January 1 – Pedro Company P200,000


Consolidated net income attributable to controlling interest 55,000
Retained earnings, December 31, 2011 P255,000

5. Pedro Company
Consolidated Statement of Financial Position
December 31, 2011

Assets
Current assets P190,000
Non-current assets
Fixed assets (P662,500 – P132,250) 530,250
Total assets P720,250

Liabilities and Stockholders’ Equity


Current liabilities P100,000
Stockholders’ Equity:
Controlling interest:
Common stock P300,000
Retained earnings 255,000
Total P555,000
Non-controlling interest (P62,500 – P1,000 + P3,750) 65,250 620,250
Total liabilities and equity P720,250

75
Problem 16-2

1. Eliminations and adjustments:

a to c are the same as in Problem 16-1:

d. Depreciate the fixed asset for the current year and one prior year:

Retained earnings, Jan. 1 – Sulu (prior year) 6,250


Expenses (current year) 6,250
Fixed assets 12,500

e. Recognize NCI in subsidiary net income:

NCI in subsidiary net income 1,750


NCI 1,750

e. Assign to the NCI their share of the increase in the subsidiary’s


Adjusted undistributed earnings of prior year:

Retained earnings, January 1- Sulu 2,750


NCI 2,750
Retained earnings, January 1, 2009 P170,000
Retained earnings, January 2, 2008 150,000
Increase in undistributed earnings P 20,000
Amortization in prior years 6,250
Adjusted undistributed earnings P 13,750
NCI % 20%
NCI P 2,750

2. Pedro Company
Consolidated Income Statement
Year Ended December 31, 2011

Sales P300,000
Expenses (P245,000 + P6,250) 251,250
Consolidated net income P 48,750
Attributable to NCI 1,750
Attributable to controlling interest P 47,000

76
Problem 16-3

Amortization Schedule

Annual
Accounts Adjustments Life Amount 2008 2009 2010 2119
Inventory 1 P 6,250 P 6,250

Amortization:
Investments 3 5,000 5,000 5,000 5,000 5,000
Buildings 20 12,500 12,500 12,500 12,500 12,500
Equipment 5 34,500 34,500 34,500 34,500 34,500
Patent 10 2,250 2,250 2,250 2,250 2,250
Trademark 10 2,000 2,000 2,000 2,000 2,000
Discount on bonds payable 5 2,500 2,500 2,500 2,500 2,500
Total P 65,000 P 65,000 P 58,750 P 58,750 P58,750

Problem 16-4

Allocation Schedule
Price paid P206,000
Less: Book value of interest acquired 140,000
Excess P 66,000
Allocation:
Equipment P(40,000)
Buildings 10,000 (30,000)
Goodwill (not impaired) P 36,000

a. Investment in Stag Company – 12/31/09 (at acquisition cost) P 206,000

b. Non-controlling interest P -0-

c. Consolidated Net Income


Net income from own operations – Pony (P310,000 – P198,000) P 112,000
Net income from own operations – Stag (P104,000 – P74,000) 30,000
Amortization: Equipment (P40,000/8) P5,000
Buildings (P10,000/20) (500) ( 4,500)
Consolidated net income P 137,500

d. Consolidated Equipment
Total book value (P320,000 + P50,000) P 370,000
Allocation 40,000
Amortization (P5,000 x 3 years) (15,000)
Total P 395,000

77
Problem 16-4 concluded

e. Consolidated Buildings
Total book value P 288,000
Allocation ( 10,000)
Amortization (P500 x 3 years) 1,500
Total P 279,500

f. Consolidated Goodwill (not impaired) P 36,000

g. Consolidated Common Stock (Pony) P 290,000

h. Consolidated Retained Earnings


Retained earning, Dec. 31, 2011 – Pony P 410,000
Add: Pony’s share of Stag’s adjusted increase in earnings
Net earnings – 2011 (P30,000 – P20,000) P10,000
Amortization ( 4,500) 5,500
Retained earnings, December 31, 2011 P 415,500

Problem 16-5

a. Working Paper Elimination Entries, Dec. 31, 2011

(1) Dividend income 10,000


Dividends declared – Short 10,000
To eliminate intercompany dividends.

(2) Common stock – Short 100,000


Retained earnings – Short 50,000
Investment in Short Company 150,000
To eliminate equity accounts of Short at
date of acquisition

(3) Depreciable asset 30,000


Investment in Short Company 30,000
To allocate excess

(4) Depreciation expense 5,000


Depreciable asset 5,000
To amortize allocatedexcess

78
Problem 16-5 concluded

b. Pony Corporation and Subsidiary


Consolidation Working Paper
December 31, 2011

Pony Short Adjustments & Eliminations Consoli-


Corporation Company Debit Credit dated
Income Statement
Sales 200,000 120,000 320,000
Dividend income 10,000 (1) 10,000 -
Total 210,000 120,000 320,000
Depreciation 25,000 15,000 (3) 5,000 45,000
Other expenses 105,000 75,000 180,000
Total 130,000 90,000 225,000
Net income carried forward 80,000 30,000 95,000

Retained Earnings
Retained earnings, Jan. 1 230,000 50,000 (2) 50,000 230,000
Net income from above 80,000 30,000 95,000
Total 310,000 80,000 325,000
Dividends declared 40,000 10,000 (1) 10,000 40,000
Retained earnings, Dec. 31
Carried forward 270,000 70,000 285,000

Statement of FP
Cash 15,000 5,000 20,000
Accounts receivable 30,000 40,000 70,000
Inventory 70,000 60,000 130,000
Depreciable asset (net) 325,000 225,000 (3) 30,000 (4) 5,000 575,000
Investment in Short company 180,000 (2)150,000 -
(3) 30,000
Total 620,000 330,000 795,000

Accounts payable 50,000 40,000 90,000


Notes payable 100,000 120,000 220,000
Common stock
Pony 200,000 200,000
Short 100,000 (2)100,000
Retained earnings, Dec. 31
From above 270,000 70,000 285,000
Total 620,000 330,000 195,000 195,000 795,000

79
Problem 16-6

a. Working Paper Elimination Entries


(1) Dividend income 8,000
NCI 2,000
Dividends declared – Sisa 10,000

(2) Common stock – Sisa 100,000


Retained earnings – Sisa 50,000
Investment in Sisa stock 120,000
NCI 30,000

(3) NCI in net income of subsidiary 6,000


NCI 6,000

b. Popo Corporation and Subsidiary


Consolidated Working Paper
December 31, 2011
Popo Sisa Adjustments & Eliminations Consoli-
Corporation Company Debit Credit dated
Income Statement
Sales 200,000 120,000 320,000
Dividend income 8,000 (1) 8,000 -
Total revenue 208,000 120,000 320,000
Depreciation expense 25,000 15,000 40,000
Other expenses 105,000 75,000 180,000
Total expenses 130,000 90,000 220,000
Net income 78,000 30,000 100,000
NCI in net income of Sub. (3) 6,000 ( 6,000)
Net income carried forward 78,000 30,000 94,000

Retained Earnings
Retained earnings, 1/1 230,000 50,000 (2) 50,000 230,000
Net income from above 78,000 30,000 94,000
Total 308,000 80,000 324,000
Dividends declared 40,000 10,000 (1) 10,000 40,000
Retained earnings, 12/31
Carried forward 268,000 70,000 284,000

Statement of FP
Current assets 173,000 105,000 278,000
Depreciable assets 500,000 300,000 800,000
Investment in Sisa Company 120,000 (2)120,000 -
Total 793,000 405,000 1,078,000

Accumulated depreciation 175,000 75,000 250,000


Current liabilities 50,000 40,000 90,000
Long-term debt 100,000 120,000 220,000
Common stock 200,000 100,000 (2)100,000 200,000
Retained earnings , 12/31
From above 268,000 70,000 284,000
NCI (1) 2,000 (2) 30,000 34,000
(3) 6,000
Total 793,000 405,000 166,000 166,000 1,078,000

80
Problem 16-6 - Concluded
c. Consolidated Financial Statements

Popo Corporation and Subsidiary


Consolidated Statement of Financial Position
December 31, 2011

Assets
Current assets P278,000
Depreciable assets P800,000
Less: Accumulated depreciation 250,000 550,000
Total assets P828,000

Liabilities and Stockholders’ Equity


Current liabilities P 90,000
Long-term debt 220,000
Total liabilities P310,000
Stockholders’ Equity
Common stock P200,000
Retained earnings, 12/31 284,000
Minority interest in net assets of subsidiary 34,000 518,000
Total liabilities and stockholders’ equity P828,000

Popo Corporation and Subsidiary


Consolidated Income Statement
Year Ended December 31, 2011

Sales P320,000
Expenses:
Depreciation expense P 40,000
Other expenses 180,000 220,000
Consolidated net income P100,000
NCI in net income of subsidiary 6,000
Attributable to parent P 94,000

Popo Corporation and Subsidiary


Consolidated Retained Earnings
Year Ended December 31, 2011

Retained earnings, Jan. 1 – Popo P230,000


Consolidated net income attributable to parent 94,000
Total P324,000
Dividends paid – Popo 40,000
Consolidated retained earnings, Dec. 31 P284,000

81
Problem 16-7

a. Palo Corporation and Subsidiary


Consolidation Working Paper
December 31, 2011

Palo Sebo Adjustments & Eliminations Consoli-


Corporation Company Debit Credit dated
Income Statement
Sales 300,000 150,000 450,000
Investment Income 19,000 (1) 19,000 -
Total revenues 319,000 150,000 450,000
Cost of goods sold 210,000 85,000 295,000
Depreciation expense 25,000 20,000 45,000
Other expenses 23,000 25,000 48,000
Total cost and expenses 258,000 130,000 388,000
Net income carried forward 61,000 20,000 62,000

Retained Earnings
Retained earnings, Jan. 1 230,000 50,000 (2) 50,000 230,000
Net income from above 61,000 20,000 62,000
Total 291,000 70,000 292,000
Dividends declared 20,000 10,000 (1) 10,000 20,000
Retained earnings, Dec. 31
carried forward 271,000 60,000 272,000

Statement of FP
Cash 37,000 20,000 57,000
Accounts receivable 50,000 30,000 80,000
Inventory 70,000 60,000 130,000
Buildings and equipment 300,000 240,000 540,000
Investment in Sebo Company 229,000 (1) 9,000 -
(2)200,000
(3) 20,000
Goodwill (3) 20,000 20,000
Total 686,000 350,000 827,000

Accumulated depreciation 105,000 65,000 170,000


Accounts payable 40,000 20,000 60,000
Taxes payable 70,000 55,000 125,000
Common stock 200,000 150,000 (2)150,000 200,000
Retained earnings, Dec. 31
from above 271,000 60,000 272,000
Total 686,000 350,000 239,000 239,000 827,000

82
Problem 16-7 - Concluded
b. Consolidated Financial Statements

Palo Corporation and Subsidiary


Consolidated Income Statement
Year Ended December 31, 2011

Sales P450,000
Cost of goods sold 295,000
Gross profit 155,000
Expenses:
Depreciation expenses P45,000
Other expenses 48,000 93,000
Consolidated net income P 62,000

Palo Corporation and Subsidiary


Consolidated Retained Earnings
Year Ended December 31, 2011

Retained earnings, January 1 – Palo P230,000


Consolidated net income 62,000
Total 292,000
Dividends paid – Palo 20,000
Retained earnings, December 31 P272,000

Palo Corporation and Subsidiary


Consolidated Statement of Financial Position
December 31, 2011

Assets
Cash P 57,000
Accounts receivable 80,000
Inventory 130,000
Buildings and equipment P540,000
Less: Accumulated depreciation 170,000 370,000
Goodwill 20,000
Total P657,000

Liabilities and Stockholders’ Equity


Accounts payable P 60,000
Taxes payable 125,000
Common stock 200,000
Retained earnings, Dec. 31 272,000
Total P657,000

83
Problem 16-8

1. Determination and Allocation of Excess Schedule:


Company Parent Price NCI Value
Estimated FV (80%) (20%)
Fair value of subsidiary P945,000 P756,000 P189,000
Less book value of interest acquired:
Common stock – S Company 300,000
Retained earnings – S Company 400,000
Total equity 700,000 700,000 700,000
Interest acquired 80% 20%
Book value 560,000 140,000
Excess of fair value over book value 245,000 196,000 49,000
Allocations:
Inventory (30,000)
Land (50,000)
Building (100,000)
Equipment 75,000
Patent (40,000)
Total 145,000

Goodwill P 100,000

Working Paper Elimination Entries - December 31, 2011(not required)

(1) Investment income 94,800


NCI 10,000
Dividends declared – S Company 50,000
Investment in S Company 54,800

(2) Common stock – S 300,000


Retained earnings, Jan. 1 – S 400,000
Investment in S Company 560,000
NCI 140,000

(3) Inventories 30,000


Land 50,000
Building 100,000
Patents 40,000
Goodwill 100,000
Equipment 75,000
Investment in S Company 196,000
NCI 49,000

(4) Cost of goods sold 30,000


Inventory 30,000

Equipment (P75,000 / 10) 7,500


Expenses (amortization) 1,500
Buildings (P100,000 / 20) 5,000
Patents (P40,000 / 10) 4,000

(5) NCI in net income of subsidiary 23,700


NCI 23,700
To recognize NCI in subsidiary net income (P150,000 – 31,500)x 20%

84
Problem 16-8, Concluded
2. P Company and Subsidiary
Consolidated Working Paper
Year Ended December 31, 2011
P S Adjustments & Eliminations Consoli-
Company Company Debit Credit dated
Income Statement
Sales 1,000,000 500,000 1,500,000
Cost of sales 400,000 150,000 (4) 30,000 580,000
Gross profit 600,000 350,000 920,000
Expenses 360,000 200,000 (4) 1,500 561,500
Operating income 240,000 150,000 358,500
Investment income 94,800 - (1) 94,800 -
Net /consolidated income 334,800 150,000 358,500
NCI in net income of
Subsidiary (5) 23,700 (23,700)
Net income carried forward 334,800 150,000 334,800

Retained earnings
Retained earnings, 1/1 600,000 400,000 (2)400,000 600,000
Net income from above 334,800 150,000 334,800
Total 934,800 550,000 934,800
Dividends declared 100,000 50,000 (1) 50,000 100,000
Retained earnings, 12/31
Carried forward 834,800 500,000 834,800

Statement of FP
Cash 200,000 100,000 300,000
Accounts receivable 150,000 50,000 200,000
Inventories 100,000 40,000 (3) 30,000 (4) 30,000 140,000
Land 150,000 (3) 50,000 200,000
Buildings (net) 200,000 (3)100,000 (4) 5,000 295,000
Equipment (net) 298,000 450,000 (4) 7,500 (3) 75,000 680,500
Patent - - (3) 40,000 (4) 4,000 36,000
Investment in S Company 810,800 (1) 54,800 -
(2)560,000
(3)196,000
Goodwill (3) 100,000 100,000
Total 1,558,800 1,090,000 1,951,500

Accounts payable 124,000 190,000 314,000


Common stock 200,000 300,000 (2)300,000 200,000
Additional paid-in capital 400,000 - 400,000
Retained earnings, 12/31
from above 834,800 500,000 834,800
NCI (1) 10,000 (2)140,000 2022,700
(3) 49,000
(5) 23,700
Total 1,558,800 1,090,000 486,200 486,200 1,951,500

85
Problem 16-9

a. Investment in Sally Products Co. 160,000


Cash 160,000
To record acquisition of 80% stock of Sally.

Cash 8,000
Dividend income 8,000
To record dividends received from Sally (P10,000 x 80%)

b. Working Paper Eliminating Entries – Dec. 31, 2011

(1) Dividend income 8,000


NCI 2,000
Dividends declared – Sally 10,000

(2) Common stock – Sally 100,000


Retained earnings, 1/1/08 –Sally 50,000
Investment in Sally Products 120,000
NCI 30,000

(3) Building and equipment 50,000


Investment in Sally Products 40,000
NCI 10,000

(4) Retained earnings, 1/1 – Sally (prior year) 5,000


Depreciation expense (current year) 5,000
Accumulated depreciation – Bldg 10,000

(5) Accounts payables 10,000


Cash and receivables 10,000

(6) NCI in net income of subsidiary 5,000


NCI 5,000
(P30,000 – P5,000) x 20%

(7) Retained earnings, 1/1 – Sally 7,000


NCI 7,000
To recognize NCI in subsidiary’s prior year earnings
[(P50,000 – P90,000) – P5,000] x 20%

86
Problem 16-9, Concluded
c. Pilar Corporation and Subsidiary
Consolidation Working Paper
December 31, 2011

Pilar Sally Wood Adjustments & Eliminations Consoli-


Corporation Products Debit Credit dated
Income Statement
Sales 200,000 100,000 300,000
Dividend income 8,000 (1) 8,000 -
Total revenue 208,000 100,000 300,000

Cost of goods sold 120,000 50,000 170,000


Depreciation expense 25,000 15,000 (4) 5,000 45,000
Inventory losses 15,000 5,000 20,000
Total cost and expenses 160,000 70,000 235,000
Net /consolidated income 48,000 30,000 65,000

NCI in net income of


subsidiary (6) 5,000 (5,000)

Net income carried forward 48,000 30,000 60,000

Retained earnings statement


Retained earnings, 1/1 298,000 90,000 (2) 50,000 326,000
(4) 5,000
(7) 7,000
Net income from above 48,000 30,000 60,000
Total 346,000 120,000 386,000
Dividends declared 30,000 10,000 (1) 10,000 30,000
Retained earnings, 12/31
carried forward 316,000 110,000 356,000

Statement of FP
Cash and receivables 81,000 65,000 (5) 10,000 136,000
Inventory 260,000 90,000 350,000
Land 80,000 80,000 160,000
Buildings and equipment 500,000 150,000 (3) 50,000 700,000
Investment in Sally 160,000 (2)120,000 -
(3) 40,000
Total 1,081,000 385,000 1,346,000

Accumulated depreciation 205,000 105,000 (4) 10,000 300,000


Accounts payable 60,000 20,000 (5) 10,000 70,000
Notes payable 200,000 50,000 250,000
Common stock 300,000 100,000 (2)100,000 300,000
Retained earnings from above 316,000 110,000 356,000
NCI (1) 2,000 (2) 30,000 50,000
(3) 10,000
(6) 5,000
(7) 7,000

Total 1,081,000 385,000 242,000 242,000 1,346,000

87
Problem 16-10

Determination and Allocation of Excess Schedule (not required)

Price paid P220,000


Less book value of interest acquired:
Common stock – Star Company P150,000
Retained earnings, 1/1 – Star Company 50,000 200,000
Goodwill P 20,000

a. Eliminating entries:

E(1) Dividend Income 20,000


Dividends Declared 20,000
Eliminate dividend income from subsidiary.

E(2) Common Stock – Star Company 150,000


Retained Earnings, January 1 50,000
Investment in Star Company Stock 200,000
Eliminate subsidiary equity accounts.

E(3) Goodwill 8,000


Retained Earnings, January 1 12,000
Investment in Star Company 20,000
Assign excess at beginning of year

Porno Corporation and Star Company


Consolidated Working Paper
December 31, 2011

Porno Star Eliminations


_____Item_____ Corporation Company Debit Credit Consolidated
Income Statement
Sales 350,000 200,000 550,000
Dividend income 20,000 - (1) 20,000 _______
Credits 370,000 200,000 550,000
Cost of goods sold 270,000 135,000 405,000
Depreciation expense 25,000 20,000 45,000
Other expenses 21,000 10,000 31,000
Debits (316,000) (165,000) __ - ____ (481,000)
Net income, carry forward 54,000 35,000 20,000 - 69,000

Retained Earnings Statement


Retained earnings, Jan. 1 262,000 60,000 (2) 50,000
(3) 12,000 260,000
Net income, from above 54,000 35,000 20,000 69,000
316,000 95,000 329,000
Dividends declared (20,000) (20,000) ___ - (1) 20,000 (20,000)
Retained earnings, Dec. 31,
carry forward 296,000 75,000 82,000 20,000 309,000

88
Problem 16-10, Concluded

Statement of FP
Cash 46,000 30,000 76,000
Accounts receivable 55,000 40,000 95,000
Inventory 75,000 65,000 140,000
Buildings and equipment 300,000 240,000 540,000
Investment in Star Company 220,000 (2)200,000
(3) 20,000
Goodwill - - (3) 8,000 8,000
Debits 696,000 375,000 859,000

Accumulated depreciation 130,000 85,000 215,000


Accounts payable ` 20,000 30,000 50,000
Taxes payable 50,000 35,000 85,000
Common stock
Light Corporation 200,000 200,000
Star Company 150,000 (2)150,000
Retained earnings, from above 296,000 75,000 82,000 20,000 309,000
Credits 696,000 375,000 240,000 240,000 859,000

89
90
CHAPTER 17

MULTIPLE CHOICES - COMPUTATIONAL


17-1: b

Consolidated sales
Sales – Papa P 900,000
Sales – San 500,000
Elimination of inter-company sales ( 50,000)
Consolidated sales P 1,350,000

Consolidated cost of goods sold


Cost of goods sold – Papa P 490,000
Cost of goods sold – San 190,000
Eliminations:
Realized profit in beginning inventory (4,000)
Unrealized profit in ending inventory 10,000
Intercompany purchases ( 50,000)
Consolidated cost of goods sold P 636,000

17-2: c

Net income – Sisa P 60,000


Unrealized profit in ending inventory – upstream ( 10,000)
Adjusted net income – Sisa P 50,000
NCI proportionate share 20%
NCI in net income of subsidiary P 10,000

17-3: d

Net income from own operation – Pat P 200,000


Adjusted net income of Susan:
Net income – Susan P200,000
Realized profit in beginning inventory
(P112,000 x 50%/150%) 37,500
Unrealized profit in ending inventory
(P33,000 x 50%/150%) (11,000) 226,500
Consolidated net income P 426,500
Attributable to NCI (P226,500 x 30%) 67,950
Attributable to parent P 358,550

17-4: b

Net income from own operations- Patton P 300,000


Unrealized profit in ending inventory – DS (P200,000 x .25) (50,000)
Adjusted net income for own operations – Patton 250,000
Solis net loss from own operations (150,000)
Consolidated net income P 100,000

90
17-5: d

Pardo’s share of Santos’ net income (P300,000 x 75%) P 225,000


Unrealized profit in ending inventory – Upstream
(P200,000 x 25%/125%) x 75% ( 30,000)
Realized profit in beginning inventory – Upstream
(P150,000 x 25%/125%) x 75% 22,500
Investment income account balance, Dec. 31, 2011 P 217,500

17-6: d
Net income from own operation – Puzon P 200,000
Suazon’s adjusted net income from own operations:
Net income P110,000
Unrealized profit in ending inventory-
Upstream (P25,000 x 40%) ( 10,000) 100,000
Consolidated net income P 300,000
Attributable to NCI (P100,000 x 25%) (25.000)
Attributable to parent P 275,000

17-7: b
2010 2011
Net income from own operation – Pat P 500,000 P 550,000
Unrealized profit in ending inventory:
2010 (P20,000 x .40) (8,000)
2011 (P30,000 x .50) (15,000)
Realized profit in beginning inventory 8,000
Realized income 492,000 543,000
Sun net income 200,000 225,000
Consolidated net income P 692,000 P 768,000

17-8: a

Net income from own operation – Pip P 400,000


Adjusted net income from own operation - Sol
Net income P 250,000
Realized profit in beginning inventory-
Upstream (P40,000 x 40%) 16,000
Unrealized profit in ending inventory-
Upstream (P70,000 x 30%) ( 21,000) 245,000
Consolidated net income - 2011 P 645,000

91
17-9: a
Net income from own operations – Popo P 500,000
Unrealized profit in ending inventory – Downstream ( 15,000)
Realized net income from own operation – Popo P 485,000
Adjusted net income from own operations - Sotto
Net income P 360,000
Realized profit in beginning inventory-
Upstream 10,000 370,000
Consolidated net income P 855,000
Attributable to NCI (P370,000 x 5%) 18,500
Attributable to parent P 836,500

17-10: d, should be P(19,200).


Net income – Sand Company P200,000
Realized profit in beg. Inventory (P120,000 x .20) 24,000
Unrealized profit in ending inventory (P360,000 x .20) (72,000)
Amortization of allocated excess P1.000,000 / 5) (200,000)
Adjusted net loss – Sand Company P(48,000)
NCI (P48,000 x 40%) P(19,200)

17-11: d
Gross profit rate – Short (P110,000 / P200,000) 55%

Inventories
Inventory from outsiders – Power P 5,000
Inventory from outsiders – Short 25,000
Power’s inventory acquired from Short – at cost:
[P5,000 – (P5,000 x 55%)} 2,250
Consolidated ending inventories P 32,250

Investment income
Power’s share of Short’s net income (P50,000 x 75%) P 37,500
Unrealized profit in ending inventory – upstream
(P5,000 x 55%) x 75% ( 2,063)
Realized profit in beginning inventory – upstream
(P10,000 x 55%) x 75% 4,125
Investment income, Dec. 31, 2011 P 39,562

Investment in Short Company


Acquisition cost (P80,000 x 80%) P 60,000
Unrealized profit in ending inventory ( 2,063)
Realized profit in beginning inventory 4,125
Investment in Short Company, Dec. 31, 2011 P 62,062

NCI in Short company’s net income


Short’s net income from own operations P 50,000
Realized profit in beginning inventory (P10,000 x 55%) 5,500
Unrealized profit in ending inventory (P5,000 x 55%) ( 2,750)
Adjusted net income from own operations P 52,750
NCI proportionate share 25%
NCI in Short’s net income P 13,187.50

92
17-12: b

Gross profit rate of Sit (P200,000 / P500,000) 40%

Net income from own operations – Pit P 200,000


Adjusted net income of Sit:
Net income P 75,000
Realized profit in beginning inventory-
Upstream (P40,000 x 40%) 16,000
Unrealized profit in ending inventory-
Upstream (P25,000 x 40%) ( 10,000) 81,000
Consolidated net income P 281,000
Attributable to NCI (P81,000 x 10%) ( 8,100)
Attributable to parent P 272,900

17-13: b

Gross profit of Sir (P120,000 / P400,000) 30%

Consolidated cost of sales


Cost of sales – Pig P 600,000
Cost of sales – Sir 280,000
Eliminations:
Realized profit in beginning inventory (P70,000 x 30%) ( 21,000)
Unrealized profit in ending inventory (P60,000 x 30%) 18,000
Intercompany purchases (200,000)
Consolidated cost of sales P 677,000

Consolidated net income


Net income from own operations – Pig P 200,000
Sir’s adjusted net income:
Net income P 80,000
Realized profit in beginning inventory 21,000
Unrealized profit in ending inventory (18,000) 83,000
Consolidated net income 283,000
Attributable to NCI (P83,000 x 10%) (8,300)
Attributable to parent P 274,700

93
17-14: a
2009 2010 2011
Pal Corp net income 150,000 240,000 300,000
Intercompany profit in ending inventory:
2009 (14,000) 14,000
2010 (21,000) 21,000
2011 ( 24,000)
Pal net income from own operation 136,000 233,000 297,000
Solo net income from own operation 100,000 90,000 160,000
Consolidated net income 236,000 323,000 427,000
Attributable to NCI
2009(100,000 – 14,000) x 40% 34,400
2010(90,000 +14,000 – 21,000) 40% 33,200
2011(160,000 + 21,000 – 24,000) 40% 62,800
Attributable to Parent 201,600 289,800 394,200

17-15: a

Total sales 600,000


Intercompany sales (30,000 + 80,000) (110,000)
Consolidated sales 490,000

17-16: c
Total cost of goods sold (250,000 +120,000) 370,000
Adjustments due to intercompany sale:
COGS charged for intercompany sale (20,000 + 50,000) 70,000
COGS charged by: Star (30,000 – 6,000) 24,000
Polo (80,000 – 20,000) 60,000
Total 154,000
Cost of goods sold for consolidated entity:
20,000 x (24,000/30,000) (16,000)
50,000 x (60,000/80,000) (37,500) (100,500)
Consolidated cost of goods sold 269,500

17-17: c
Polo Corp. net income from own operation (105,000 – 25,000) 80,000
Unrealized profit in ending inventory-DS (6,000 x 10/30) (2,000)
Adjusted Polo Corp. net income from own operation 78,000
Star Corp. net income from own operation:
Net income 45,000
Unrealized profit in EI-US (20,000 x 30/80) (7,500)
Amortization (20,000/10 years) (2,000) 35,500
Consolidated net income 113,500
Attributable to NCI (35,500 x 40%) (14,200)
Attributable to Parent 99,300

94
17-18: a

Pepsi net income from own operation 160,000


Sarsi net income 90,000
Unrealized profit in EI (45,000 x 60/180) (15,000) 75,000
Consolidated net income 235,000
Attributable to NCI (75,000 x 30%) (22,500)
Attributable to Parent-2011 212,500

17-19: a

Inventory-Pepsi P 30,000
Less: unrealized profit in books of Sarsi:
(135,000 – 90,000) x (30,000/135,000) (10,000) 20,000
Inventory-Sarsi P110,000
Less: unrealized profit in books of Pepsi:
(280,000 – 140,000) x (110,000/280,000) (55,000) 55,000
Consolidated inventory 12/31/11 75,000

17-20: a

Cost of goods sold on sale of inventory on hand-1/1/10:


[45,000 x (120,000/180,000)] 30,000
Cost of goods sold on purchases from Sarsi- 2010
[(135,000 – 30,000) x (90,000/135,000)] 70,000
Cost of goods sold on purchases from Pepsi- 2010
[(280,000 – 110,000) x (140,000/280,000)] 85,000
Consolidated cost of goods sold-2011 185,000

17-21: b

Pepsi net income 220,000


Sarsi net income 85,000
Realized profit in beginning inventory - 2011 15,000
Unrealized profit in ending inventory- Sarsi (10,000)
Unrealized profit in ending inventory- Pepsi (55,000)
Consolidated net income – 2011 255,000

95
17-22: b
Net income from own operations – P Company P200,000
S Co. adjusted net income:
Net income – S P30,000
Unrealized profit in ending inventory –
Upstream (P9,000 x 50/150) (3,000)
Realized profit in beginning inventory-
Upstream (P6,000 x 50/150) 2,000 29,000
Consolidated net income 229,000
Attributzble to NCI (P29,000 x 30%) 8,700
Attributable to parent P220,300

17-23: b

NCI, December 31, 2010 [(P245,000/70%) x 30%] P105,000


NCI in subsidiary dividends (P20,000 x 30%) ( 6,000)
NCI in net income of subsidiary 8,700
NCI in S Company, December 31, 2011 P107,700

17-24: c

P Company (P400,000 x 20%) P 80,000

S Company:
Sales P416,000
Cost of goods sold (P400,000 x 80%) P320,000
Add write down of ending inventory 10,000 330,000
Gross profit P 86,000

17-25 a

Sales P416,000
Consolidated cost of goods sold 256,000*
Gross profit P160,000

* Purchases at cost (P400,000 x 80%) P320,000


Less ending inventory at cost (P80,000 x 80%) 64,000
Consolidated cost of goods sold P256,000

Note that cost is lower than market

96
17-26: a

Sales P270,000
Cost of goods sold 171,250
Gross profit 98,750
Other income -
Other expenses 47,000
Consolidated net income 51,750
Attributable to NCI 3,350
Attributable to controlling interest P 48,400

Supporting computations:

Sales:
Pablo Company P220,000
Sally Company 120,000
Intercompany sales (70,000)
Consolidated sales P270,000

Consolidated cost of goods sold:


Pablo Company P150,000
Sally Company 90,000
Intercompany sales ( 70,000)6
Realized profit in beginning inventory (P15,000 x 25%) ( 3,750)
Unrealized profit in ending inventory (P20,000 x 25%) 5.000
Consolidated costs of goods sold P171,250

Other income:
Pablo Company P 5,000
Computer services (5,000)
:
Other expenses:
Pablo Company P 40,000
Sally Company 12,000
Computer services (5,000)
Consolidated other expenses P 47,000

NCI in net income of Sally


Net income P 35,000
Realized profit in beginning inventory (upstream) 3,750
Unrealized profit in ending inventory (upstream) (5,000)
Adjusted net income P 16,750
NCI proportionate share 20%
NCI P 3,350

97
PROBLEMS

Problem 17-1

The computation of the selected consolidation balances are affected by the inter-company profit
in downstream intercompany sales as computed below:

Unrealized profit in ending inventory, Dec. 31, 2010 – Downstream


Intercompany profit (P120,000 – P72,000) P 48,000
Inventory left at year end x 30%
Unrealized profit, Dec. 31, 2010 P 14,400

Unrealized profit in ending inventory, Dec. 31, 2011 – Downstream


Intercompany profit (P250,000 – P200,000) P 50,000
Inventory left at year end x 20%
Unrealized profit, Dec. 31, 2011 P 10,000

a. Consolidated Sales
Apo P800,000
Bicol 600,000
Intercompany sales – 2011 (250,000)
Total P1,150,000
b. Cost of goods sold
Apo’s book value P 535,000
Bicol’s book value 400,000
Intercompany sales-2011 (250,000)
Realized profit in beginning inventory – 2011 ( 14,400)
Unrealized profit in ending inventory – 2011 10,000
Consolidated cost of goods sold P 680,600
c. Operating expenses
Apo P 100,000
Bicol 100,000
Total P 200,000

d. Dividend Income – 0 (eliminated)

e. NCI in Net Income of Subsidiary (P100,000 x 20%) P 20,000

f. Inventory
Apo P 298,000
Bicol 700,000
Unrealized profit in ending inventory, Dec. 31, 2011 (10,000)
Consolidated inventory P 988,000

98
Problem 17-1, continued:
g. NCI
NCI, December 31, 2010 [ (P902,000/80%) x 20%] P225,500
NCI in dividends paid by Bicol (P50,000 x 20%) (10,000)
NCI in net income of subsidiary (P100,000 x 20%) 20,000
Total NCI, 12/31/11 P235,500

Problem 17-2

P Company and Subsidiary


Consolidated Income Statement
Year Ended December 31, 2011

Sales (P2,000,000 + P1,000,000 – P600,000) P2,400,000


Cost of goods sold (Schedule 1) 704,000
Gross profit 1,696,000
Expenses 600,000
Income before income tax 1,096,000
Provision for income tax 440,000
Consolidated net income after income tax 656,000
Attributable to NCI (Schedule 2) 44,000
Attributable to parent P 612,000

Schedule 1:
Cost of sales – P Company P 800,000
Purchases from S Company (600,000)
Intercompany profit in beginning inventory (P60,000 x 25%) ( 15,000)
Intercompany profit in ending inventory (P76,000 x 25%) 19,000
Total P 204,000
Cost of sales – S Company 500,000
Consolidated cost of sales P 704,000

Schedule 2:
Net income – S Company P 180,000
Realized profit in beginning inventory – Upstream 15,000
Unrealized profit in ending inventory – Upstream (19,000)
Adjusted net income P 176,000
NCI proportionate share x 25%
NCI in net income of subsidiary P 44,000

Problem 17-3

a. Working Paper Eliminating Entries

(1) Dividend income 32,000


NCI (20%) 8,000
Dividends declared- D (P32,000 / 80%) 40,000
To eliminate intercompany dividends.

99
Problem 17-3, Continued

(2) Common stock – S 90,000


Retained earnings – S 220,000
Investment in S Co. stock 248,000
NCI 62,000
To eliminate equity accounts of S on the date of
acquisition.

(3) NCI 4,000


Retained earnings, Jan. 1 16,000
Cost of goods sold 20,000
To eliminate realized profit in beginning inventory

(4) Sales 150,000


Cost of goods sold 135,000
Inventory, Dec. 31 (P45,000 x 33.33%) 15,000
To eliminated intercompany sales and unrealized
profit in ending inventory.

(5) NCI in net income of subsidiary 9,000


NCI 9,000
To establish minority interest in net income of S Co.
computed as follows:
Sales P200,000
Cost and expenses (P140,000 +P20,000) 160,000
Net income 40,000
Realized profit in beginning inventory – Upstream 20,000
Unrealized profit in ending inventory – Upstream (15,000)
Adjusted net income P 45,000
NCI proportionate share x 20%
NCI in net income of subsidiary P 9,000

b. Consolidated Net Income


P Company net income from own operations (P250,000 – P32,000) P 218,000
S Company adjusted net income 45,000
Consolidated net income P 263,000

c. Non-controlling Interest
NCI, August 30, 2011 [(P248,000/80%) x 20%] P 62,000
NCI in subsidiary dividends [(P32,000/80%) x 20%] ( 8,000)
NCI in net income of subsidiary 9,000
NCI P 63,000

100
Problem 17-4

a. Consolidated Sales
Reported total sales (P600,000 + P510,000) P1,170,000
Intercompany sales (P140,000 + P240,000) (380,000)
Consolidated sales P 790,000

b. Consolidated Cost of Goods Sold


Cost of goods sold:
Pato (P660,000 / 140%) P 471,429
Sales (P510,000 / 120% 425,000
Amount to be eliminated (P128,000 + P232,000) see entry below ( 360,000)
Total P 536,429

Elimination of intercompany sales and intercompany profit in inventory:

Downstream Sales
Sales 140,000
Inventory (P42,000 x 40/140) 12,000
Cost of goods sold 128,000

Upstream Sales
Sales 240,000
Inventory (P48,000 x 20/120) 8,000
Cost of goods sold 232,000

c. Consolidated Net Income


Net income from own operations – Pato P 70,000
Unrealized profit in ending inventory – Downstream (12,000)
Adjusted net income – Pato P 58,000
Adjusted net income of Sales Co.
Net income P20,000
Unrealized profit in ending inventory – Upstream (8,000) 12,000
Consolidated net income P 70,000

d. Consolidated Inventory, Dec. 31, 2011


Inventory reported – Pato P 48,000
Inventory reported – Sales 42,000
Unrealized profit in ending inventory (P8,000 + P12,000) (20,000)
Consolidated inventory P 70,000

101
Problem 17-5

P Company and Subsidiary S Company


Consolidation Working Paper
Year Ended December 31, 2011

Eliminations Adjustments Consoli-


P Company S Company Debit Credit dated

Income Statement
Sales 12,000,000 1,300,000 (5) 400,000 12,900,000
Dividend income 210,000 (1) 210,000 -
Total revenue 12,210,000 1,300,000 12,900,000
Cost of goods sold 7,000,000 750,000 (7) 30,000 (5) 400,000 7,380,000
Operating expenses 4,210,000 50,000 (4) 40,000 4,300,000
Total cost and expenses 11,210,000 800,000 11,680,000

Net income to retained earnings 1,000,000 500,000 1,220,000

Statement of Retained
Earnings
Retained earnings, January 1 5,500,000 2,200,000 (2)2,200,000 5,500,000
Net income from above 1,000,000 500,000 1,220,000
Total 6,500,000 2,700,000 6,720,000
Dividends declared - 210,000 (1) 210,000 -
Retained earnings,12/31 to BS 6,500,000 2,490,000 6,720,000

Statement of FP
Cash 810,000 170,000 980,000
Accounts receivable 425,000 445,000 (6) 25,000 845,000
Inventory 600,000 275,000 (7) 30,000 845,000
Property, plant and equipment 4,000,000 2,300,000 (3) 400,000 (4) 40,000 6,660,000
Investment in S Company 3,200,000 (2)2,800,000 -
(3) 400,000

Total assets 9,035,000 3,1900,000 9,330,000

Accounts payable 35,000 100,000 (6) 25,000 110,000


Common stock 1,000,000 400,000 (2) 400,000 1,000,000
Additional paid in capital 1,500,000 200,000 (2) 200,000 1,500,000
Retained earnings from above 6,500,000 2,490,000 6,720,000

9,035,000 3,190,000 3,905,000 3,905,000 9,330,000

Eliminations and Adjustments


(1) Eliminate intercompany dividends
(2) Eliminate subsidiary’s equity balances
(3) Allocate excess to equipment
(4) Amortize allocated excess to equipment
(5) Eliminate intercompany sale of P400,000
(6) Eliminate intercompany trade balances of P25,000
(7) Eliminate intercompany profit (30%) applicable to P100,000 (P400,000 – P300,000)
of intercompany goods in P Company.

102
Problem 17-5, Continued

Determination and Allocation of Excess Schedule

Price paid by the parent P3,200,000


Less book value of interest acquired (100%)
Common stock – S Company P 400,000
Additional paid in capital – S Company 200,000
Retained earnings, Jan. 1 – S Company 2.200,000 2,800,000
Excess allocated to equipment P 400,000

Amortization (P400,000/10) P 40,000

Note: There is no NCI since this is a wholly-owned subsidiary.

Problem 17-6

Determination and Allocation of Excess Schedule:

Price paid by the parent (80%) P425,000


Non-controlling interest [(P425,000/80%) x 20%] 106,250
Total 531,250
Less book value of interest acquired:
Common stock – So P200,000
APIC – So 100,000
Retained earnings 100,000
Total equity P400,000
Interest acquired 80% 320,000
Excess allocated to goodwill P131,250

Fair Value Analysis:


Company Parent NCI
Implied Price Value
Fair Value (80%) (20%)

Company fair value P531,250 P425,000 P106,250


Fair value of net assets excluding goodwill 400,000 320,000 80,000
Goodwill P131,250 P105,000 P 26,250

103
Po Company and Subsidiary So Company
Consolidation Working Paper
Year Ended December 31, 2011
Eliminations Adjustments Consoli-
Po Company So Company Debit Credit dated
Income Statement
Sales 880,000 630,000 (6) 32,000
(8) 30,000 1,448,000
Dividend income 24,000 (2) 24,000 -
Total revenue 904,000 630,000 1,448,000
Cost of goods sold 704,000 504,000 (7) 1,320 (5) 1,350
(10) 750 (6) 32,000
(8) 700
(9) 30,000 1,146,020
Other expenses 130,000 81,000 211,000
Total cost and expenses 834,000 585,000 1,357,020
Net income 70,000 45,000 90,980
NCI in net income of Subsidiary (12) 8,990 (8,990)
Net income to retained earnings 70,000 45,000 81,990

Statement of Retained
Earnings
Retained earnings, January 1 1,105,000 140,000 (1) 8,000
(3)100,000
(5) 1,350
(8) 560 1,135,090
Net income from above 70,000 45,000 81,990
Total 1,175,000 185,000 1,217,080
Dividends declared 25,000 30,000 (2) 30,000 25,000
Retained earnings,12/31 to BS 1,150,000 155,000 1,192,080

Statement of FP
Cash 216,200 44,300 260,500
Accounts receivable 290,000 97,000 (11) 15,000 372,000
Inventory 310,000 80,000 (7) 1,320
(10) 750 387,930
Pant assets (net) 1,991,000 340,000 2,331,000
Investment in S Company 425,000 (3)320,000
(4)105,000 -
Goodwill 60,000 (4)131,250 191,250
Total assets 3,292,200 561,300 3,542,680

Accounts payable 642,200 106,300 (11) 15,000 733,500


Common stock 250,000 200,000 (3)200,000 250,000
Additional paid in capital 1,250,000 100,000 (3)100,000 1,250,000
Retained earnings from above 1,150,000 155,000 1,192,080
Non-controlling interest (NCI) (2) 6,000 (1) 8,000
(8) 140 (3) 80,000
(4) 26,250
(12) 8,990 117,100
3,292,200 561,300 659,360 659,360 3,542,680

104
Eliminations and Adjustments
(1) Recognize NCI in subsidiary’s increase in undistributed earnings (P40,000 x 20%)
(2) Eliminate intercompany dividends.
(3) Eliminate subsidiary’s equity at date of acquisition
(4) Allocate excess to goodwill.
(5) Eliminate realized profit in beginning inventory (P9,000 x 15%) = P1,350
(Downstream)
(6) Eliminate intercompany downstream sales from April 1, 2008 to March 31, 2009,
P32,000.
(7) Eliminated unrealized profit in ending inventory (downstream), P6,000 x 22% =
P1,320.
(8) Eliminate realized profit in beginning inventory (upstream) P3,500 x 20% = P700.
(9) Eliminate intecompany upstream sales on March 31, 2009, P30,000.
(10) Eliminate unrealized profit in ending inventory (upstream), P3,000 x 25% = P750.
(11) Eliminate intercompany payables and receivables ,P10,000 + P5,000 = P15,000.
(12) Recognized non-controlling interest (NCI) in net income of subsidiary computed as
follows:

Net income of So Company P45,000


Realized profit in beginning inventory (upstream) 700
Unrealized profit in ending inventory (upstream) (750)
Adjusted income P44,950
NCI share 20%
NCI in net income of subsidiary P 8,990

(2)
Po Company and Subsidiary So Company
Consolidated Income Statement
Fiscal Year Ended March 31, 2011

Sales P1,448,000
Cost of goods sold 1,146,020
Gross profit 301,980
Expenses 211,000
Consolidated net income P 90,980
Attributable to NCI 8,990
Attributable to controlling interest P 81,990

105
Problem 17-7

a. Unrealized Profit in Beginning Inventory


Beginning inventory - Downstream P 100,000
Gross profit rate (P240,000/ P400,000) x 60%
Unrealized profit in beginning inventory P 60,000

Unrealized Profit in Ending Inventory


Ending inventory – Downstream (P200,000 x 80%) P 160,000
Gross profit rate x 60%
Unrealized profit in ending inventory P 96,000

b. Intercompany Sales
Sales – P Company P2,000,000
Sales – S Company 1,000,000
Intercompany sales – 2011 (400,000)
Consolidated sales P2,600,000

Intercompany Cost of Sales


Cost of sales – P Company P 800,000
Cost of sales – S Company 600,000
Intercompany purchases (400,000)
Intercompany profit in beginning inventory ( 60,000)
Intercompany profit in ending inventory 96,000
Consolidated cost of sales P1,036,000

c. Parent’s interest (40,000 shares / 50,000 shares) 80%

P Company Entries – 2011:


(1) Investment in S Company stock 96,000
Income from subsidiary 96,000
To record P’s share of S Co. income
(P120,000 x 80%)

(2) Cash 48,000


Investment in S Company stock 48,000
To record dividends received from S
(P60,000 x 80%)

(2) Income from subsidiary 36,000


Investment in S Company 36,000
To adjust income from subsidiary for intercompany
profit in :
Ending inventory (96,000)
Beginning inventory 60,000
Net adjustment ( 36,000)

106
Problem 17-7, continued:
d. Working Paper Eliminating Entries:

(1) Income from subsidiary 60,000


NCI (P60,000 x 20%) 12,000
Dividends declared – S 60,000
Investment in S Company 12,000
To eliminate intercompany dividends.

(2) Common stock – S Co. 500,000


Retained earnings – S Co. 860,000
Investment in S Company stock 1,088,000
NCI 272,000
To eliminate equity accounts of S Company as of
beginning of year.

(3) Goodwill 60,000


Investment in S Company 60,000
To allocate excess to goodwill.

(4) Retained earnings – Jan. 1 60,000


Cost of sales 60,000
To eliminate realized profit in beginning inventory-
Downstream.

(5) Cost of sales 96,000


Inventories 96,000
To eliminate unrealized profit in ending inventory-
Downstream.

(6) Sales 400,000


Cost of sales 400,000
To eliminate intercompany sales.

(7) Accounts payable 50,000


Accounts receivable 50,000
To eliminate intercompany payables and receivables.

(8) NCI in net income of subsidiary 24,000


NCI 24,000
To recognize NCI share in S Company net income
(P120,000 x 20%)

e. Consolidated Net Income


Net Income from own operations – P Company (P480,000 – P60,000) P420,000
Realized profit in beginning inventory 60,000
Unrealized profit in ending inventory ( 96,000)
Adjusted net income – P Compay P384,000
S Company net income 120,000
Consolidated net income P504,000

107
CHAPTER 18

MULTIPLE CHOICES – COMPUTATIONAL

18-1: a

Equipment – at original cost P500,000

Accumulated depreciation:
Time of sale P250,000
Current depreciation based on
Original cost (P500,000/10 years 50,000 P300,000

18-2: b

Net income – Sol P100,000


Unrealized gain on sale of computer, Dec. 31 ( 30,000)
Adjusted net income P 70,000
NCI proportionate share 30%
NCI in net income of subsidiary P 21,000

18-3: b
2005 2006
Net income from own operations – Prime P200,000 P250,000
Unrealized gain – Downstream (30,000) __-
Adjusted net income – Prime P170,000 P250,000
Second Company net income 100,000 150,000
Consolidated net income P270,000 P400,000

18-4: c

Net income – Saw P100,000


Unrealized loss-Upstream 12,000
Realized loss ((P12,000 / 5) x 6/12 ( 1,200)
Adjusted net income – Saw P110,800

NCI in net income of subsidiary (P110,800 x 25%) P 27,700

18-5: c

Equipment – at original cost P1,000,000

Accumulated depreciation:
Time of sale P360,000
Current depreciation (P900,000/10) 90,000 P 450,000

108
18-6: a

Adjusted net income – Susie (P12,000 / 40%) P 30,000


Add back unrealized gain – Upstream 90,000
Net income of Susie – 2011 P120,000

18-7: a

Original cost P100,000


Amount debited to Truck account (48,000)
Selling price of the truck – Amount paid P 52,000

18-8: c

Net income – Po P200,000


Unrealized gain, Dec. 31 – DS (30,000)
Net income from own operation – Po 270,000
Net income of So 180,000
Consolidated net income, Dec. 31, 2011 P350,000
Attributable to NCI (P180,000 x 20%) (36,000)
Attributable to parent P314,000

18-9: b

NCI, January 1, 2011 (P1,000,000 x 20%) P 200,000


NCI in dividends paid by subsidiary (P30,000 x 20%) ( 6,000)
NCI in net of subsidiary (P65,00 x 20%) 13,000
NCI, December 31, 2011 P 207,000

18-10: c

Consolidated net income attributable to parent:


Net income – Pink P300,000
Unrealized gain, July 1- Downstream ( 50,000)
Realized gain, Dec. 31 (P50,000 / 10) x 6/12 2,500
Adjusted net income – Pink P252,500
Soda’s adjusted net loss:
Net loss P(40,000)
Unrealized loss, 1/1 – Upstream 15,000
Realized loss, 12/31 (P15,000/5) ( 3,000) (28,000)
Consolidated net income, Dec. 31, 2011 P224,500
Attributable to NCI (P28,000 x 20%) 5,600
Attributable to parent P230,100

109
18-10, Continued:
Non-controlling interest (NCI)
NCI, January 1, 2011 [(P1,240,000/80%) x 20%) P 310,000
NCI share in dividends paid by subsidiary (P30,000 x 20%) ( 6,000)
NCI in adjusted net income (loss) of subsidiary ( 5,600)
NCI, December 31, 2011 P 198,400

18-11: a

Net assets, Dec. 31, 2011


NCI ,Dec. 31, 2011 (based on fair value of net assets) P188,960
Add: NCI share of unrealized profit in ending inventory -Upstream
(P36,000 x 20%) x 20% 1,440
NCI share of unrealized gain on sale of equipment- Upstream
(P60,000 x 20%) – (P12,000 / 5) 9,600
NCI before adjustment P200,000

Net assets – Steve, Dec.31, 2011 (P200,000 / 20%) P1,000,000

Investment in Steve Company stock – Equity method


Investment cost:
Net assets, Dec. 31, 2011 P1,000,000
Less net income – steve
NCI P36,960
NCI share of unrealized profit in ending
Inventory – Upstream 1,440
BCI share of unrealized gain on sale of
Equipment – Upstream 9,600
NCI per book P48,000
Divided by 20% 240,000
Net assets, Jan. 1, 2011 P 760,000
Parent’s proportionate share x 80%
Book value of interest acquired P 608,000
Add: difference 20,000
Price paid P 628,000
Add investment income:
Peter’s share of Steve net income (P240,000 x 80%) P 192,000
Unrealized profit in ending inventory – Downstream
(P24,000 x 20%/120%) x 100% ( 4,000)
Unrealized profit in beginning inventory – Upstream
(P36,000 x 25/125%) x 80% ( 5,760)
Unrealized gain on sale of equipment – Upstream
(P48,000 – 9,600) ( 38,400)
Investment in Steve Company, Dec. 31, 2011 P 771,840

110
18-12: a

Net income from own operations – Pipe P400,000


Adjusted net income - Smoker
Net income P100,000
Unrealized gain, July 1, 2011 – Upstream (50,000)
Realized gain, Dec. 31, 2011 (P50,000/5)x ½ 5,000 55,000
Consolidated net income, Dec. 31, 2011 P455,000

18-13: d
2010 2011
Net income from operations – Parent P100,000 P120,000
Adjusted net income of Sub:
Net income P 60,000 P 75,000
Unrealized gain – Upstream ( 9,000) -
Realized gain: 2010 (P9,000/3) x ¼ 750
2011 (P9,000/3) - 3,000
Adjusted net income P 51,750 P 78,000
Consolidated net income P151,750 P198,000
Attributable to NCI (10,350) (15,600)
Attributable to parent P141,400 P182,400

18-14: d

Investment in Sili Company stock – Equity method


Price paid P500,000
Investment income net of dividends – 2007 to 2010:
Increase in earnings (P500,000 – P200,000) x 75% 225,000
Investment income, Dec. 31, 2010:
Share of Sili’s net income (P60,000 x 75%) 45,000
Unrealized gain on sale of land – Downstream (15,000)
Unrealized loss on sale of building – Downstream 10,000
Realized loss on sale of building (P10,000 / 5) x 75% ( 1,500) 38,500
Investment income, Dec. 31, 2011:
Share of Sili’s net income (P70,000 x 75%) 52,500
Realized loss (P10,000 / 5) (2,000) 50,500
Dividends received:
2010: (P10,000 x 75%) 7,500
2011: (P20,000 x 75%) 15,000 (22,500)
Investment in Sili Company stock, Dec. 31, 2011 P791,500

111
18-15: a

Investment in Saw Company stock, Dec. 31, 2011


Price paid P550,000
Investment income – 2005 to 2009:
Increase in earnings (P500,000 – P300,000) x 90% 180,000
Investment income – 2010 (see above) 101,250
Investment income – 2011:
Power’s share of Saw’s net income (P120,000 x 90%) P108,000
Realized loss on sale of warehouse (P20,000/2) x 90% (9,000) 99,000
Dividends received:
2010: ( P20,000 x 90%) P 18,000
2011: ( P30,000 x 90%) 27,000 (45,000)
Investment in Saw Company stock account balance 12/31/11 P885,250

PROBLEMS

Problem 18-1

Computation of the missing amounts in the working paper eliminations for P Corporation and S
Company:
(1) P640 (P3,200 x 20%)
(2) P2,560 (P3,200 x 80%)
(3) P1,600 (P800 x 2)
(4) P320 (P1,600 x 20%)
(5) P1,280 (P1,600 x 80%)
(6) P3,200 (P800 x 4)

Problem 18-2

a. Consolidated Net Income


Net income from own operations – P Company P200,000
Unrealized gain on sale of equipment, Dec. 31 – Downstream (30,000)
Adjusted net income – P Co, P170,000
S Company net income 180,000
Consolidated net income P350,000

b. NCI in net income of subsidiary (P180,000 x 20%) P 36,000

c. Non-controlling interest (NCI):


NCI, January 1, 2011 [(P720,000/80%) x 20%] P180,000
NCI in dividends paid by subsidiary (P60,000 x 20%) (12,000)
NCI in net income of subsidiary (P180,000 x 20%) 36,000
NCI, December 31, 2011 P204,000

112
Problem 18-3

Pony Corporation and Subsidiary


Consolidated Income Statement
Year Ended December 31, 2011

Sales (P500,000 + P300,000) P800,000


Gain on sale of machinery (schedule 1) 20,000
Total revenue 820,000
Cost of sales P200,000 + P130,000) 330,000
Gross profit 490,000
Expenses:
Depreciation (P50,000 +P30,000 – P5,000) P 75,000
Other expenses (P80,000 + P140,000) 220,000 295,000
Consolidated net income 785,000
Attributable to NCI [(P190,000 + P5,000) +10,000) x 25%] (28,750)
Attributable to parent P266,250

Schedule 1:
Selling price – Dec. 28, 2011 P36,000
Book value (P65,000 ÷ 5) x3 26,000
Gain on sale 10,000
Unrealized gain (P25,000 – P15,000) 10,000
Total gain P20,000

Problem 18-4

a. Consolidated Net Income


Net income from own operations – P Company P300,000
Adjusted net income of S Company:
Net income – S P150,000
Unrealized gain, 4/1/11 - Upstream ( 30,000)
Realized gain, 12/31/11 (P30,000/5) x 9/12 4,500 124,500
Consolidated net income 424,500
Attributable to NCI (P124,500 x 20%) (24,900)
Attributable to parent P399,600

b. Non-controlling interest (NCI)


NCI, January 1, 2011 [(P800,000/80% x 20%] P200,000
NCI share in dividends paid by subsidiary (P50,000 x 20%) ( 10,000)
NCI in adjusted net income of subsidiary 24,900
NCI, December 31, 2011 P214,900

113
Problem 18-5
Texas Company and Subsidiary
Consolidated Income Statement
Year Ended December 31, 2011

Sales P1,500,000
Cost of goods sold 650,000
Gross profit 850,000
Expenses (P200,000 + P100,000 – P8,000 ) 292,000
Consolidated net income P 558,000
Attributable to NCI (P150,000 x 25%) 37,500
Attributable to parent P 520,500

Adjustment for expenses (depreciation) = P40,000 / 5 years.

Problem 18-6
a. Working Paper Elimination Entries – Dec. 31, 2011

(1) Dividend income 4,000


NCI 1,000
Dividends declared – Jupiter 5,000
To eliminate intercompany dividends

(2) Common stock – Jupiter 100,000


Retained earnings – Jupiter 50,000
Investment in Jupiter Company 120,000
NCI 30,000
To eliminate equity accounts of Jupiter as of the
date of acquisition

(3) Goodwill 40,000


Investment in Jupiter Company 40,000
To allocate excess to goodwill

(4) Retained earnings, Jan. 1 - Vincent 8,000


NCI 2,000
Land 10,000
To eliminate unrealized gain on sale of land – Upstream.

(5) Gain on sale of equipment 20,000


Building and equipment 5,000
Accumulated depreciation 25,000
To eliminate gain on sale of equipment

(6) Accumulated depreciation 2,000


Depreciation 2,000
To adjust excess depreciation

(7) Accounts payable 7,000


Accounts receivable 7,000
To eliminate intercompany payables and receivables.

(8) NCI in net income of subsidiary 6,000


NCI 6,000
(P40,000 – 10,000) x 20%

114
Problem 18-6, Continued:
b. Vincent Company and Subsidiary
Consolidation Working Paper
December 31, 2011

Vincent Jupiter Adjustments & Eliminations Consoli-


Company Company Debit Credit dated
Income Statement
Sales 240,000 120,000 360,000
Gain on sale of equipment 20,000 (5) 20,000 -
Dividend income 4,000 (1) 4,000 -
Total revenues 264,000 120,000 360,000
Cost of goods sold 140,000 60,000 200,000
Depreciation 25,000 15,000 (6) 2,000 38,000
Other expenses 15,000 5,000 20,000
Total cost and expenses 180,000 80,000 258,000
Net/consolidated income 84,000 40,000 102,000
NCI in net income of subsidiary (8) 6,000 (6,000)
Net income carried forward 84,000 40,000 96,000

Retained Earnings Statement


Retained earnings, Jan.1 294,000 105,000 (2) 50,000 341,000
(4) 8,000
Net income from above 84,000 40,000 96,000
Total 378,000 145,000 437,000
Dividends declared 30,000 5,000 (1) 5,000 30,000
Retained earnings, Dec. 31
Carried forward 348,000 140,000 407,000

Statement of FP
Cash and receivables 113,000 35,000 (7) 7,000 141,000
Inventory 260,000 90,000 350,000
Land 80,000 80,000 (4) 10,000 150,000
Buildings and equipment 500,000 150,000 (5) 5,000 655,000
Investment in Jupiter Company 160,000 (2)120,000 -
(3) 40,000
Goodwill (3) 40,000 40,000
Total 1,113,000 355,000 1,336,000

Accumulated depreciation 205,000 45,000 (6) 2,000 (5) 25,000 273,000


Accounts payable 60,000 20,000 (7) 7,000 73,000
Bonds payable 200,000 50,000 250,000
Common stock 300,000 100,000 (2)100,000 300,000
Retained earnings from above 348,000 140,000 407,000
NCI (1) 1,000 (2) 30,000 33,000
(4) 2,000 (8) 6,000

Total 1,113,000 355,000 245,000 245,000 1,336,000

115
.Problem 18-6, Continued:
c. Consolidated Financial Statements

Vincent Company and Subsidiary


Consolidated Statement of Financial Positionj
December 31, 2011

Assets
Cash and receivables P 141,000
Inventory 350,000
Land 150,000
Buildings and equipment P655,000
Less: Accumulated depreciation 273,000 382,000
Goodwill 40,000
Total assets P1,063,000

Liabilities and Stockholders’ equity


Liabilities
Accounts payable P 73,000
Bonds payable 250,000
Total liabilities P 323,000
Stockholders’ Equity
Common stock P300,000
Retained earnings 407,000
NCI 33,000 740,000
Total liabilities and stockholders’ equity P1,063,000

Vincent Company and Subsidiary


Consolidated Income Statement
Year Ended December 31, 2011

Sales P 360,000
Cost of goods sold 200,000
Gross profit 160,000
Expenses: Depreciation P 38,000
Other expenses 20,000 58,000
Consolidated net income 102,000
Attributable to NCI 6,000
Attributable to parent P 96,000

Vincent Company and Subsidiary


Consolidated Retained Earnings
Year Ended December 31, 2011

Retained earnings, Jan. 1 – Vincent P 294,000


Retained earnings, Jan. 1 – Jupiter 47,000
Total 341,000
Consolidated net income attributable to parent 96,000
Dividends declared – Vincent ( 30,000)
Consolidated retained earnings P 407,000

116
Problem 18-7

P Company and Subsidiary


Consolidated Working Paper
Year Ended December 31, 2011

Eliminations/ Adjustments Conso-


P Company S Company Debit Credit lidated
Income Statement
Sales 600,000 315,000 (7) 40,000 875,000
Dividend income 16,000 (3) 16,000 -
Total revenue 616,000 315,000 875,000
Cost of goods sold 350,000 150,000 (9) 5,000 (7) 40,000
(8) 10,000 455,000
Operating expenses 150,000 60,000 (6) 6,250 (11) 3,000 213,250
Total costs and expenses 500,000 210,000 668,250
Net income 116,000 105,000 206,750

NCI in net income of S (2) 20,750 (20,750)

Net income carried forward 116,000 105,000 186,000

Retained Earnings Statement


Retained earnings, January 1 280,000 150,000 (1) 6,250 285,000
(4)100,000
(6) 18,750
(8) 8,000
(10) 12,000
Net income from above 116,000 105,000 186,000
Dividends declared (60,000) (20,000) (3) 20,000 (60,000)
RE, 12/31 carried forward 336,000 235,000 411,000

Statement of FP
Inventory 130,000 50,000 (5) 12,500 (6) 12,500 175,000
(9) 5,000
Other current assets 241,000 235,000 476,000
Investment in S Company 200,000 (4)160,000
(5) 40,000
Goodwill (5) 12,500 12,500
Other long-term investments 20,000 20,000
Land 140,000 80,000 220,000
Buildings and equipment 375,000 200,000 (5) 25,000 (10) 15,000 585,000
Intangible assets 20,000 20,000
Totals 1,106,000 585,000 1,508,500

Accumulated depreciation 120,000 30,000 (10) 3,000 (6) 12,500 156,500


(11) 3,000
Current liabilities 150,000 70,000 220,000
Non-current liabilities 200,000 150,000 350,000
Common stock 200,000 50,000 (4) 50,000 200,000
APIC 100,000 50,000 (4) 50,000 100,000
RE, 12/31 from above 336,000 235,000 411,000

NCI (3) 4,000 (1) 6,250 71,000


(8) 2,000 (2) 20,750
(4) 40,000
(5) 10,000
Totals 1,106,000 585,000 395,000 395,000 1,508,000

117
Problem 18-7, continued:
Determination and Allocation of Excess Schedule

Company Parent NCI


Implied Price Value
Fair Value (80%) (20%)
Price paid for investment P250,000 P200,000 P 50,000*
Less book value of interest acquired:
Total equity 200,000 P200,000 P200,000
Interest acquired 80% 20%
Book value of interest acquired P160,000 P 40,000
Excess P 50,000 P 40,000 P 10,000
Allocations
Inventory (12,500)
Equipment (25,000)
Total (37,500)

Goodwill P12,500

* (P200,000/80%) x 20% = P50,000

Amortization
Inventory P12,500
Equipment (P25,000/4) 6,250

Explanations of Eliminations and Adjustments:

(1) To recognize NCI share in subsidiary’s adjusted prior year’s undistributed earnings
(P50,000 – P18,750) 20% = P6,250.
(2) To recognized NCI in net of subsidiary for the current year
(P105,000 + P10,000 – P5,000 – 6,250) x 20% = P20,750
(3) To eliminated intercompany dividends paid the subsidiary.
(4) To eliminate equity of the subsidiary at date of acquisition.
(5) To allocate excess.
(6) To amortize allocated excess.
(7) To eliminate intercompany sales.
(8) To eliminate beginning inventory profit.
(9) To eliminate ending inventory profit.
(10) To eliminate fixed asset gain at beginning of year.
(11) To eliminate realized gain on fixed assets.

118
Problem 18-8

Supporting computations
(1) Determination and allocation of excess schedule;

Total Price paid (60%) NCI (40%)


Company fair value P620,00 P372,000 P248,000
Less book value of interest acquiree:
Small’s equity 350,000 350,000 350,000
Interest acquired 60% 40%
Book value of interest acquired 210,000 140,000
Excess 270,000 162,000 108,000
Allocated to patents ( 120,000)
Goodwill P150,000

Amortization of patents (P120,000 / 12) P 10,000

(2) Unrealized gain on intercompany sale of building – Upstream, Jan. 1, 2011:


Unrealized gain at date of sale (P80,000 – P30,000) P 50,000
Realized gain (P50,000 / 5) x 2 years (20,000)
Unrealized gain as of Jan. 1, 2011 P 30,000

(3) Realized profit from intercompany sale of inventory – Downstream, 1/1/11:


Remaining inventory as of Dec. 31, 2010 P 50,000
Gross profit rate on sales – 2010 (P30,000 / P150,000) x 20%
Realized profit as of Jan. 1, 2011 P 10,000

(4) Unrealized profit from intercompany sale of inventory – Downstream, 12/31/11


Remaining inventory as of Dec. 31, 2011 P 40,000
Gross profit rate on sales – 2011 (P48,000 / P160,000) x 30%
Unrealized profit as of Dec. 31, 2011 P 12,000

Consolidated balances – 2011

a. Cost of goods Sold


Cost of goods sold – Apex P 460,000
Cost of goods sold – Small 205,000
Intercompany sale of inventory – 2011 (160,000)
Realized profit on beginning inventory ( 10,000)
Unrealized profit on ending inventory 12,000)
Consolidated P 507,000

b. Operating Expenses
Operating expenses – Apex P 170,000
Operating expenses – Small 70,000
Amortization (No. 1 above) 10,000
Excess depreciation (P50,000 / 5 years) (10,000)
Consolidated P 240,000

119
c. Consolidated Net Income Attributable to Parent
Sales (after elimination of intercompany sales) P 840,000
Cost of goods sold (a) (507,000)
Operating expenses (b) (240,000)
NCI in net income of subsidiary:
Net income – Small P25,000
Realized gain on sale of building – Upstream 10,000
Adjusted net income P35,000
NCI x 40% ( 14,000)
Attributable to parent P 79,000

d. Consolidated Retained Earnings, Jan. 1, 2011


Retained earnings, Jan. 1, 2010 – Apes P 690,000
Amortization of patents – 2005 to 2010(P10,000 x 6) (60,000)
Unrealized profit on inventory, 2010– Downstream (10,000)
Unrealized gain on sale of building, 1/1/11 - Upstream (P30,000 x 60%) (18,000)
Consolidated retained earnings, Jan. 1, 2011 P 602,000

e. Consolidated Inventory
Inventory – Apex P 233,000
Inventory – Small 229,000
Unrealized profit in inventory – Dec. 31, 2011 ( 12,000)
Consolidated inventory P 450,000

f. Consolidated Building
Buildings – Apex P 308,000
Buildings – Small 202,000
Unrealized gain, Jan. 1, 2011 (50,000)
Realized gain, 2009– 2009 (P10,000 x 3 ) 30,000
Consolidated buildings P 490,000

g. Consolidated Patents
Patents – Small P 20,000
Allocation 120,000
Amortization, 2009 – 2011 (P10,000 x 7) ( 70,000)
Consolidated patents (net) P 70,000

h. Consolidated Common Stock = P300,000 (Apex common stock)

120
Problem 18-9

P Company and Subsidiary


Consolidated Worksheet
Year Ended December 31, 2011

Eliminations Adjustments Conso-


P Company S Company Debit Credit lidated
Income Statement
Sales 1,900,000 1,500,000 (7)180,000 3,220,000
Dividend income 40,000 (1) 40,000 -
Total revenue 1,940,000 1,500,000 3,220,000
Cost of goods sold 1,180,000 870,000 (8) 18,000 (7)180,000 1,888,000
Operating expenses 550,000 440,000 (4) 9,000 (6) 4,000 995,000
Total costs and expenses 1,730,000 1,310,000 2,883,000
Net income carried forward 210,000 190,000 337,000

Retained Earnings Statement


Retained earnings, Jan. 1 250,000 206,000 (2)156,000 258,000
(4) 18,000
(5) 24,000
Net income from above 210,000 190,000 337,000
Dividends paid (40,000) (1) 40,000 -
RE, 12/31, carried forward 460,000 356,000 595,000

Statement of FP
Cash 285,000 150,000 435,000
Accounts receivables (net) 430,000 350,000 (9) 75,000 705,000
Inventories 530,000 410,000 (8) 18,000 922,000
Land, buildings, and equipment 660,000 680,000 (3) 54,000 (5) 30,000 1,364,000
Investment in S Company 750,000 (2)636,000
(3)114,000
Goodwill (3) 60,000 60,000
Totals 2,655,000 1,590,000 3,486,000

Accumulated depreciation 185,000 210,000 (5) 6,000 (4) 27,000 412,000


(6) 4,000
Accounts payable 670,000 544,000 (9) 75,000 1,139,000
Common stock, P10 par 1,200,000 400,000 (2)400,000 1,200,000
APIC 140,000 80,000 (2) 80,000 140,000
Retained earnings from above 460,000 356,000 595,000

Totals 2,655,000 1,590,000 1.124,000 1,124,000 3,486,000

Determination and Allocation of Excess Schedule

Price paid by the parent P750,000


Less book value of interest acquired (100%)
Common stock – S Company P400,000
Additional paid in capital – S Company 80,000
Retained earnings – S Compay 156,000 636,000
Excess P114,000
Allocated to machinery (54,000)
Goodwill P 60,000

121
Problem 18 – 10

Pluto Corporation and Subsidiary Star Corporation


Comparative Consolidated Income Statement
Years Ended December 31, 2010 and 2011

. December 31 .
. 2011 2010 .
Sales P800,000 P660,000
Cost of goods sold 442,000 368,000 .
Gross profit 358,000 292,000
Operation expenses 178,000 138,000 .
Consolidated net income 180,000 154,000
NCI in net income of subsidiary 10,000 10,000 .
Attributable to equity holders of Pluto P170,000 P144,000 .

Supporting computations:
. .
. 2011 2010 .
Consolidated sales:
Combined sales P850,000 P700,000
Less: intercompany sales (50,000) (40,000) .
Consolidated sales P800,000 P660,000 .

Consolidated cost of goods sold:


Combined costs of good sold P490,000 P400,000
Intercompany sales (50,000) (40,000)
Unrealized profit in ending inventory 10,000 8,000
Unrealized profit in beginning inventory (8,000) .
Consolidated cost of goods sold P442,000 P368,000 .

Consolidated operating expenses


Combined operating expenses P180,000 P140,000
Realized gain on sale of equipment (P10,000/.2) (2,000) (2,000) .
Consolidated operating expenses P178,000 P138,000 .

NCI in net income of subsidiary


Star Company’s reported net income P65,000 P50,000
Gain on upstream sale of land (5,000)
Unrealized gain in upstream, inventory sales (10,000) .
Realized net income P50,000 P50,000
NCI 20% 20% .
NCI in net income of subsidiary P10,000 P10,000 .

122
123
CHAPTER 19

MULTIPLE CHOICES - COMPUTATIONAL

19-1: d.

Direct exchange rate:


December 1 1 ÷ 2.22 yen = P 0.45
December 31 1 ÷ 2.70 yen = 0.37
Decrease in forex rate P 0.08

Forex gain (200,000 yen x P0.08) P 16,000

19-2: c.

Forex rate, December 1 P 0.45


Forex rate, December 31 0.47
Increase in forex rate P 0.02

Forex gain (1,500,000 yen x P0.02) P 30,000

19-3: d.

September 30:
Forex rate, September 1 P 5.61
Forex rate, September 30 5.59
Decrease in forex rate P 0.02

Forex gain (200,000 hkg.$ x P0.02) P 4,000

December 31:
Forex rate, October 1 P 5.59
Forex rate, December 30 5.62
Increase in forex rate P 0.03

Forex loss (200,000 hkg.$ x P0.03) P (6,000)

19-4: c.

Forex loss on importation of merchandise:


Peso equivalent, January 10, 2011 P 600,000
Peso equivalent, April 20, 2011 608,000
Forex loss (increase) P (8,000)

Forex loss on notes payable:


Peso equivalent, September 1, 2011 P 3,000,000
Peso equivalent, December 31, 2011 3,200,000
Forex loss on principal P (200,000)
Add: Forex loss on interest
Based on P 3,2000,000 P 120,000
Based on P 300,000,000 (P3,000,000x10%x4/12) 100,000 20,000
Forex loss P (220,000)

Total forex loss (P 8,000 + P220,000) P (228,000)

123
19-5: a.

Direct forex rate – Transaction date (P 1 ÷ $0.018) P 55.5555


Direct forex rate – Balance sheet date (P 1 ÷ $0.017) 58.8235
Direct forex rate – Settlement date (P 1 ÷ $0.020) 50.0000

Forex gain (loss), 2010


Transaction date ($10,000 x P55.5555) P 555,555
Balance sheet ($10,000 x P 58.8235) 588,235
Forex loss (increase) P ( 32,680)

Forex gain (loss), 2011


Balance sheet date ($10,000 x P58.8235) P 588,235
Settlement data ($10,000 x P 50.00) 500,000
Forex gain (decrease) P 88,235

19-6: b.

Adjusted value of accounts receivable, 6/30 P 315,000


Peso equivalent, 7/27 300,000
Forex loss P (15,000)

19-7: a.

2010
Forex rate, 11/5/10 P 0.4295
Forex rate, 12/31/10 0.4245
Decrease in forex rate P 0.0050
Payable in foreign currency 50,000
Forex gain P 250

2011
Forex rate, 12/31/10 P 0.4245
Forex rate, 1/15/11 0.4345
Decrease in forex rate P 0.0100
Payable in foreign currency 50,000
Forex loss P (500)

19-8: a. (1000,000 FC x P 0.85)

19-9: c. (50,000 FC x P 0.6498)

19-10: b

Forward rate, 3/31/11 P 0.25


Selling spot rate, 4/30/11 0.22
Decrease P 0.03
Forward contract receivable 100,000 FC
Forex loss P 3,000

124
19-11: d. forex gain (loss) on purchase commitments is based on the changes in the forward rates.

Forward rates – December 31, 2010 P .0055


90-day forward rate .0055

On December 31, 2010, no changes in forward rates occurred, so no forex gains (losses) are to be
recognized on December 31, 2010 under both transactions.

19-12: b.

Forward contract receivable (P100,000 Baht x P1.650) P 165,000


Spot rate (100,000 Baht x P1.600) 160,000
Forex loss P (5,000)

19-13: d.

Import transaction – Based on spot rates:


12/31/10: Forex loss [1,000,000 Francs x (P6.01 – P6.16)] P (150,000)
Forward Contract – Based on forward rates:
12/31/10: Forex gain [1,000,000 Francs x (P6.06 – P6.07)] P 10,000
Net forex loss P (140,000)

19-14: b.

12/31/10: Forex gain [$5,000 x (P56.50 – P56.60)] P 500


3/31/10 : Forex loss:
Forward contract receivable ($5,000 x P56.60) P 283,000
Settlement at spot rate ($5,000 x P56.32) 281,600 (1,400)
Net forex loss P (900)

19-15: a.

Increase in forward rates:


Forward contract receivable, 11/1/10 (10,000 fc x P.78) P 7,800
Forward contract receivable, 12/31/10 (10,000 fc x P82) 8,200
Forex loss P (400)

19-16: a. The forex gain or loss (changes in forward rates) is offset by gain or loss in firm
commitment to purchase machinery. The hedge was perfect.

19-17: 1. a
June 30: 400,000 FC x (P1.381 – P1.370) P(4,400)
July 31: 400,000 FC x (P1.385 – P1.381) (1,600)
Net forex loss P (6,000)

2. c
June 30: (P2,600 – P1,400) P1,200
July 31: [(P1.385 – P1.375) x 400,000FC] – P2,600 1,400
Net gain on option P2,600
Note that the option has expired and, therefore, there is no time value.

125
3. a
Down payment (50,000 FC x P1.350) P 67,500
Balance due (400,000 FC x P1.370) 548,000
Cost of machinery P615,500

19-18: 1. a
FCA FCB
Number of FC in commitment:
P549,600/P1,200 458,000
P297,975/P0.685 435,000
Change in spot rate from commitment date to
Transaction date:
P1.200 – P1.160 P 0.04
P0.685 – P0.692 P 0.007
Gain (loss) on commitment:
458,000 FCA x P0.04 P(18,320)
435,000 FCB x P0.007 P 3,030

2. a
FCA FCB
Receivables at spot rate at transaction date:
458,000 FCA x P1.160 P531,280
435,000 FCB x P0.692 P301,020
Receivables at spot rate at settlement date:
458,000 FCA x P1.170 535,860
435,000 FCB x P0.720 313,200
Exchange gain (loss): 4,580 12,180

19-19: 1. a
2. d
Computations: Forward
Contract Option
Prior to transaction date:
Gain (loss) on commitment [100,000 x (P1.250 – P1.320)] P (7,000) P (7,000)
Gain (loss) on hedging instrument:
Forward contract [100,000 x (P1.320 – P1.250)] 7,000
Option [100,000 x (P1.320 spot 0 P1.250 strike)] 7,000
Gain (loss) excluded from hedge effectiveness:
Forward contract [100,000 x P1.270 – P1.250)] (2,000)
Option (premium paid is all time value) - (1,200)
Effect on earnings prior to transaction date P (2,000) P (2,100)

19-20: a
12/01/11: A$70,000/P42,000 = 1.667 A$ to P1.00
12/31/11: A$70,000/P41,000 = 1.679 A$ to P1.00

19-21: a (A$70,000 x P.57)

19-22: a, The balance will not change, because it is denominated in Philippine peso.

19-23: a
P82,000/KRW 400,000 = P.205
The P82,000 is the amount of the peso payable to bank. This amount is computed using the
forward rate.

126
Problems
Problem 19-1

Foreign Foreign
Currency Currency
Accounts Accounts Transactions Transactions
Receivable Payable Exchange Loss Exchange Gain

Case 1 NA P 160,000 (a) NA P 20,000 (b)

Case 2 P 38,000 © NA NA P 2,000 (d)

Case 3 NA P 13,500 (e) P 1,500 (f) NA

Case 4 P 6,250 (g) NA P 1,250 (h) NA

(a) $40,000 x P4.00


(b) $40,000 x (P4.00 – P4.50)
(c) $20,000 x P1.90
(d) $20,000 x (P1.90 – P1.80)
(e) $30,000 x P.45
(f) $30,000 x (P.45 – P.40)
(g) $2,500,000 x P.0025
(h) $2,500,000 x (P.0025 – P.003)

Problem 19-2

a. May 1 Inventory (or purchases) 800,000


Accounts payable 800,000
Foreign purchases denominated in
Philippine pesos.

June 20 Accounts payable 800,000


Cash 800,000
Settlement.

July 1 Accounts receivable 500,000


Sales 500,000
Foreign sales denominated in
Philippine pesos.

August 10 Cash 500,000


Accounts receivable 500,000
Collections.

127
Problem 19-2, continued:

b. May 1 Inventory (or purchases) 800,000


Accounts payable 800,000
Foreign purchases denominated in yen:
P800,000 / P.40 = 2,000,000 yen

June 20 Foreign currency transaction loss 100,000


Accounts payable 100,000
P900,000 = 2,000,000 yen x P.45
800,000 = 2,000,000 yen x P.40
P100,000

Accounts payable 900,000


Cash or foreign currency 900,000
Settlement denominated in yen.

July 1 Accounts receivable 500,000


Sales 500,000
Foreign sale denominated in Hongkong $
P500,000 / P5.20 = 96,154 Hkg $

August 10 Accounts receivable 1,924


Foreign currency transaction gain 1,924
P501,924 = 96,154 Hkg. $ x P 5.22
500,000 = 96,154 Hkg. $ x P 5.20
P 1,924

Cash or foreign currency 501,924


Accounts receivable 501,924
Collections

Problem 19-3

a. No net exposure between November 1 and March 1. Michael, Inc. has hedged its foreign currency
purchase commitment with a forward contract to receive an equal number of foreign currency
units.

b. November 1: Forward contract receivable 3,076,800


Forward contract payable 3,076,800
To record forward contract at forward rate:
240,000 Ringgit x P12.82

December 31: Forex loss 4,800


Forward contract receivable 4,800
To record forex loss for the decrease in
forward rate, P240,000 x P.02

128
Problem 19-3, continued:
December 31: Firm commitment for merchandise 4,800
Forex gain 4,800
To record increase in fair value of the
Purchase commitment, and resultant
gain or the decrease in the forward rate.

March 1: Forward contract payable 3,076,800


Cash 3,076,800
To record settlement of forward contract.

Cash (240,000 x P12.86) 3,086,400


Forex loss (240,000 x P.02) 4,800
Forward contract receivable 3,091,200
To record receipt of 240,000 Ringgit when
the spot rate is P12.86.

Firm commitment for merchandise 4,800


Forex gain 4,800
To record change in value of the firm
commitment.

Purchases (240,000 x P12.82) 3,076,800


Firm commitment for merchandise 9,600
Cash 3,086,400
To record purchases of merchandise.

Problem 19-4

June 1: Purchases 460,000


Accounts payable 460,000
To record purchases (¥ 1,000,000 x P.46).

Forward contract receivable (fc) 480,000


Forward contract payable 480,000
To record purchase of ¥ 1,000,000 for delivery
in 60 days at forward rate of P.48.

June 30: Forex loss 20,000


Accounts payable 20,000
To record forex loss for the increase in spot
rate, ¥ 1,000,000 x (P.46 – P.48)

Forward contract receivable 20,000


Forex gain 20,000
To record forex gain for the increase
in forward rate, ¥ 1,000,000 x (P.48 – P.50).

129
Problem 19-4, continued:
August 1: Accounts payable 480,000
Forex loss (¥ 1,000,000 x P.03) 30,000
Cash (¥ 1,000,000 x P.51) 510,000
To record settlement.

Cash (¥ 1,000,000 x P.51) 510,000


Forex gain 10,000
Forward contract receivable 500,000
To record receipt of ¥ 1,000,000 at spot rate

Forward contract payable 480,000


Cash 480,000
To record settlement of forward contract.

Problem 19-5

December 1: Accounts receivable 1,280,000


Sales 1,280,000
To record sale (100,000 Rial x P12.80).

Forward contract receivable 1,240,000


Forward contract payable (fc) 1,240,000
To record forward contract to sell 100,000 Rial
at a 90-day forward rate of P12.40.

December 31: Forex loss 10,000


Accounts receivable 10,000
To adjust receivable for the decrease in spot rate
and record forex loss, 100,000 Rial x (P12.80 – P 12.70).

Forex loss 20,000


Forward contract payable (FC) 20,000
To record forex gain for the increase in forward rate,
100,000 Rial x (P12.40 – P12.60).

March 1: Cash 1,290,000


Forex gain(100,000 Rial x P.20) 20,000
Accounts receivable 1,270,000
To record collection of accounts receivable at spot rate.

Forward contract payable (FC) 1,260,000


Forex loss 30,000
Cash (100,000 Rial x P12.60) 1,290,000
To record delivery of 100,000 Rial.

Cash 1,240,000
Forward contract receivable 1,240,000
To record collection for forward contract.

130
Problem 19-6

October 1: Forward contract receivable 17,400


Forward contract payable (fc) 17,400
(15,000 Baht x P1.16)

December 31: Forex loss 150


Forward contract payable (fc) 150
15,000 Baht x (P1.16 – P1.17).

Firm commitment for materials 150


Forex gain 150
To record increase in fair value of sales
commitment.

April 1: Cash 17,400


Forward contract receivable 17,400
To record collection of forward contract.

Forward contract payable 17,550


Forex gain 150
Cash /fc (15,000 Baht x P1.16) 17,400
To record delivery of 15,000 Baht at forward rate
of P1.16.

Forex loss 150


Firm commitment for materials 150

Cash/fc (15,000 Baht x P1.18) 17,700


Sales 17,700
To record sales.

Problem 19-7

Contract 1:

October 1: Forward contract receivable (fc) 160,000


Forward contract payable 160,000
To record forward contract to buy ¥400,000 at P40.

December 31: Forward contract receivable (fc) 4,000


Forex gain 4,000
To record forex gain for the increase in forward
rate of P.01.

April 1: Cash (¥ 400,000 x P.43) 172,000


Forward contract receivable (fc) 164,000
Forex gain 2,000
To record receipt of ¥400,000 at spot rate of
P.43.

Forward contract payable 160,000


Cash 160,000
To record payment of forward contract.

131
Problem 19-7, continued:
Contract 2:

December 1: Forward contract receivable 9,200


Forward contract payable (fc) 9,200
To record forward contract to sell 2 million Rupiah
at P.0046.

December 31: Forward contract payable 200


Forex gain 200
To record forex gain for the decrease in forward
Rate by P.0001.

March 1: Cash 9,200


Forward contract receivable 9,200
To record settlement of forward contract.

Forward contract payable (fc) 9,000


Forex loss 800
Cash 9,800
To record payment of 2 million Rupiah at spot
rate of P.0049.

Problem 19-8

June 1: Inventory – Used Equipment 158,400


Accounts payable 158,400
To record purchase of the equipment
(220,000 FC x P0.720)

Investment in call option 1,000


Cash 1,000
To record purchase of call option.

Accounts receivable 216,000


Sales 216,000
To record sale of equipment
(300,000 FC x P0.720)

Forward contract receivable – P 218,700


Forward contract payable – FC 218,700
To record forward contract to sell FC
(300,000 FC s P0.729)

June 15: Memo entry to buy equipment

Forward contract receivable – FC 292,400


Forward contract payable – P 292,400
To record forward contract to buy FC
(400,000 FC x P0.731)

132
Problem 19-8, continued:
June 20: Inventory- Used Equipment 21,960
Cash 21,960
To record reconditioning the equipment
(30,000 FC x P0.732)

Accounts receivable 226,920


Sales 226,920
To record sales of used equipment
(310,000 FC x P0.732)

Cost of goods sold 180,360


Inventory – used equipment 180,360
To record costs of used equipment
sold (P158,400 + P21,960)

30: Foreign currency (cash) 220,500


Accounts receivable 216,000
Forex gain 4,500
To settle accounts receivable
(300,000 FC x P0.735)

Loss on call option 1,800


Forward contract payable – FC 1,800
To record change in value of the June 1
Forward Contract
[300,000 FC x (P0.735 – P0.729)

Forward contract payable – FC 220,500


Cash 218,700
Foreign currency (cash) 220,500
Forward contract receivable – P 218,700
To record settlement of the June 1 contract

Investment in call option 2,200


Gain on call option 2,200
To record change in value of option
(P3,200 – P1,000)

Loss on firm commitment 2,388


Firm commitment 2,388
To record loss on the commitment
(see Schedule 1)

Forward contract receivable – FC 2,388


Forex gain 2,388
To record change in value of the June 15
Forward Contract (see Schedule 1)

133
Problem 19-8, continued:
Schedule 1
June 15 June 30
Number of FC 400,000 400,000
Forward rate remaining time – 1 FC P0.731 P0.737

Fair value of original contract:


Original forward rate P292,400
Current forward rate 294,800
Gain (loss) in forward rate P 2,400

Present value of the change:


n = 1, I = ½% (.995 x P2,400) P 2,388

Problem 19-9

1. Cash (fc) 168,000


Accounts receivable (fc) 167,000
Forex gain 1,000
To record collection of 100,000 Baht from Queens Company.

Forward contract payable (fc) 167,000


Forex loss 1,000
Cash (fc) 168,000
To record delivery of 100,000 Baht in settlement of the
forward contract denominated in Baht.

Cash 164,000
Forward contract receivable 164,000
To record receipt of Phil. Pesos in settlement of the
forward contract receivable.

2. Forward contract payable 76,000


Cash 76,000
To record payment of forward contract payable.

Cash (fc) 75,000


Forex loss 500
Forward contract receivable (fc) 75,500
To record collection of forward contract receivable:
(10,000,000 Rupiah x P.00750)

Accounts payable (fc) 75,500


Cash (fc) 75,000
Forex gain 500
To record payment of accounts payable to Indon Co.
(1,000,000 Rupiah x P.00750)

134
Problem 19-10

1. Schedule of forward contract items at December 31, 2011:

Current assets:
Forward contract receivable (Siam hedge: in Phil. pesos) P 168,000
Forward contract receivable (Indon hedge: 10,000,000 x P.0077) 77,000
Forward contract receivable (Speculation in Yen: 200,000 x P.670) 134,000
Change in value of firm commitment 1,000

Current liabilities:
Accounts payable (Indon account: 10,000,000 x P.0077) P 77,000
Forward contract payable (Siam hedge: 100,000 Baht x P1.690) 169,000
Forward contract payable (Speculation in Yen: payable in Phil. pesos) 130,000

2. Forex gain or loss for 2011:

Indon: P2,000 loss on account payable offset by P2,000 gain on


Forward contract receivable P -

Siam: Forex loss is offset by the change in the value of firm


commitment -

Speculation: The speculation is accounted for at the forward rate


throughout the life of the contract. Therefore, the forward
contract receivable is adjusted to P 134,000 (the rate for
60-day futures at December 31 and the P4,000 gain is
recognized). 4,000
Forex gain for 2004 in the income statement P 4,000

Problem 19-11

a. Entry to record the purchase of the call options on November 30, 2010

Call Options 20,000


Cash 20,000
Purchase call options for 10,000 barrels
of oil at a premium of P2 per barrel for
March 1, 2008. The options are at the money
of P30 per barrel; therefore, the entire
P20,000 is time value

b. Adjusting entry on December 31, 2010:

Loss on call option 14,000


Call options 14,000
Record the decrease in the time value
of the options to current earnings.

Call options 10,000


Other comprehensive income 10,000
Record the increase in the intrinsic value
of the options to other comprehensive income.

135
Problem 19-11, continued:
c. Entries to record March 1, 2011, expiration of options, the sales of option, and the purchase
of oil.

Loss on call option 6,000


Call options 6,000
Record the decrease in the time value
of the options to current earnings.
The options have expired.

Call options 20,000


Other comprehensive income 20,000
Record the increase in the intrinsic value
of the options to other comprehensive
income.

Cash 30,000
Call options 30,000
Record the sale of the call options.

Oil inventory 330,000


Cash 330,000
Record the purchase of 10,000 barrels
of oil at the spot price of P33 per barrel.

d. June 1, 2011, entries to record the sale of the oil and other entries:

Cash 340,000
Sales 340,000
Record the sale of 10,000 barrels
of oil at P34 per barrel

Cost of goods sold 330,000


Oil inventory 330,000
Recognize the cost of the oil sold.

OCI 30,000
Cost of goods sold 30,000
Reclassify into earnings the other
comprehensive income from the
cash flow hedge.

136
137
CHAPTER 20

MULTIPLE CHOICES - COMPUTATIONAL

20-1: b

Bad debt expense (S$ 6,000 x P28.20) P169,200


Amortization of patents (S$ 4,000 x P28.20) 112,800
Rent expense (S$ 10,000 x P28.20) 282,000
Total P564,000
Average rate (P28.20) is used to translate all expenses since this is a reasonable
estimation.

20-2: b

Machinery [(24,000 Ringgit ÷ 10) x P10.42] P 25,008


Equipment [(12,000 Ringgit ÷ 10) x P10.42] 12,504
Total depreciation P 37,512

20-3: d

Accounts receivable P120,000


Prepaid expenses 55,000
Property and equipment (net) 275,000
Total P450,000

20-4: a

Depreciation expense (H$ 12,000 x P5.80) P 69,600


Bad debts (H$ 8,000 x P5.80) 46,400
Rent (H$ 20,000 x P5.80) 116,000
Total P232,000

Average rate for the year is used in translating depreciation expense because this is more
reasonable estimation than the rate when the related asset was acquired (P4.80).

20-5: d

[25,000 LCU x (1 ÷ 2)]

20-6: d

Long-term receivable: [1,500,000 LCU x (1 ÷ 1.5 LCU)] P1,000,000

Long-term debt: [2,400,000 LCU x (1 ÷ 1.5 LCU)] P1,600,000

20-7: b (NT Dollar 10,000 x P1.70)

136
20-8: b

Beginning inventory 40,000 Rupee


Purchases 300,000
Goods available for sale 340,000
Ending inventory 30,000
Cost of goods sold 310,000 Rupee

Translated cost of goods sold (310,000 Rupee x P.5745) P178,095

20-9: c

NZ Dollar Rate Phil Peso


Net assets, 1/1/05 20,000 P15 P300,000
Increase in net assets:
Net income, 2005 (30,000 – 20,000) 10,000 P19 190,000
Net assets 12/31/05 30,000 P490,000
Net assets at current rate 30,000 P21 630,000
Translation adjustment, 2005 (positive) P140,000

20-10: b

Equipment [800,000 x (1 ÷ 50)] P16,000

Accumulated depreciation [560,000 x (1 ÷ 50)] P11,200

Depreciation [80,000 x (1÷ 50)] P 1,600

20-11: a (25,000 Rupee x P1.24)

20-12: d (5,000 Rupee x P1.30)

20-13: c

Investment cost, Jan. 1, 2005 P402,000


Less: Book and fair value of net assets acquired
(300,000 Rp x P1.20) 360,000
Goodwill P 42,000

Pesos Rupee
Goodwill P42,000 35.000 (P42,000 / P1.20)
Impairment 4,340 (3,500 Rp x P1.24) 3,500
Balance P37,660 31,500

Translated goodwill (31,500 Rp x P1.32) P41,580

137
20-14: b

Translation adjustment from translating the trial balance P12,000 Cr


Translation adjustment from translating goodwill (per 20-13) 3,920 Cr
Total translation adjustment P15,920

20-15: b

Investment in Subsidiary account, Jan. 1, 2011 P1,600,000


Share in subsidiary net income [(800,000 yen x 70%) x P.57] 319,200
Translation adjustment (P25,000 x 70%) 17,500
Share of subsidiary dividends [(50,000 yen x 70%) x P.59] ( 20,650)
Investment in Subsidiary account, December 31, 2011 P1,916,050

20-16: d

20-17: a
Phil Peso Thailand Baht
Initial inventory transfer date:
Selling price P120,000÷1.60 75,000 B
Cost (80,000)
Profit 40,000

Ending Inventory (75,000 x 1.70) 127,500

20-18: a (P127,500 – 40,000)

20-19: a
Yen Exchange Rate Phil Peso
Net asset beginning 200,000 .44 88,000
Net income 200,000 .46 92,000
Net asset translated at rate:
During the year 400,000 180,000
At end of year 400,000 .48 192,000

OCI - Translation adjustment (credit) (12,000)

20-20: a (70,000 rupee x P1.50)

20-21: c
Investment cost P1,210,000
Book value of interest acquired (1,100,000 x 1.10) x .80 968,000
Goodwill 242,000

138
PROBLEMS
Problem 20-1

a.
Pilipino Company
Translation Working Paper
December 31, 2011

Yen Exchange Rate Phil. Pesos


Cash 40,000 .40 CR 16,000
Accounts receivable 120,000 .40 CR 48,000
Inventory 100,000 .40 CR 40,000
Plant and equipment 700,000 .40 CR 280,000
Cost of sales 360,000 .425 AR 153,000
Operating expenses 140,000 .425 AR 59,500
Depreciation expenses 60,000 .425 AR 25,500
Total 1,520,000 622,000
Accumulated Other Comprehensive Income -
Translation Adjustment 25,000
Total debits 647,000

Accumulated depreciation 240,000 .40 CR P96,000


Accounts payable 80,000 .40 CR 32,000
Common stock 200,000 .44 HR 88,000
Retained earnings, Jan. 1 400,000 .44 HR 176,000
Sales 600,000 .425 AR 255,000
Total credits 1,5200,000 647,000

CR – Current Rate
AR – Average Rate
HR – Historical Rate

b. Proof of Translation Adjustment

Yen Translation Rate Phil. Pesos


Net assets at beginning of year 600,000 .44 264,000
Adjustment for changes in net assets
Position during year
Net income for the year 40,000 .425 17,000
Net assets translated at rates in effect
For those items 281,000
Net assets at end of year 640,000 .40 256,000
Change in translation adjustment during year
(to OCI) – net decrease (debit) 25,000

Accumulated OCI – translation adjustment,1/1 -0-

Accumulated OCI – translation adjustment,


Dec. 31 (debit) 25,000

139
Problem 20-2

(1) Trial Balance Translation


Thailand Translation Philippine
Baht Rate Pesos
Cash 7,000 1.60 CR 11,200
Accounts receivable (net) 20,000 1.60 CR 32,000
Receivable from Davao 5,000 1.60 CR 8,000
Inventory 25,000 1.60 CR 40,000
Plant and equipment 100,000 1.60 CR 160,000
Cost of goods sold 70,000 1.50 AR 105,000
Depreciation expense 10,000 1.50 AR 15,000
Operating expenses 30,000 1.50 AR 45,000
Dividends paid 15,000 1.54 HR 23,000
Total debits 282,000 439,300

Accumulated depreciation 10,000 1.60 CR 16,000


Accounts payable 12,000 1.60 CR 19,200
Bonds payable 50,000 1.80 CR 80,000
Common stock 60,000 1.46 HR 87,600
Sales 150,000 1.50 AR 225,000
Total 282,.000 427,800
Accumulated other comprehensive
Income – Translation adj. (credit) 11,500
Total credits 439,300

CR – Current Rate
AR – Average Rate
HR – Historical Rate

(2) Proof of Translation Adjustment

Thailand Translation Philippine


Baht Rate Pesos
Net assets at beginning of year 60,000 1.46 87,600
Adjustments for changes in net
asset position during year:
Net income for year (sch. 1) 40,000 1.50 60,000
Dividends paid (15,000) 1.54 (23,100)
Net assets translated at:
Rates during year 124,500
Rates at end of year 85,000 1.60 136,000
Change in OCI – translation adj.
during year – Net increase 11,500
Accumulated OCI – translation
adjustment – Jan. 1 -0-
Change in OCI – translation
adjustment, Dec. 31 (credit) 11,500

140
Problem 20-2, continued:
Schedule 1:
Sales 150,000 Thailand Baht
Cost of goods sold ( 70,000)
Depreciation expense ( 10,000)
Operating expenses ( 30,000)
Net income 40,000 Thailand Baht

(b) The change in the translation adjustment of P11,500 is included as a credit in the other
comprehensive income on the Statement of Comprehensive Income. The other comprehensive
income is then accumulated and reported in the stockholders’ equity section of the consolidated
balance sheet as presented below:

Net assets P136,000

Common stock P 87,600


Retained earnings, Dec. 31 36,900
Accumulated Other Comprehensive Income 11,500
Total P136,000

Problem 20-3

a. Translation Work paper

Exchange Philippine
Brunei $ Rate Pesos
Cash 1.600 33 CR 52,800
Accounts receivable 2,500 33 CR 82,500
Inventory 4,000 33 CR 132,500
Plant and equipment 35,000 33 CR 1,155,000
Cost of sales 17,000 31 AR 527,000
Operating expenses 7,000 31 AR 217,000
Depreciation expense 3,000 31 AR 93,000
Dividends 1,500 32 HR 48,000
Total debits 71,600 2,307,300

Accumulated depreciation 9,000 31 AR 297,000


Accounts payable 2,600 33 CR 85,800
Common stock 20,000 30 HR 600,000
Retained earnings, Jan. 1 10,000 30 HR 300,000
Sales 30,000 31 AR 930,000
Total 71,600 2,212,800
Accumulated OCI – Translation
Adjustment 94,500
Total credits 2,307,300

141
Problem 20-3, continued:
Proof of Translation Adjustment (not required)

Translation
Brunei $ rate Philippine Pesos
Net assets at beginning of year 30,000 30 900,000
Adjustment for net assets position
during the year:
Net income 3,000 31 93,000
Dividends paid (1,500) 32 (48,000)
Net assets translated at rates
in effect for those items 945,000
Net assets at end of year 31,500 33 1,039,500
Change in translation adjustment during
Year to OCI – net increase (credit) 94,500
Accumulated OCI – translation adj. 1/1 -0-
Accumulated OCI – translation
Adjustment – 12/31 (credit) 94,500

b. Parent Company entries affecting Investment in Moslem Co. (equity method)

Jan. 2: Investment in Moslem Co. 900,000


Cash 900,000
To record investment cost.

Oct. 15: Cash 48,000


Investment in Moslem Co. 48,000
To record dividends received

Dec. 31: Investment in Moslem Co. 93,000


Investment income 93,000
To record equity in income of Moslem

Investment in Moslem Co. 94,500


Other Comprehensive Income – Translation
adjustment 94,500
To record parent’s share of change in translation
Adjustment

142
Problem 20-4

UK Company
Translation Working Paper
Year Ended December 31, 2011

Exchange In
In Pounds Rate Phil. Pesos
Income Statement
Sales 90,000 P67.50 (A) 6,075,000
Cost of sales (80,000) 67.50 (A) (5,400,000)
Depreciation expense (1,500) 67.50 (A) (101,250)
Other expenses (5,750) 67.50 (A) (388,125)
Net income carried forward 2,750 185,625

Retained Earnings Statement


Balance, 1/1 2,500 B 119,500
Net income from above 2,750 F 185,625
Balance, 12/31 5,250 305,125

Statement of Financial Position


Cash 2,500 67.60 (C) 169,000
Accounts receivable 4,000 67.60 (C) 270,400
Inventories, at cost 5,500 67.60 (C) 371,800
Prepaid expenses 750 67.60 (C) 50,700
Property, plant and equipment (net) 9,000 67.60 (C) 608,400
Total assets 21,750 1,470,300

Accounts payable 3,500 67.60 (C) 236,600


Current portion of long-term debt 500 67.60 (C) 33,800
Long-term debt 7,500 67.60 (C) 507,000
Capital stock 5,000 67.20 (H) 336,000
Retained earnings from above 5,250 1,418,525
Total
Cumulative translation adjustment:
Balance, 1/1 50,000
Current translation adjustment G 1,775
Balance, 12/31 51,775
Total liabilities and stockholders’ equity 21,750 1,470,300

Translation Code:
C = Current rate
H = Historical rate
A = Average rate
B = Balance in Philippine pesos at the beginning of the year.
F = Per Income Statement

143
Problem 20-5

Goodluck Corporation
Foreign Exchange Translation Worksheet
Year Ended December 31, 2011

Trial Trial Income State.


Balance Exchange Balance Statement of FP
(In Pounds) Rate (In Pesos) (In Pesos) (In Pesos)
Cash 15,000 0.95 C 14,250 14,250
Marketable securities 25,000 0.95 C 23,750 23,750
Accounts receivable 60,000 0.95 C 57,000 57,000
Inventories 80,000 0.95 C 76,000 76,000
Property, plant and equip-net 420,000 0.95 C 399,000 399,000
Cost of goods sold 150,000 0.90 A 135,000 135,000
Depreciation expense 40,000 0.90 A 36,000 36,000
Other expenses 10,000 0.90 A 9,000 9,000
Totals 800,000 750,000 180,000 570,000

Accounts payable 50,000 0.95 C 47,500 47,500


Current portion of LT debt 40,000 0.95 C 38,000 38,000
Long-term debt 120,000 0.95 C 114,000 114,000
Sales 200,000 0.90 A 180,000 180,000
Other revenues 50,000 0.90 A 45,000 45,000
Capital stock 250,000 0,87 H 217,500 217,500
Retained earnings, 1/1 90,000 G 70,000 70,000
FC translation adjustment
Balance, 1/1 G 1,500 1,500
Current year B 36,500 36,500
Net income B (45,000) 45,000

Totals 800,000 750,000 180,000 570,000

Translation Code:
A = Average rate
B = Current rate
H = Historical rate
G = Given
B = Balancing amount

Problem 20-6

a. Direct and indirect exchange rates

Direct A$ Indirect
January 1, 2010 P.03333=1 A$30=P1
December 31, 2010 P.02857=1 A$35=P1
December 31, 2011 P .025=1 A$40=P1

144
Problem 20-6, continued:
The peso strengthened during 2010 because the number of A$ one Phil. Peso could acquire
at the end of the year (35) is greater than the number of A$ that could be acquired at the
beginning of the year (30); therefore, the value of the peso has increased relative to the A$
during 2010. The peso continued to strengthen during 2011.

b. Translated December 31, 2010, Statement of Financial Position:


Subsidiary’s Direct Translated
Trial Balance Exchange Trial Balance
_ (in A$)__ Rate ( in $)___
Cash A$ 100,000 P.02857 P 2,857
Receivables 400,000 P.02857 12,857
Inventory 680,000 P.02857 19,428
Fixed assets 1,000,000 P.02857 28,570
Total R 2,230,000 P 63,712
Accumulated OCI-
translation adjustment (debit) 2,903
Total debits P 66,615

Current payables A$ 260,000 P.02857 P 7,428


Long-term debt 1,250,000 P.02857 35,713
Common stock 500,000 P.03333 16,665
Retained earnings 220,000 P.03333 6,809
Total credits A$2,230,000 P 66,615

P.03333= average of beginning and ending exchange rates, rounded to 4 decimal points:
P.030945= [(P.03333 + P.02856) /2]

(Not required: Proof of translation adjustment (debit) of P 2,903)

Translation
___A$___ _ Rate_ _Dollars_
Net assets, 1/1/10 A$ 500,000 P.03333 P 16,665
Adjustment for changes in
net assets during year:
Net income 220,000 P.03095 6,809
Net assets translated at:
Rates during year P 23,474
Rates at end of year A$ 720,000 P.02857 (20,570)
Change in translation
Adjustment during year (debit) P 2,904*

*Difference of P1 (P 2,904 – P 2,903) due to rounding of exchange rates.

145
Problem 20-6, continued:
c. Translated December 31, 2011, statement of financial position:

Subsidiary’s Direct Translated


Trial Balance Exchange Trial Balance
(in A$) __Rate (in P)__
Cash A$ 80,000 P.025 P 2,000
Receivables 550,000 P.025 13,750
Inventory 720,000 P.025 18,000
Fixed assets 900,000 P.025 22,500__
Accumulated other A$ 2,250,000 P56,250
comprehensive income-
translation adjustment (debit) 5,635___
Total debits P61,885

(a)The retained earnings in pesos would begin with the December 31, 2010, peso balance
(P6,809) that would be carried forward. To this would be added 2011’s net income of
A$90,000, which is the change in retained earnings in A$ multiplied by the 2011 exchange
rate of P.02679 [(P.02857 + P.025/2)] which equals P2, 411. Therefore, translated retained
earnings on December 31, 2011, is P9, 220 (P9, 220= P6, 809 + P2, 411)

(Not required: Proof of translation adjustment (debit) of P5, 635)

Australian Translation
Dollar _ Rate Pesos___
Net assets, 1/1/11 A$ 720,000 P.02857 P20, 570
Adjustment for changes in
net assets during year:
Net income 90,000 P.02679 2,411___
Net assets translated at:
rates during year P22, 981
Other comprehensive income-
rate at end of year A$ 810,000 P.025 (20,250)__
Change in other comprehensive
income- translation
adjustment during year (debit) P2, 731
Accumulated other comprehensive
income- translation adjustment, 1/1/11 2,904___
Accumulated other comprehensive
income- translation adjustment, 12/31/11 (debit) P5, 635

d. The P2, 731 change in the accumulated other comprehensive income- translation
adjustment during 2011 would be reported as a component of other comprehensive
income on 2011 statement of other comprehensive income.

146
147
CHAPTER 21

MULTIPLE CHOICES - COMPUTATIONAL

21-1 b

21-2 a

21-3 a

21-4 b

21-5 b

21-6 a

21-7 c

21-8 a

21-9 a

21-10 c

21-11 d

21-12 b

21-13 b

21-14 a

21-15 a

Excess of income over expenses P 200


Depreciation 70
Increase in due from national government agencies ( 10)
Increase in prepaid rent ( 15)
Increase in accounts payable 30
Cash provided by operating activities P 275

147
PROBLEMS
Problem 21-1

1. Memo entry in the RAOPS, RAOMO, RAOCO and RAOFE.

2. Cash – National Treasury – MDS 2,000,000


Subsidy Income from National Government 2,000,000

3. Memo entry in the RAOPS, RAOMO, RAOCO and RAOFE.

4. Office equipment 50,000


Accounts payable 50,000

5. Cash – Disbursing Officer 40,000


Cash – National Treasury – MDS 40,000

6. Salaries and wages – Regular 44,000


Personal economic relief allowance (PERA) 3,000
Additional compensation 3,000
Due to BIR 3,500
Due to GSIS 5,500
Due to Pag-ibig 400
Due to Philhealth 600
Cash – Disbursing Officer 40,000

7. Due to GSIS 5,500


Due to Pag-ibig 400
Due to Philhealth 600
Cash – National Treasury – MDS 6,500

8. Life and retirement contribution 5,500


Pag-ibig contribution 400
Philhealth contribution 600
Cash – National Treasury – MDS 6,500

9. Electricity 5,000
Telephone expense – Landline 4,000
Accounts payable 50,000
Due to BIR 5,000
Cash – National Treasury – MDS 54,000

10. Due to BIR 4,500


Subsidy income from national government 4,500

11. Cash – Collecting Officer 90,000


Sales revenue 40,000
Permit fees 30,000
Miscellaneous income 20,000

12. Cash in Bank – Local currency – Current account 90,000


Cash – Collecting Officer 90,000

148
Problem 21-2

Building

1. Memo entry in RAOCO.

2. Advances to contractor 240


Cash – National Treasury – MDS 240

3. Construction in progress – Other Public Infrastructure 400


Advances to contractor 240
Accounts payable 160

4. Accounts payable 160


Due to BIR 40
Cash – National Treasury – MDS 120

5. Construction in progress – Other Public Infrastructure 400


Accounts payable 400

6. Accounts payable 400


Due to BIR 40
Cash – National Treasury 360

7. Due to BIR 80
Subsidy income from national government 80

8. Office Building 800


Construction in progress – OPI 800

Repairs of Building

1. Memo entry in RAOCO.

2. Construction materials inventory 70


Accounts payable 70

3. Accounts payable 70
Due to BIR 7
Cash – National Treasury – MDS 63

4. Construction in progress – Other Public Infrastructure 60


Construction materials inventory 60

5. Memo entry in the RAOCO

149
Problem 21-1, continued:
6. Cash – Disbursing Officer 36
Cash – National Treasury – MDS 36

7. Construction in progress – Other Public Infrastructure 40


Due to BIR 4
Cash – Disbursing Officer 36

8. Due to BIR 47
Cash – National Treasury – MDS 47

9. Office building 100


Construction in progress – OPI 100

Land:

1. Memo entry in the RAOCO, P100.

2. Land 100
Accounts payable 100

3. Accounts payable 100


Due to BIR 10
Cash – National Treasury – MDS 90

150
Problem 21-3

(a) Journal Entries:

1. Memo entry in the Registry of Obligations and Allotments.

2. Cash – National Treasury – MDS 2,500


Subsidy income from national government 2,500

3. Memo entry in Registry of Obligations and Allotments.

4. Office equipment 120


Accounts payable 120

5. IT equipment and software 30


Accounts payable 30

6. Prepaid rent 60
Cash – National Treasury – MDS 60

7. Electricity expense 50
Cash – National Treasury – MDS 50

8. Telephone expense – Landline 40


Cash – National Treasury – MDS 40

9. Petty cash fund 45


Cash – National Treasury – MDS 45

10. Accounts payable 120


Due to BIR 12
Cash – National Treasury – MDS 108

11. Accounts payable 30


Cash – National Treasury – MDS 30

12. Due to BIR 12


Subsidy income from national government 12

13. Cash – Collecting Officer 50


Other service income 10
Sales revenue 40

14. Cash in bank – LCCA 50


Cash – Collecting Officer 50

151
Problem 21-3, continued:
(b) Pre-closing Trial Balance

Petty cash fund 45


Cash – National Treasury – MDS 2,167
Cash in Bank – Local Currency – Current Account 50
Prepaid rent 60
Office equipment 120
IT equipment and software 30
Other service income 10
Sales revenue 40
Subsidy income from national government 2,512
Electricity expense 50
Telephone expense – landline 40
Total 2,562 2,562

(c) Adjusting Entries

(1) Depreciation – Office equipment & software 20


Depreciation – IT equipment 5
Accumulated depreciation – Office equipment 20
Accumulated depreciation – IT equip & software 5

(2) Rent expense 30


Prepaid rent 30

Closing Entries:

(1) Unused National Clearing Account (NCA)


Subsidy income from national government 2,167
Cash – National Treasury – MDS 2,167

NCA received during the year 2,500


Less: MDS check issued 333
Unused NCA 2,167

(2) Income accounts:


Other service income 10
Sales revenue 40
Subsidy income from national government 345
Income and expense summary 395

(3) Expense accounts:


Income and expense summary 90
Electricity expense 50
Telephone expense – landline 40

(4) Income and expense summary 305


Retained operating surplus 305

(5) Retained operating surplus 305


Government equity 305

152
Problem 21-4

Agency VV
Statement of Income and Expenses
Year Ended December 31, 2011

Income:
Subsidy income from national government P1,700
Less: Reversion of unused NCA 800 P900

Less: Expenses
Salaries and wages – Regular P 320
Personnel Economic Relief Allowance 40
Additional compensation 40
Life and retirement insurance contribution 60
Pag-ibig contribution 10
Philhealth contgribution 10
Traveling expense – Local 35
Office supplies expense 60
Electricity expense 75
Telephone expense – landline 45
Janitorial services 30
Security services 35
Repairs and maintenance – Office building 65
Depreciation – Office building 15
Depreciation – office equipment 10
Depreciation – furniture and fixtures 5
Depreciation – IT equipment and software 5 860
Net income over expenses P 40

153
Agency VV
Balance Sheet
As of December 31, 2011

ASSETS
Current Assets
Cash:
Cash in vault P 200
Cash – collecting officer 500
Cash – disbursing officer 1,000
Petty cash fund 150
Cash in bank – LCCA 350 P2,200
Receivables:
Accounts receivable P 120
Less: Allowance for doubtful accounts 20 100
Inventories:
Office supplies inventory 30
Other current assets 15
Long-term investment:
Investment in stock 400
Property, Plant and Equipment:
Land 600
Office building 650
Less: accumulated depreciation 50 600
Office equipment 250
Less: accumulated depreciation 20 230
Furniture and fixtures 110
Less: accumulated depreciation 10 100
IT equipment and software 190
Less: accumulated depreciation 25 165 1,695
Total assets 4,440

LIABILITIES AND EQUITY


Liabilities
Current liabilities
Accounts payable 185
Due to BIR 50
Due to GSIS 30
Due to Pag-ibig 25
Due to Philhealth 25
Other payables 15 330
Equity:
Government equity 4,110
Total liabilities and equity 4,440

154
155
CHAPTER 22

MULTIPLE CHOICES - COMPUTATIONAL


22-1: b. (P500,000 – P300,000)

22-2: d.

The total tuition fees for educational and general purposes.

22-3: d. (P1,240,000 – P160,000)

22-4: a.

Unrestricted cash contribution received from donors are to be reported as increase in net cash
provided by operation.

22-5: d.

The remaining contribution of P5,000 on December 31, 2011.

22-6: b.

Unregistered pledges from donors are treated as revenues at the time of the pledge.

22-7: d.

Patient revenues P 5,000,000


Nursing services 1,000,000
Professional services 500,000
Total revenues P 6,500,000
Less: Staff discounts P50,000
Allowances 230,000
Third party payors 800,000 1,080,000
Net revenues P 5,420,000

Bad debts are treated in the usual manner as expense.

22-8: a.

As of July 31, 2011, all of the funds are properly includible in the Plan Funds, for a total of
P900,000.

22-9: c. (P800,000 – P110,000)

22-10: d.

Patient revenues (net of charity care) P 600,000


Less: contractual adjustments 200,000
Net patient service revenues P 400,000

22-11: c.

155
22-12: c.

The contributed services are debited to Salary Expense account and credited to Contribution
Revenue account.

22-13: c.

The net effect on unrestricted net assets of spending P10,000 on research is zero.

22-14: b. (P5,000,000 + P50,000)

The P1,000,000 contribution from the donor, who stipulated that the contribution be invested
Indefinitely, should be reported as permanently restricted revenue.

22-15: c.

22-16: b.

Both are treated as a financing activity on the statement of cash flows.

22-17: a.

Cash flows from operating activities would include both the cash received from patient service
Revenue of P300,000 and the cash received from gift shop sales of P25,000.

22-18: b.

Cash received from patient revenue (collection of receivables) and from tuition revenue are both
included in the amount reported for cash flows from operating activities. The other cash receipts
would be reported as increases in cash flows provided by financing activities.

22-19: b.

Expirations of donor restrictions on temporarily restricted net assets should be reported on the
Statement of operations as net assets released from restrictions.

22-20: c.

Current funds revenues include (1) all unrestricted gifts and other unrestricted resources earned
during the reporting period, and (2) restricted current funds to the extent that such funds were
expended for current operating purpose. Therefore, the amount that should be included in current
funds revenue is:

Unrestricted gifts received:


Expended P600,000
Not expended 75,000
Restricted gifts received
Expended 100,000
Total P775,000

156
Problems

Problem 22-1

1. Pledges receivable 300,000


Allowance for uncollectible pledges 10,000
Contribution revenue 270,000

2. Cash 260,000
Pledges receivable 260,000

3. Cash 40,000
Fund raising expense 5,000
Fund raising revenue 45,000

4. Investment 35,000
Cash 35,000

5. Cash 5,000
Sales – public revenue 5,000

6. Salaries 90,000
Employee fringe benefits 15,000
Payroll taxes 16,000
Supplies 7,000
Telephone 1,500
Utilities 6,000
Rent 10,000
Conference, conventions and meetings 5,000
Cost of sales to public 1,000
Miscellaneous 3,000
Cash 154,500

7. Utilities 1,000
Salaries 5,000
Accounts payable or accrued expense payable 6,000

8. Fund Balance - Unrestricted 10,000


Fund balance – Restricted to purchases
of new equipment. 10,000

157
Problem 22-2

(1) Accounts receivable 80,000


Patient service revenues 80,000
To record gross patient service revenue for the month
at full rates.

Accounts receivable 2,500


Patient service revenues 2,500
To record receivable from Social Medicare.

Contractual adjustments 6,000


Accounts receivable 6,000
To record contractual adjustments allowed.

Doubtful accounts 8,000


Allowance for doubtful accounts 8,000
To provide allowances for doubtful accounts.

(2) Salaries expense 9,800


Contribution revenues 9,800
To record donated services (10,000 – 200).

(3) Pledges receivable 5,000


Contribution revenues 5,000
To record pledges received from donors.

Cash 3,500
Pledges receivable 3,500
To record pledges collected.

Provisions for doubtful pledges 800


Allowance for doubtful pledges 800
To provide doubtful pledges.

(4) Cash 3,000


Fund balance 3,000
To record receipt of cash from restricted fund.

Plant assets 3,000


Cash 3,000
To record acquisition of new surgical equipment.

158
Problem 22-3

Plant Fund Ledger Account:

(1) Equivalent 50,000


Fund Balance 50,000
To record acquisitions of computers from unrestricted fund.

(2) Buildings 2,000,000


Cash 250,000
Mortgage notes payable 1,750,000
To record construction of new building financed in
part by 5% mortgage note payable.

Quasi-Endowment Fund Ledger Account:

(3) Cash 110,000


Investments 100,000
Payable to Unrestricted fund 10,000
To record sale of investments at a gain, the use of
which is unrestricted.

Unrestricted Fund Ledger Account:

(1) Undesignated fund balance 50,000


Cash 50,000
To record acquisitions of computers to be carried in
Plant Fund.

(2) Cash 2,000,000


Contribution revenues 2,000,000
To record receipt of unrestricted gift.

(3) Receivable from quasi-endowment fund 10,000


Investment income 10,000
To record investment gain receivable.

159
Problem 22-4

Nonprofit Trade Association


Statement of Activities
Year Ended June 30, 2011

Revenues and Gains:


Membership dues P184,000
Conferences and meetings 321,000
Publications and advertising sales 143,000
Special assessments 50,000
Investment income, including net gains 11,000
Total P709,000
Expenses:
Member services P 56,000
Conferences and meetings 166,000
Technical services 218,000
Communications 61,000
General administration 154,000
Membership development 27,000 682,000
Increase in unrestricted net assets 27,000
Net assets, beginning of year 285,000
Net assets, end of year P312,000

Nonprofit Trade Association


Statement of Financial Position
June 30, 2011

ASSETS
Current assets
Cash P 7,000
Short-term investments 217,000
Accounts receivable (net) 25,000
Publications inventory 61,000
Total current assets 310,000
Long-term investments 120,000
Plant assets (net) 33,000
Other assets 28,000
Total assets P491,000

LIABILITIES AND NET ASSETS


Current liabilities
Accounts payable and accrued liabilities P 48,000
Deferred membership dues 131,000
Total current liabilities 179,000
Net assets (unrestricted) 312,000
Total liabilities and net assets P491,000

160
Problem 22-5

Children Association
Statement of Activities
Year Ended December 31, 2011

Changes in unrestricted net assets:


Revenues and gains:
Contributions P320,000
Membership dues 25,000
Program service fees 30,000
Investment income 10,000
Total unrestricted revenues and gains P385,000
Expenses:
Programs P270,000
Management and general expenses 47,000
Fund raising 8,000 325,000
Increase in unrestricted net assets P 60,000
Changes in temporarily restricted net assets:
Contributions P 15,000
Expenses:
Management and general expenses P 4,000
Fund raising expenses 1,000 5,000
Increase in temporarily restricted net assets P 10,000
Increase in net assets P 70,000
Net assets, beginning of year (P12,000 + P26,000 + P3,000) 41,000
Net assets, end of year P111,000

Children Association
Statement of Financial Position
December 31, 2011

ASSETS
Cash (P40,000 + P9,000) P 40,000
Bequest and interest receivable (P5,000 + P1,000) 6,000
Pledges receivable (net) (P12,000 – P3,000) 9,000
Investments, at cost 100,000
Total assets P164,000

LIABILITIES AND NET ASSETS


Liabilities P 51,000
Accounts payable and accrued liabilities (P50,000 + P1,000) 2,000
Deferred revenues P 53,000
Total liabilities
Net assets:
Unrestricted (P38,000 + P60,000) P 98,000
Temporarily restricted (P3,000 + P10,000) 13,000
Total net assets P111,000
Total liabilities and net assets P164,000

161
Problem 22-6

San Pedro Hospital


Statement of Financial Position
June 30, 2011

ASSETS
Current assets
Cash P 222,000
Accounts receivable (net of allowance of P5,000) 20,000
Inventories 50,000
Prepaid expenses 10,000
Total current assets P 302,000
Investments 660,000
Property, plant and equipment (net of accumulated depreciation of P140,000) 160,000
Total assets P1,122,000

LIABILITIES AND NET ASSETS


LIABILITIES
Current liabilities:
Accounts payable P 45,000
Accrued expenses 17,000
Deferred revenues 11,000
Current portion of long-term debt 24,000
Total current liabilities P 97,000
Mortgage payable 125,000
Total liabilities P 222,000
NET ASSETS
Unrestricted P 148,000
Temporarily restricted 232,000
Permanently restricted 520,000
Total net assets P 900,000
Total liabilities and net assets P1,122,000

162

You might also like